You are on page 1of 184

CSEC Physics Past Papers 1990 to 2011

Year
Jun 11 P2
Jan 11 P2
Jun 10 P2
Jan 10 P2
Jun 09 P2
Jan 09 P2
Jun 08 P2
Jan 08 P2
Jan 08 P3
Jun 07 P2
Jun 07 P3
Jun 06 P2
Jun 06 P3
Jun 05 P2
Jun 05 P3
Jun 04 P2
Jun 04 P3
Jun 03 P2
Jun 03 P3
Jun 02 P2
Jun 02 P3

Page
1
7
11
15
22
18
26
28
31
34
38
40
43
45
48
55
57
50
53
60
64

Year
Jun 01 P2
Jun 01 P3
Jun 00 P2
Jun 00 P3
Jun 99 P2
Jun 99 P3
Jun 98 P2
Jun 98 P3
Jun 97 P2
Jun 97 P3
Jun 96 P2
Jun 96 P3
Jan 07 P2
Jan 07 P3
Jan 06 P2
Jan 06 P3
Jan 05 P2
Jan 05 P3
Jan 04 P2
Jan 04 P3
Jan 03 P2

Page
66
70
72
76
78
82
84
89
91
93
97
101
103
105
107
110
113
115
119
121
123

Year
Jan 03 P3
Jan 02 P2
Jan 02 P3
Jan 01 P2
Jan 01 P3
Jan 00 P2
Jan 00 P3
Jan 99 P2
Jan 99 P3
Jun 95 P2
Jun 95 P3
Jun 94 P2
Jun 94 P3
Jun 93 P2
Jun 93 P3
Jun 92 P2
Jun 92 P3
Jun 91 P2
Jun 91 P3
Jun 90 P2
Jun 90 P3

Page
126
128
132
134
137
140
145
147
153
155
157
160
162
164
165
168
171
174
176
178
181


June 2011

and use the graph to determine the total distance travelled.

[4]

SECTION A
Attempt ALL questions.
You MUST write your answers in this answer booklet.
1. A taxi driver was taking his friends to a popular 20/20 cricket match. Table 1
shows the vehicle's velocity for the first minute of the journey.

Velocity, V/ms

-1

Time, t/s

0.0

0.0

11.0

10.0

23 .0

20.0

34 .0

30.0

47 .0

40.0

57.0

50.0

69.0

60.0
Table 1

(a) Plot on page 3, a graph of velocity (V) versus time (t).


(b) From the graph, calculate the slope, p.

[7]

[4]
(c) What quantity does the slope, p, represent?
[1]
(d) For the next minute, the vehicle's velocity was constant, then for the
final minute, the vehicle decelerated uniformly to a stop on reaching the
stadium.
(i) Continue the velocity-time graph on page 3, to show this information

(ii) Calculate the average velocity of the taxi for the complete journey.

[2]

(iii) If the total mass of the loaded taxi was 1 500 kg, determine the
momentum of the vehicle when it is travelling at constant velocity.


[3]

(e) (i)Define the term 'displacement'.



[2]

(ii) Complete Table 2 by ticking () the appropriate column that


represents the
quantity, given in the table.





[2]
TABLE 2

present temperature. Assume no heat losses.


Quantity

Scalar

Vector

Displacement
Acceleration

Total 25 marks

2. (a) (i)Complete Table 3 by inserting the correct symbol and Sl Unit which
relate to the quantity shown in Column 1.
[3]
TABLE 3

[Specific Heat Capacity of water = 4200 J Kg-1K-1


Specific Latent Heat of Fusion of lce = 340 000 J Kg-1]

(ii)
If
this
Specific Heat Capacity
c
melt
ing
and
Specific Latent Heat of
heat
ing
Vaporization
acti
vity took place over 300 s, calculate the rate at which the ice I water
was receiving heat.

[3]
Quantity

(ii) Define the term 'heat capacity of a substance' .



[2]

Symbol

SI Unit

(iii) Write the equation that relates specific heat capacity with heat
capacity.[1]

(b) A busy housewife left 25 g of ice in an open insulated container while


she was answering her cellphone. When she returned, the ice at 0 C was
converted to water at 3.0 "C.
(i) Calculate the energy needed for the ice to totally melt and to reach its

Total 15 marks

(iii) Name the material used for the negative terminal in a dry cell.

[1]

3. Figure 1 shows a simple cell connected to a circuit.


(iv) In the circuit in Figure 1, if the current is steady at 0.1 A, calculate
the charge that goes through the copper wire in 60 s.

[3]

(v) Determine how much charge goes through the simple cell in 60 s.

[1]

(b) (i) Identify the type of current (d.c. or a.c.) which generated the specific
waveforms A, B and C shown in Table 4.



[3]

(a) (i)How does the flow of the current within the simple cell differ
from that in a copper wire?





[2]

(ii) Name the material used for the positive terminal in a dry cell.

[1]

SECTION B
Attempt ALL questions.
You MUST write your answers in the space pro vided after each question .

4 (a) Eron, a form five Physics student, has available to him a bright
filament light in a ray box, an opaque screen with two closely spaced
narrow slits, and a large white screen.
Describe how he can set up the apparatus to be able to observe an
interference pattern of light and dark bands on the white screen and briefly
explain why this pattern forms.
[6]
(b) Figure 2 shows a narrow beam of white light which is shone onto a
(ii) Determine the period and frequency for Waveform C.

[4]

triangular prism at an angle of incidence of 30.

Total 15 marks

(i) If the angle of refraction for the violet light, v is 20, what is the
refractive index of this prism for violet light?

[3]

(ii) Calculate the speed of the violet light in the prism.

[3]

(iii) Calculate the frequency of violet light if the wavelength in air is 430
nm.
[1 nm = 10-9 m


Velocity of light in air = 3 108 m s-1]

[4]

[3]

(ii) The mass of the sun is lost at the rate of 2.0 x 109 kg s-1 . If the speed of

Total 15 marks

light in a vacuum is 3.0 x 108 m s-1, calculate the power output of the sun in

5. Many modern electrical appliances have been designed to operate with a.c.

kilowatts.

[5]

Total 15 marks

power rather than d.c. power because electrical power is transmitted as a.c.
rather than d.c.
(a)

(i)State two advantages of using a.c. to transmit electrical power.

[2]

(ii) Draw a diagram of a simple transformer indicating the features,


which enhance efficiency.
(b)

[4]

Electrical power produced by Powergen in Trinidad is stepped up from


11 000 V at 8 000 A to 110 000 v for transmission to Tobago.
(i) If the number of turns in the secondary coil is 900, calculate the
number of turns in the primary coil for an ideal transformer
[3]
(ii) Calculate the transmission current for the ideal transformer
referred to (b) (i).

[3]

(iii) Calculate the transmission power if the transformer is 70%


efficient.[3]
Total 15 marks

6. (a) (i)Outline THREE uses of radioisotopes in medicine.


[3]
(ii) State THREE safety precautions to be taken when using radioactive
substances.

[3]

(b) (i) A gram of living plant material with Carbon - 14, decays at a rate of
about 16 disintegrations per minute. When the plant dies, the Carbon - 14
decays, with
a half-life of 5 600 years. A gram of the dead plant is found to decay at
about
1 disintegration per minute. Calculate the probable age of the plant.

(2)

January 2011 Paper 2


(e) Calculate the resultant force acting on the sprinter after 6.0 s.

SECTION A
Attempt ALL questions.
You MUST write your answers in this answer booklet.
1. A popular sprinter of mass 60 kg, was running a 100 m race. Her velocity
was measured over a 10.0 s period. The results are recorded in Table 1.


(3)
(f) Using the graph drawn on page 3, determine how far from the finish
line the sprinter would be after 10.0 s.

(a) Define the following terms:

(5)

Velocity

Total 25 marks
(2)

Acceleration
(2)
(b) Use the results from Table 1 to plot a graph of Velocity versus Time on
page 3.




(7)

2. Electromagnetic waves consist of visible light and radiations higher or


lower than the wavelength of light.
(a) (i) Identify ONE type of radiation with wavelength:
a) longer than visible light _
b) shorter than visible light. _
(2)
(ii) Complete Table 2, which relates to the sources and uses of
electromagnetic waves.
(5)

(c) Determine the slope of the graph over the first 6.0 s of the race.



(4)
(d) How is the slope of the graph related to the acceleration of the
sprinter?

(b) (i) If gamma rays have a wavelength of 3.0 x 10

-12

m, calculate the


frequency of this type of radiation.
[Velocity of all electromagnetic waves =3.0 x 108m s- l]

(3)

(5)

Total 15 marks

(ii) A ray of red light emerges from a glass block as show n in Figure 1.


3. (a)Complete Table 3 which relates electrical circuit symbols to the names
of electrical components .
[6]


(b) Figure 2 shows a circuit diagram.


If the block has a refractive index of 1.5, determine the value of the angle
e.



Figure 2
In the circuit show n in Figure 2, R1= 4 ,R3 = 8 and the supply is 24Y.
With switch S open, the current I through the ammeter is 2 A.
(i) State the formula which relates voltage, V, current, I and resistance , R.
(1)









(3)

Total 15 marks

SECTIONB
Attempt ALL questions.
You MUST write your answers in the space provided after each question.
4. (a) Describe how the apparatus show n in Figure 3 below may be used to
show how the temperature of a substance such as naphthalene , varies during
cooling. Include in your description any other equipment that may be
necessary.


(ii) Calculate the value of R2.

(3)

[2]

(iii) What is the reading on voltmeter V?






(iv) If switch S is closed , what would be the new reading on ammeter, A?

(6)

10


(b) In an experiment to determine the specific latent heat of ice , the following
results were obtained.
Initial mass of water at 30C = 100 g
Final mass of water (initial water plus melted ice) at 20C = 110 g

of the car will affect the charging of the batteries


[3]
(iii) In a certain section of road , 200 KW of power is expended to charge 100
cars If each car converts 600 Watts of power to charging the battery, calculate
the conversion efficiency.
[3]
Total 15 marks
6. (a)Some great scientists contributed to the modern-day view of the model
of the atom. Two of these were J. J . Thompson and Ernest Rutherford.
For EACH of these scientists, describe briefly TWO main ideas they put
forward
concerning the nature of the atom, including the name of EACH of their
models.
[6]
(b) In final revision for your CSEC Physics examinations, your teacher gave
you the following data on some unknown elements:



Assume the container has negligible heat capacity and that the heat lost to the
surroundings is equal to the heat gained from the surroundings during the
experiment.
(i)
Determine the mass of the melted ice.


[1]
(ii)
Calculate the heat lost by the water.


[3]

(iii) Write down an ex press ion for the TOTAL heat gained by the ice in
melting and the melted ice in warming to 20C.


[2]
(iv) Hence calculate the specific latent heat of fusion of ice.
[3]
[Specific Heat Capacity of water = 4.2 J g- l K- 1]

Total 15 marks
5. (a)With the aid of at least one diagram, describe how the magnitude of the
electromotive force (e.m.f.) induced in a conductor depends on the rate of
change of magnetic flux that the conductor experiences.
[6]
(b) To make electric cars more efficient, they will need to be charged
regularly using a magnetic field below the road surface. One solution is to
designate one lane of the highway for recharging batteries.
(i) In the charging lane cars are required to keep moving. Why should this be
so?
[3]
(ii) Assuming the magnetic field is constant, state with reason , how the speed


You are required to determine:
(i) which nuclides have the identical mass number
(ii) how many neutron s are in the heaviest nuclide
(iii) which nuclides are isotopes [5]
(c) In a half-life experiment, the data shown in Table 4 were obtained.
TABLE 4


Use the data to determine:
(i) TWO different values of the half-life of the element
(ii) The average half-life of the element [4]
Total 15 marks



10

11

June 2010 P2
SECTION A
Answer ALL questions.
You MUST write your answers in the spaces provided in this booklet.
1.Testing of a new material to be used as an anti-reflective coating for eye
glasses has yielded the results shown in Table 1.
TABLE 1

(a) Complete Table 1 by calculating the values for sin i and sin P.
(b) Use the readings from the completed Table 1 to plot a graph of sin i
against sin r on the
graph paper on page 2.
(c) Calculate the gradient of the graph.


(4 marks)
(d) State the TWO laws of refraction.


(4 marks)

(iv)
a)
What MAJOR form of energy did BOLT possess when
he crossed the finish line?
(1 mark )
Calculate the value of this energy if his mass was 86 kg.
(2 marks) Total 15 marks
3. (a)
A student connects the circuit in Figure 1 below to investigate
rectification using a single
diode.

(i)

(4 marks)

Sketch the V., t graph when switch S is open.


(1 mark )

(7 marks)

(e)
Calculate the angle of refraction if the angle of incidence is 90 for this new
material. (3 marks)
(f)
The anti-reflective coating works best if its refractive index is the square root
of the refractive index of the lens in the eye glasses. Determine the refractive index
of the lens that gives the best result.
0

(3 marks) Total 25 marks


2.
(i)
(ii)
(iii)

(a) Define EACH of the following terms.


Velocity
(2 marks)
Acceleration
(2 marks)
Linear momentum
(2 marks)

(1 mark)

(ii)

Sketch the V t graph across diode D when switch S is closed.

(b)In 2008, at the Beijing Olympics, Usain Bolt of Jamaica reclaimed his title as the world's
fastest man. He completed the 100 m final in a world record time of
9.69 s. He accelerated uniformly from rest for the first 6.5 seconds,
covering 60 m before coasting at maximum speed to the finish.
(i)
Calculate his average speed for the first 6.5 s.
( 2 marks)
(ii)
What was his maximum speed?

(2 marks)
(iii) What was his acceleration during the first 6.5 s? (2 marks)

11

12








(1 mark)







(iii) Sketch the V t graph across a battery power source.

(8 marks) Total 15 marks





(iv)
(v)


(1 mark)
4.

What is the similarity in the voltage between (ii) and (iii)? ( 1 mark)
How can you use the circuit in Figure 1 to determine whether or not a
semiconductor diode is defective?
(2 marks)

(i)
(ii)

SECTION B
Answer ALL questions.
You MUST write your answers in the space provided after each question.
(a)With reference to the diagram of a simple d.c. motor shown in Figure 2
explain how this motor works ( 5 marks)
give the purpose of the commutator. ( 1 mark )

(b) (i)The truth table of a logic gate is shown below for inputs A and B and output C.

State the type of logic gate.


(1 mark)
(ii)
Complete the truth table for the following logic circuit.

Figure 2. D.C. Motor


(b) A 24 V d.c. motor was used to lift a small appliance of mass 25 kg from the ground
to the

12

13


second floor of a multi-storey car park. The floor is 30 m above the ground. The motor,
operating at 100% efficiency, works at a steady rate and takes 5 s to complete the
activity.
(i) Calculate the power provided by the motor.
(ii) Calculate the current drawn from the d.c. supply.
(iii) If the power required to lift the appliance was greater, what effect would this have
on the value of the current?
(1 mark )
Assume the voltage rating remains constant.
Total 15 marks
[g = 10 N Kg-1]

5. (a) A CSEC Physics teacher gave two groups of students projects on Specific Heat
Capacity.
One group used an electrical method to calculate the specific heat capacity of a metal
block. The circuit used in this method is shown in Figure 3


(4 marks)
(4 marks)

Figure 3
Describe the method used by this group to determine the specific heat capacity of the
metal block.
( 6 marks)
(b) The second group of students determined the specific heat capacity of a liquid. One
set of results is described below:
It took 300 s for the temperature of 0.1 kg of the liquid to be elevated from 25C to
50C. The energy supplied was 13.6 KJ during this period.
(i)
Assuming that no heat was lost and ignoring the heat capacity of the container,
determine the value of the specific heat capacity of the liquid.
(ii) If the liquid was heated twice as long, what impact, if any, would this have on
the specific heat capacity of the liquid? ( 1 mark )
(iii)
Justify your response in part (ii). ( 2 marks)







6.

Total 15 marks

(a) You were asked to present to your class, a comparison of alpha (a) with
gamma (y)

13

14


radiation. Compare these two types of radiation in terms of their
(i)
range in air
(ii) behaviour in an electric field
(iii) type of track in a cloud chamber. ( 6 marks)
(b)
A nuclear scientist proposed the following nuclear reaction based on an
artificial radioactive decay process to produce energy

Table 2 below gives the atomic mass for each nuclide. (u = 1.66 x 10-" Kg.)
Table 2


(i)
(ii)
(iii)

Calculate the number of neutrons in Xenon (Xe).


Determine the energy released in the proposed nuclear reaction.
If the energy released in an alternative, natural decay reaction is 9.98 10-13 J
which would be the preferred method for the production of a nuclear power station?
Justify your choice.
( 2 marks)
(iv)
Give the reason for your decision in (iii). (1 mark )
[ c = 3.0 x 108 ms-1]
Total 15 marks

( 2 marks)
( 4 marks)

14

15

15

16

16

17

17

18

CSEC

6920

January 2009 Paper 2


(a) On a grid, plot the graph of P against h.

SECTION A
Attempt ALL questions.

1. A student conducted an experiment to determine how the pressure


in a liquid varies with depth, see Figure 1

[8]

(b) (i) Write an equation linking the liquid pressure, density and
depth.
[2]

You MUST write your answers in this answer booklet.

(ii) Calculate the slope, S, of the graph.

[5]

(iii) Table 2 shows three liquids and their densities. Determine


which of these liquids was used in the experiment.
[4]
TABLE 2

Liquid

Petrol

800

Water

1,000

Density/ kg m-3

Mercury

13,600

(c) Using the graph you plotted on page 3, find the TOTAL pressure
exerted on the base of the container in Figure 1.

[2]

[Gravitational field strength, g = 10 Nkg-1]

The student obtained the following results as shown in Table 1.

[Atmospheric pressure = 1.03 105 Pa]


(d) The diagram in figure 2 shows a manometer being used to measure
the pressure of the gas in a container.

TABLE 1

0.70

Change in Pressure, P/Pa

Change in Depth, h/m

1210

0.10

1890

0.20

2900

0.30

4300

0.40

4750

0.50

18

19

Total 15 marks
3. (a) (i) Define the lower fixed point on the Celsius scale. [2]
(ii) A changing temperature can cause the physicsl properties of
a substance to vary.
Complete Table 4 by listing TWO other types of thermometers and the
physical property which varies in EACH of the three thermometers.


(i) State, giving a reason, if the pressure of the gas is LESS than or
GREATER than the atmospheric pressure.
[2]
(ii) Calculate the EXCESS pressure in mm Hg.

[2]
Total 25 marks

Thermometer

Physical Property

1 Thermocouple


(b) The temperature of the top of a popular waterfall is 22.00C. The
temperature at its base, 210 m lower, is 22.50C.

2. (a) Complete Table 3 below to show the link between an


electromagnetic wave, its source and its use or effect.

[g = 10 m s-2]

Electromagnetic wave
Visible light
Microwaves

Source



Use or Effect
Photosynthesis

Cancer treatment
Sunburn

(i) Calculate the specific heat capacity of the water. [3]


(ii) If the waterfall was twice as tall, determine by how much the
water temperature would change from its initial value.
[2]
(c) A gas of volume 60.0 cm3 and initial pressure 2.0 105 Pa has its
pressure reduced to 1.6 105 Pa. If the temperature remains constant,
what is the final volume of this gas?


[3]


(b) Microwaves travel at a speed of 3.0 108 m s-1 in a vacuum and
have a frequency of 1.5 1010 Hz. Calculate the wavelength of
microwaves, in centimetres.




[4]
(c) As the microwaves enter water at an angle of incidence of 450, their
speed is reduced to 2.0 108 m s-1. Calculate the wavelength of the
microwaves in water.



[4]

Total 15 marks

SECTION B
Attempt ALL Questions.
You MUST write your answers in the space provided after
each question.
19

20

(b) With reference to the symbol

4. (a) With the aid of a labeled diagram, describe an experiment to


show that the angle of incidence is equal to the angle of reflection.

(c) (i) Describe the relationship between the shell model of the
atom and the periodic table.

(b) Figure 3 below shows a ray of light entering a rectangular glass


block ABCD of refractive index 1.5. The angle of incidence is 600.

(ii) Explain the meaning of the term isotope.

[1]

[2]

(e) Radioactive Lead decays in three stages to form stable Lead




(i)
of refraction a the AD boundary.

Stage 1

Stage 2

Stage 3

Calculate the angle



(ii) What is the value of the angle of emergence at the BC
boundary?

(i) Copy the three stages above and complete EACH equation


(iii) If the critical angle for glass is 420, describe what would
happen to the ray if its angle of incidence on boundary BC was 42.50.

(ii) Explain why lead is used when handling radioisotopes.

(iv) State the reason for your conclusion at (iii) above.

[9]

[4]
Total 15 marks

(i)

Atomic number

6.
Figure 4 shows an ideal transformer being used to step down a
220V a.c.
supply, in
order
to operate a
low
voltage lamp.

(ii)

Mass number

(iii)

Neutron number

Total 15 marks
5. (a) Explain the meaning of EACH of the following terms:

(d) One gram of a living plant containing Carbon-14, decays at about


16 disintegrations per minute. It is found that one gram of a dead plant
decays at 1 disintegration per minute. The half life of Carbon-14 is
about 5,500 years. Calculate the probable age of the plant.
[5]

[3]

20

21







(a) Explain the principles involved in the workings of the transformer,
and how it is able to step down the input voltage.

[5]
(b) A lamp is connected to the secondary coil of the transformer by long
leads which have a resistance of 2.5 . The power input to the primary
coil is 44 W.
Calculate
(i)

the voltage across the secondary coil of the transformer

(ii)

the current in the secondary coil of the transformer

(iii)

the electrical power dissipated in the lamp.

[9]

(c) High voltages are used to transmit electrical energy over long
distances through long distance cables. State an advantage of using high
voltages.






[1]
Total 15 marks

21

22

Physics June 2009 Paper 2



1. In determining the half-life of an alpha-emitting radioactive source, a
student recorded the following data in Table 1.

(a) Plot on page 3, a graph of count rate (R) versus time (t).
(b) What is meant by the 'half-life' of a radioactive sample?

(1 mark )
(c) Describe the procedure a student would use to obtain the results in
Table 1. State ONE precaution.
(3 marks)


(d) (i)From the graph, make TWO calculations of the half-life of the
sample.
( 6 marks)
(ii)From your results in (d) (i), calculate the mean (average) half-life.
( 2 marks)
(e) Estimate the count rate of the sample after
(i)
425 s
(ii)
4 half-life periods.
(6 marks) Total 25 marks

2. (a) (i)Give the difference between a 'vector quantity' and a 'scalar


quantity' .
(1 mark)
(ii)Complete Table 2 by writing EACH of the physical quantities
from the list below in the appropriate column:
Mass, Momentum, Displacement, Heat Capacity, Temperature,
Half-Life, Upthrust.
(6 marks)

( 7 marks)


(b)A taxi reaches a traffic police officer while travelling at 30 m s-1
on the highway. The officer immediately signals the taxi which
decelerates uniformly and comes to a complete stop in 5 s.
(Assume zero reaction time.)
(i)In the space below, draw a velocity-time graph to represent
the motion of the taxi from the moment the officer signals, to
when the taxi comes to a stop.
(2 marks)
(ii) Determine the distance travelled by the taxi from the time the
police signals until the taxi stops
( 3 marks)
-1
(iii) If the speed limit is 80 km h , determine whether or not the
driver should receive a speeding ticket.
(3 marks)
Total 15 marks

22

23
3.
(a)When travelling along a hot road on a hot day a traveller
may see a distant object with its image directly below it as shown in
Figure 1. The image is called a mirage. Three light rays on the
diagram are labelled A, B and C.

(i)In terms of the distances a and d, write an expression for

Figure 1
(i)Why does Ray A travel in a straight line from the object to the
observer?
( 1 mark )
(ii)What is the name given to the bending of Ray B as it approaches
the hot surface?
( 1 mark )
(iii)With reference to the normal, the temperature and the density of
the air, explain the direction of the bending of Ray B at Point x.
( 2 marks)
(iv)The mirage is a virtual image. With reference to Rays B and C,
explain how the mirage is formed and why it is described as a virtual
image.
( 2 marks)

a) the distance travelled by the sound pulse after reflection


from the front of the skull to the sound detector
( 1 mark )
b) the distance travelled by the sound pulse after reflection from
the back of the skull to the sound detector.
( 1 mark )
(ii)The oscilliscope screen shows that the time between the first
and the second echo is 0.11 ms. The speed of the sound pulses is
1200 m
Determine
a) the difference in distance travelled between the first and the
second echo
(2 marks)
b) the diameter, d, of the baby's skull.
(1 mark )
(c)Figure 3 shows how a human eye focuses to see an object
0. C represents the optical centre of the lens in the eye.

(b)Ultrasounds may be used in determining the growth of a baby in


its mother's womb. Sound pulses are directed towards the baby's
skull and reflected pulses (echoes) from the front and back of the
baby's skull are detected and displayed on an oscilliscope screen, as
shown in Figure 2.

Figure 3
The object 0 is 100 m from the centre of the eye lens and
the magnification of the eye lens is 0.005.
(i)

23

24
Determine the distance of the retina from the centre of the eye
lens.
( 3 marks)
(ii) Write a formula for calculating the magnification of the eye
lens using the object height and the image height
(1 mark)
Total 15 marks

4.
(a)(i)State the law of the conservation of
energy.
(ii)
What is the difference between 'potential energy' and
'kinetic energy'?
(iii) Does the fact that heat is lost when work is done against
friction violate the law of conservation of energy? Justify your
answer.
( 6 marks)
(b)The three-week-long cycling race, the Tour de France,
is said to be one of the most grueling sporting events in the
world.
(i)
If a cyclist of mass 70 kg uses a bicycle of mass 7 kg,
how much work must the cyclist do against gravity in order to
ascend to 2100 m from sea level (0 m)?
(ii)
One particular descent goes from 2100 m to 1600 m.
Assuming the work done
against friction is 90% of the potential energy change of the
cyclist and the cycle, what INCREASE in speed in km/h can a
rider attain by the end of the descent?
(iii)
What is the average rate of energy conversion of the
cyclist and cycle if the descent in part (ii) takes 1 minute at
constant speed?
(Acceleration due to gravity, g = 10 m s-2)
Total 15 marks

5.

Magnets are characterized by magnetic field lines.


(a) Sketch the magnetic field associated with (i) a single bar
magnet and (ii) between two strong bar magnets with their north
poles facing, and in line with, each other.
( 4 marks)
(b) An emergency flashlight is an essential item during
hurricanes and blackouts. One model has a crank handle
connected to an alternating current generator, rechargeable
batteries and light emitting diodes (LEDs) as shown in Figure 4.
When the handle is turned, a permanent magnet also turns in the
presence of a stationary coil. This produces a current which
charges the batteries.

Figure 4
(i) How is the electromotive force (e.m.f.) generated in the
stationary coil?
(ii) What ADDITIONAL component is needed to be able to
charge the batteries?
(iii) At a reasonable cranking rate, the unit generates 6.2 V.
( 9the
marks)
Calculate
current in the circuit if the resistance is 310 .
( 5 marks)
(c)A similar device includes a transformer so that an MP3 player can
also be charged. The primary coil has 300 turns.
24

25
(i) How many turns are needed in the secondary winding if the
voltage is stepped up from 6.2 V to 15.5 V?
(ii) Given that the current in the primary winding is 10 mA, what
power is transmitted to the secondary windings if the transformer is
77% efficient?
( 6 marks)
Total 15 marks

6. (a)You are given a container of negligible heat capacity and a


thermometer.
Describe how the specific latent heat of fusion of ice can be
determined using the method of mixtures.
(6 marks)
(b)A physics student was provided with 25 g of ice to convert to
steam. How much heat is needed to change this 25 g of ice at 0
C to steam at 100 C?
( 9 marks)
(Specific heat capacity of ice = 2 x 103 Jkg-1K-1)
(Specific heat capacity of water = 4.2 x 103 Jkg-1K-1)
(Specific latent heat of vaporisation of water = 2.3 x 106 Jkg-1)
(Specific latent heat of fusion of ice = 3.4 x 105 Jkg-1)
Total 15 marks

25

26
(h) (i)

PHYSICS JUNE 2008 - PAPER 2


General Profi,ciency - 2 Yz hours

(2)

(ii) Describe the constructional features of commercial transformers which


minimize the losses stated in Part (h) (i).
(2)
(Total 25 marks)

SECTION A - Answer ALL questions


1.

State TWO major sources of energy loss in transformers.

In the experimental setup shown in Figure I, a variable a.c. voltage


source, V J , drives the primary of a step down transfonner of turns
ratio 5000 : 600. A resistor, R = 8 n, is connected across the
secondary winding.

2. (a)

Table 2 is an incomplete table showing the quantity to be measured, the


instrument used to measure the quantity and its S.l. unit.
Complete Table 2 by writing in the missing information.
Table 2
Quantity to be measured
Instrument
S.I. Unit
Volume of a liquid
Clinical thermometer
Newton
Time
Stop clock
(7)
(b) A swimming pool 30 metres long is filled with water to a depth of I
metre at the shallow end and 5 metres at the deep end (see Figure 2 not
drawn to scale).

r
VI

l
Figure 1
Table I shows the values obtained for V2 when VI is varied from 0 V
to 90 V.
Table 1

1m

T
(a) Use the readings from Table I to plot a graph of V2 vs VJ.
(b) Find the slope, S, of your graph.

30m

~~A

(6)

B
/~-

/""- ~

'" '/ / 'C"A />:- ,,",--_


~://y~-......
'\~

:"".::.",

(4)
(c) What does the slope of your graph represent?

Figure 2
(1)

(d) Use your graph to detennine the value of V2 when VJ = 50 V.

(1)
(e) Calculate the current h in the secondary windings when V 1 = 50 V
(3)
(f) Use the slope of your graph and the equation
I J = 812
to determine the current I} in the primary winding.
/
(2)
(g) (i) Calculate the power delivered to the(resistor, R, by the secondary
windings.
.
(3)
(ii) Calculate, also, the power input to the primary windings by the
(1)
source.
\
\

ABCD the vertical cross-section through the pool has the shape of a
trapezium with ar~ given by:
Area ABCD = Y:z '(AB + CD) x AD
Calculate the area ABCD
(2)
(ii) Determine the volume of water in the pool, given that it is 20 metres
wide.
_ _ _ _ _ _ _ _ _ _ _ _ _ _ _ _ (3)
(iii) Calculate the mass of water in the pool.
[density of water = 1000kg m .3]
(3)
(Total 15 marks)
(i)

3.

(a)

The THREE MAIN particles in an atom are:

26

/~

27

A.

B. '

-'--

C
(b)

5.

The corresponding location ofEACH ofthese particles is:


A.

B.
C

(c)

The particle with NO charge is called

(d)

How many half-lives would it take for a sample of Carbon-14 to


be reduced to 1/32 of its original mass?
(2)
Given that Carbon - 14 has a half-life of 5700 years, detennine
how long it would take for this reduction to occur.
(2)
Carbon dating involves t~~ of Ca.rl)ori-=-i4to det:ennine the
age of ancient objects. Explain how this is accomplished.

(a) Describe an experiment to verify SneIl's law. State the apparatus


used. your method, and the results you would take to arrive at your
conclusion.
(6)
(b) Figure 4 shows a ray of light, XV, incident on a right-angled prism,
PQR, of refractive index 1.5. The point of incidence on PR is such
that the refracted ray inside the prism is incident
on PQ.
p

(1)

(e)

(t)

(4)

(a) (i) Define the 'moment of a force'.


(ii) Draw a diagram to illustrate the action of a force multiplier lever.
(iii) Explain the operation of a force multiplier lever.
(6)
(b) Figure 3 shows a wheelbarrow and stones with a total mass of 43 kg.
The wheelbarrow is in eq~i1ibrium with two ofthe three forces acting
on it shown in Figure 3. I 6." I
~~pned

k
lEi

wheelbarrow

vm

6.

I~

)10'.

~n I

(F)

i..

!.Sill . . . .

~i

. Figure 3
.
(i) Identify the nature and point of action ofthe third force acting on the
wheelbarrow. Write down an equation showing the relationship
between the THREE forces.
(ii) Calculate
(a) the weight, W (see Figure 3)
(b) the value of the applied force, F (see Fi~ure 3).
[acceleration due to gravity. g::: IOms - ]
(9)
(Total 15 marks)
I

'Q

Figure 4
(i) Calculate the angle of refraction at the boundary, PR.
(ii) Given that the critical angle for the glass-air boundary, PQ. is
41.8, deduce the whether or not there would be total internal
reflection at this boundary.
(9)
(Total 15 marks)

(Total 15 marks)
SECTION B - Answer ALL questions
4.

C'"h.l'l

(a) The method of mixtures is used to detennine the specific heat capacity
of liquids and solids by experiment. Describe the procedure, the
measurements to be taken and the use of these measurements to
obtain the result in such an experiment.
(6)
(b) It is recommended that in order to maintain good health a person
should drink at least 4 x 10 - 3 m3 [4kg] of water per day. Assume
that this entire volume of water, initially at ISC, is eventually
excreted as urine at 3TC.
Find the amount of heat removed each day by the quantity of urine.
[Assume the specinc heat capacity of urine specific heat capacity of
water =4200 J kg- I'K- I , density of urine = density of water =
lOOOkg m 3]
(6)
(c) What mass of perspiration would remove the same quantity of heat as
the urine in Part (b), when completely evaporated from the skin?
Assume that evaporation is equivalent to a change of phase from liquid
to vapour without an increase in temperature.
.
[Specific latent heat of vapourisation of water = 2.3 x 106 J kg - I] (3)
(Total 15 marks)
END OF EXAM

27
_____

'..::'l.lH'.

Il!i,~'t'

.~~. <

. .--...tJ

~.~

......

28
PHYSICS JANUARY 2008

1. . A student investigating a relationship between extension and force for a spring of original length
60.0 em obtained the results seen in Table 1 below.

Paper 02 - General Proficiency

TABLEt

IJ.hours

Mass, mig

Force,FIN

Final length of

Extension,

spnng,lIcm

elcm

'aph paper for Question 1.

20.0

605

40.0

61.0

60.0

61.5

80.0

62.0

100.0

62.4

120.0

63.0

(a)

Complete Table 1 by calculating the force. F, and the extension, e, of the spring.
[acceleration due to gravity. g:::: 10 N kg']
( 4 marks)

(b)

Plot a graph of Extension against Force on the graph paper on page 2.

(c)

From your graph, calculate the slope, S.

(10 marks)

(4marks)
(d)

The spring constant is related to the slope of the graph as follows.

.
1
Spong constant :::: S.
Calculate the spring constant.

(2 marks)

(e)

Draw a labelled diagram showing the apparatus as used in this investigation.

( Smarks)
With the aid of dotted lines. use your graph to
(i)

detemrine the force which would cause an extension of 1.8 em

(2 marks)
(li)

estimate the extension of the spring if a force of 1.3 N is applied.

(Zmarks)

il...

28

7.

If the student continues to take readings, a point may be reached where proportionality no
lOnger exists.
'

(g)

Figure 1 shows light passing through a reetang1l1ar glass block. Important features ofthis arrangement
29
are labelled A, B, C, D, E and x.

3.

jName this point


G....

blodr.

Total 30 Jnllrks ( 1 mark)


(a)

2.

(i)

State Boyle's Law.

(2 marks)
(li)

""

Using the axes below, sketchthelineargraph you would expectto obtain from aBoyle's
Law experiment.

"-"1

t
(a)

Identify the labelled parts ofFigure 1.


A

x
(6 marks)

( 4 marks)

r (b)

A police car tyre had a fixed mass ofair at a temperature of24"C. The air pressure was 210 KPa
above atmospheric pressure. After a high speed chase, the air pressure in the tyre was 260 KPa
above atmospheric pressure..
(1)

(b)

The refractive index ofthis glass block is 1.5.


(1)

1\

1\

Determine the value of angle b (b) When a == 60".

(l mark)

Apply the metic theory of matter to explain the increased pressure in that tyre.
(li)

State a reason for your answer in (b) (i).

( 4 marks)
(ii)

Detennine the new temperature ofthe air in the tyre, assuming no change in the volume.
(Atmospheric pressure

::=

1\

Calculate the value of angle d, (d).

100 KPa).

( Smacks)

Total IS marks

Oroark)
(ill)

( 4 marks)

29

30
A

(ii)

Determine the value which angle d Cd) must exceed if E is to be totally internally
reflected.

<iv)

Ii

The component labened A in Figure 2 makes 10revolutions in one second. On the grid
provided. sketch a graph to represent the variation of output curtent with lim!! durin.!! j
the fJr.St two revolutions. Indicate the SCALB used. on BACH axis.

( 3 marks)
Total 15 marks

4.

Figure 2 represents an alternating current generator.


B

(iii)

The speed of rotation of component A is tripled with no other change involved.


Determine the effect on
the period of the output current

a)

Figure 2
(a)

(i)

(2 marks)

Write the name of the components labelled A, B, C and x.

the peak value of the output current.

b)

(4 marks)

Explain why a voltage is obtained across the terminals x and y.

s.

Draw circuit diagrams showing

(a)

(1 mark)
Total IS marks

two resistors connected in series

(i)

(1 mark)

(Ii)

( 2 marks)
(b)

two resistors connected-in parallel.


( 1 mark)

The terminals x and yare connected to a 5 0. resistor. The peak output voltage across x - y is
l2V.
(i)

(b)

State ONE characteristic ofresistors connected in

Calculate the peak value of the output -current.


(i)

series

(1 mark)

(3 marks)

(ii)

parallel.

30

JII"

Figure 3 shows a circuit diagram in which devices AI' B I, A z and Bzare connected.

(c)

(i)

\8,

J\\~

Je

~p

Paper 03 - General Proficiency

31

1 hour
1.

(a)

(b)

Draw a clear labeIled diagram of a U-tube manometer and describe how it may be used for
measuring the excess pressure of a gas above atmospheric pressure.
( 8 marks)
Some farmers and gardeners use a pressurized sprayer to apply pesticide to their crops or plants.

oQ
Nom.N

Figure 3

<

Identify the devices labeIled by the

B's

If \\

-~] - II

Air.pat<

Llq1Ild plidik

tDIaI _ _
ot

(2 marks)

101_

The devices labelled AI and A z are identical with the rating 60 W,140 11. A resistance

meter is connected across the terminals P and Q. Complete Table 2 shOWing any

necessary working.

F1....... l

Figure I shows such a sprayer, consisting of a pressure vessel of volume 10 litres,


containing 8litres ~ I2l<sticiQ,e... Valve B is first closed. The space abov~d
is filled with ~ at a p~~a, pumped in with a hand pump through open
valve A until the desired pressure is reached. Valve A is then closed.

TABLE 2

Devices

Resistance Meter Reading / 11


7777)777777>7777777

B1

Bz

open

open

open

closed

closed

closed

(i)

The sprayer is operated by opening valve B thereby causing the liquid pesticide to
squirt out ofnozzleN. Explain why this occurs by considering the forces acting on
the liquid columnX - Y. Hence predict the condition under which liquid will NOT
squirt out of the nozzle when valve B is <?pened.

(ii)

The sprayer is used for a certain period of time during which the liquid level falls
and the pressure above the liquid is reduced.
Calculate the volume of pesticide which would still remain in the tank when air
pressure above the liquid is 150 KPa if all temperatures remain constant
[Assume all pressure differences due to hydrostatic pressure of liquid to be
negligible; I.e. pressure at upper surface of liquid
pressure at bottom of tank.]

(Smarks)
(d)

[Atmospheric'pressure = 100 KPa]

The resistance meter is removed and a battery is connected across P and Q. What should be the
voltage of the battery in order to deliver the rated power to both At and A z?

SHOW ALL YOUR WORKING.

Total 20 marks

2.

'\

( 4 marks)

totall' mark.s

(12 marks)'

(a)

Name TWO methods of heat transfer and describe THREE characteristics of EACH
method identified.
( 8 marks)

(b)

Figure 2 shows the curved reflecting surface of a solarcooker which focusses


sunlight onto
2
a pot containing food. The area of the reflecting surface is 0.75 m and the solar power
incident on it is 800 Wm-z. Only 76% of the energy incident on the curved surface is
reflected towards the pot.

END OF TEST

31

4.

==! I I

and fluorescent lamps are three such appliances. In EAeH case, name the MAJOR
form or enerzy into Which elecUical energyl'l. convertea.

_lot

:::;;/-4,

WI...
tvltJUtdJng

Electrical energy is used in many appliances in the home. The fan, television
setII
32

(a)

S.blrndht1eft

po'

(ii)

C_.Jpol

Describe FOUR ways by which electrical energy can be conserved in the home.
(8 marks)

(b)

Mr Williams approached Melissa, a physics student, to confirm whether his monthly electrical
bilI of $320.00 was reasonable. The appliances in his house and usage are as follows:

...

Number

(i)

the energy arriving at the curved surface each second

100 W bulbs

210 hours

(ii)

the total energy reflected towards the pot in one hour

125 W Colour Television

120 hours

(iii)

the energy absorbed by the pot and its contents if this energy is just enough to raise

their temperature by 70 C

1100 W Microwave oven

300 minutes

[Heat capacity of pot and food = 8500 J K-

(a)

Total 20 marks

(12:marks)

Calculate the total number of units of energy consumed by Mr Williams' appliances.

(ii)

Calculate the total electricity bill if $1.60 is the cost per unit.

(iii)

What should Melissa tell Mr Williams about the correctness of his bill?

(iv)

You are provided with 1 A,5 A, and 10 A fuses. Select an APPROPRIATE fuse fo~
the microwave oven given a 120 V electricity supply.

You are making a presentation to your class on the image formed by a converging lens
acting as a magnifying glass.
(i)

(12 marks)

Draw a labelled diagram to show the following clearly:


a)

The principal axis

b)

The principal focus

c)

Total 20 marks

5.

.The focal length

d)

Identify THREE differences between a real image and a virtual image.

(5 marks)
(3marks)

=!

(c)

During a nuclear fission reaction, the uranium isotope,

TABLEt
Particles

(ii)

the magnification of the lens used

(iii)

the focal length of the lens

(iv)

the position of the image formed if the object distance was doubled.

(12~ks)

MassIKg

Barium

x 10-21
390.989 x 10-27
238.893 x 10-27

Krypton

149.241 x 10-27

Uranium nucleus

the height of the image of the stamp

U is struck by a neuli'on and

Table I gives the masses of the particles involved in the nuclear reaction.

Calculate

2~~

arks)

splits into barium (Ba). krypton (Kr) and twO neutrons.

Neutron

(i)

arks)

Explain what is meant by the term 'radioactive decay'.


State the name and nature of THREE types of radioactive emissions.

A stamp collector views a stamp of height 2.0 cm with a hand lens placed 1.5 cm from the

stamp. The distance of the image from the lens is 5.0 cm. The object distance. u, image
distance, v. and focal length. f, of a lens are related by

1. + 1
u
v

(a)

The image formed

Is the image formed in this situation real or virtual?

(c)

(i)

the efficiency of the solar. cooker.

(iv)

... ,

Estimated Usage

Electrical Device

F1gu.... z

Calculate

3.

Ol,....s.nlI..tllIJ

1.675

32

33
[The speed oflight. c::: 3.0 x 10& m :.;-1].
(I)
OJ)

Represent this reaction in the standard fonn of a nuclear equation. given that the
atomIc mass of barium is 144 and the atomic number of krypton Is 36.
Calculate
a)

the total mass of the starting products: uranium nucleus and the neutron

b)

the total mass of the final reaction products: the barium and krypton nuclei
and the two neutrons.

(iii)

Explain why the answers to (c) (ii) a) and (c) (ii) b) are different.

(Iv)

Calculate the energy released in this reaction and deduce the fonn in which this
energy will appear.
(12 marks)
Total 20 marks

END OF TEST

33
~

...

"

CARIBBEAN EXAMINATIONS COUNCIL


JUNE 2007 PHYSICS
PAPER 2
Time: 1 Vi Hours
All questions must be attempted. ONLY non-programmable
calculators allowed. ALL working must be shown.

<"""" .. """""""""""""""""""" "''''''' K"""" "''' "''' ~"o " .. " '" <" "" .,-",,",,""""""" >l"" X"" x","""""""" X" X"""""""

(g)

v".<"v .. "'''''''''',. .. "'',, .... ~''''

(h)

DATA ANALYSIS

l.. .!~~p~r:ll!lIrej~<:: ..
Volume / em3
(a)

.,~~_

250
87.0

200
78.9

With the aid of a diagram, describe a method that the student could have
used to obtain her results.

,(6 marks)
(b)

(c)

Plot a graph of volume, V, (y-axis) against temperature, T, (x-axis). The


axes have been drawn for you. The scale for the x-axis should include the
range -300C to +300C, with 50C per division. Label the graph line of V
against T "Line 1".
(9 marks)
From the graph, determine VI, the volume of the gas at -120C
' d

(d)

"

,.(2 marks)

At what temperature is the volume 2V 1?

The pressureas is doubled and the experiment repellted. Draw, on


the same set of axes, the line that would be obtained if the experiment is
repeated at this higher pressure. Label this line, "Line 2".
.(2 marks)

~ ~

"" ""

~""'''''''''''''

"X""".ox

v.

.<,. __ " .. "-""

,-" .

y" """" """ "" ~ >. ~ > ~"" l<"

,
(f)

"" "" ",," """"""""" ""

A B C
..... _-- _._.- 27m ._.-.-.-.- .......-.--.----.-.-.--.-- 54m----- ..
Figure 1

Figure 1 above shows a car at A moving towards C, with a constant velocity of


30 m SI. As the car passes A, the driver sees an obstruction in the road at C. Due to
the reaction time of the driver, the car travels a distance of 27m, at the same velocity,
before the brakes are applied at B. The car now decelerates unifolIDly and comes to a
halt at C, which is 54 m from B.
(i)

Calculate the reaction time of the driver.

(ii)

The car takes 3.6 s to travel from B to C. In the space below, draw a
velocity time graph to represent the motion of the car between A and C.
(3 marks)
What is the average speed of the car between Band C?

r, ".",,', "'" ",," """ ~'" .. """

,.,,,

(4 marks)

From your graph, read off the temperature, To, at which the volume would
be zero. Explain the significance of To.

Explain what is meant by the terms 'velocity' ap.d 'acceleration'.

(b)

(iii)

DetelIDine the slope, S, of the graphs. Include its units.

(2 marks)

........................................................................................... .(3 marks)

..................................... "., ,(2 marks)

(e)

,_" '_""v""""""

TOTAL 30 MARKS
2 (a)

150
70.5

49.1 '

""""~,,~ _d.~.

"

A student investigates the relationship between the volume and temperature of a


'fixed mass of gas at a constant pressure of I atmosphere.
The results she obtained are shown in the table below

,'""" ",,",,"''''

.. ".,."
(3 marks)
The equation of how the graph line is V = mT + C. Write down the values
ofm and C.
'

1. No more than Yz hour should be spent on this question.

34

(2 marks)

"
(iv)

,. (3 marks)

If the mass ofthe car is 750 kg, fmd the braking force that slows the car
between Band C.
.,

.,

",..

"

, .,(5 marks)

TOTAL 16 MARKS

34

tt

3 (a).

35

TbeoryA

Figure 2 below shows a shiny kettle which is placed on a gas stove to heat
the water inside it.

Cover

TbeoryB

Water _.

,,
(2 marks)
(b)

Figure 2
(i)

By what process is heat transferred to the water through the kettle?


(1 mark)

(ii)

Explain why kettles are usually shiny on the outside.


,,, .... .,

., . .,

.,

\1

Table 1 below lists some of the properties of light.


Indicate, by ticketing the relevant boxes, the properties which these theories
can explain. Place an X in the box if the theory CANNOT satisfactorily
explain the property.

mark)
PROPERTY

(iii)

How does the cover prevent heat losses?

THEORY A

THEORYB

Travels in a straight line


marks)

Reflection
Conveys energy

Only about one-third ofthe thermal energy generated by burning the gas
goes into heating the water, whilst the remainder is wasted.
State where the wasted energy goes and give the process by which this
wasted heat is transferred.
marks)
(c)

If the mass of water in the kettle is l.5kg, calculate the thermal energy
required to raise its temperature from 30C to 100C.
(3 marks)

(d)

How much more energy must the water absorb for 0.2kg of it to boil off?
(3 marks)

(e)

Explain why it is better not to fill the kettle when making one or two cups
of tea.
........................... w

.......... "

,{2 marks)

....

Diffraction
Table 1
(4 marks)
(c)

An optical fibre is a flexible glass rod of extremely small diameter which is


used to carry light signals over distance of several kilometres.
Figure 3 below shows the structure of a typical optical fibre, with a glass
core ofrefractive index, 1.50, surrounded by a glass cladding of different
refractive index, n 2 Once the light enters the core, it is totally internally
reflected and remains inside the fibre until it reaches the other end of the
fibre. The ray of light, AB, in air is refracted into the core along the
path Be.

Cladding

nz

Air'

Core
n:::; 1.50

(Specific heat capacity of water = 4.2 x 10 J kg K'l


6
l
Specific latent heat of vaporization ofwater = 2.3 x 10 J kg )
TOTAL 14 MARKS
4.

In the seventeenth century, two conflicting theories concerning the nature of


light were put forward.
(i)
State what these theories considered light to be.

Cladding

Figure 3

35

(i)

36

What is meant by 'total internal reflection' at point C?


marks)

Paper
(ii)

Is the refractive index ofthe cladding greater or smaller than that of the
core?
..................... ,

(iii)

"

, .. u.u"u

uu,

'

n.\..

mark)

Draw on Figure 3, the ray that shows the path of the light continuing on
from C.
................... "

u un'''''''',,,

uu

" ""u""

FigureS
(iii)

Draw, on figure 5 above, the charge distribution on the paper


when it is in contact with the polythene strip.
(iv)

(1 mark)

(1 mark)

State whether the paper gains or loses charge and the type of
charge transferred, when it touches the, strip.
(2 marks)

(3 marks)

(iv)

Calculate the angle of incidence of the light entering the fibre.

(v)

Given that the speed of light in air is3 x l08 ms'\ , determine the speed of
light in the core ofthe fibre.

(b)

When lightning strikes the earth, there is a massive flow of electrons from a
thundercloud to the earth during a short time interval.

marks)

Thundercloud

TOTAL 16 MARKS

5.

A negatively charged polythene strip is used to attract a small piece of


insulating paper as shown in Figure 4 below.

~' 7 ~7~7777~77
~
~
77777777

Earth

Figure 6

c;5paper

(i)

Draw, on Figure 6 above, electric field lines between the cloud


(2 marks)
and the earth.

(ii)

Describe a precaution which may be taken to protect a building


(2 marks)
against lightning.

Figure 4
(c)
(i)

Show, on the diagram above, the charge distribution on the paper.


(2 marks)

(ii)

Explain why there is a net force of attraction.

During a certain lightning strike, a current of 5 x 10 A flows for a time


period of 0.1 5 ms.
Calculate the quantity of charge that flows from cloud to earth during the
strike.
(3 marks)

(2 marks)

TOTAL 14 MARKS

END OF TEST

36

5'

37

Graph for Question 1

37

PHYSICS - JUNE 2007


GENERAL PROFICIENY- PAPER 03 -lBR
'"

l.(a) State Newton's first and second laws of motion. Use the laws to
explain how'a satellite remains in orbit around the earth. (8 marks)

38

(b) (i) Pitch and loudness are terms used to describe sound. How are the
frequency, wavelength and amplitude ofa sound wave affected by
an increase in the
(a) pitch of the sound?
(b) loudness ofthe sound?

~ Spaee Shuttle

Tllrust of rocket
motors '" 540, 000 N

k""

~-

mass '" 1.1 X 10; kg


.....

,-

(ii) The note, middle C, played fITst on a piano and then on a violin
sounds different on the two instruments. The notes are said to have
(5 marks)
different quality. Explain this observation.

'"

v-

A
\
\

Height of orbit", 450 km

YOrbital velocity =I) kmls


I

" '"

""'-.-...c.""'"

----

(b) (i) The mass ofa space shuttle is 110 tones (1.1 x lOSkg). It orbits the
earth at a height of 450 km, with a speed of9 km s -1. Calculate the
kinetic energy ofthe shuttle in this orbit
(ii) The shuttle possesses gravitational potential energy. Assuming that
the gravitational field strength (g) is the same at all heights up to 450
km, calculate the total energy of the shuttle as it orbits the earth at
this height.
(7 marks)
When the space shuttle is to return to Earth, it fires its rockets in order
to slow down. The total thrust of the rocket motors is 540 000 N and
the rockets are fired for 360 s.
Calculate the
(i) deceleration ofthe space craft while the rockets are firing
(ii) reduction on speed during the 360 s.

(g= 10Nkg-1)

-- ---;-----

Figure 1

(c)

(c) The diagram below shows two loudspeakers, L 1 and L 2 connected to the
same source and vibrating in phase with each other.

(5 marks)
Total 20 marks

2. (a) Sound is a longitudinal wave that is produced by vibrating sources.


Sound waves emitted from two sources are found to be vibrating in
phase. Explain what is meant by the terms underlined.
(3 marks)

(i) The loudspeakers produce notes of frequency 132 Hz. If the speed of
sound is 330 m s"l, what is the wavelength of the notes?

(ii) Consider, at any instant, a sound wave starting at Lx and proceeding


in the direction of A. Sketch a graph of displacement (of the air
molecules) against position for the fITst 7.5 m of this wave.
(iii) A student walks along the line XY. At both A and B the sound is
loud whereas at C the sound is quite faint. Explain these
observations.
With the frequency of the notes still 132 Hz, the loudspeakers are
moved closer together. What will be the effect on the observations at
A, BandC?
(iv) The frequency of sound is now changed to 264 Hz with the
loudspeakers returned to the original positions. Will wavelength now
be larger or smaller than before? What will be the effect on the
observations at A, B and C?
(12 marks)
Total 20 marks

)
38

39
3. (a) Answer the following questions using the kinetic theory of matter.
(i) Explain why thennal energy has to be supplied to a liquid at its
boiling point to be changed into gas, without any chang~ in
temperature, and why the volume of the gas produced is much
greater than the original volume of the liquid. State the energy
change that occurs during the phase change from liquid to gas.
(ii) A gas exerts a pressure on the walls of its container. What causes
this pressure? If the temperature of a gas in a container is reduced,
the pressure in the container falls. Why does this occur? (8 marks)
(b) At standard temperature and pressure, that is, 0 C and 1.0 I x 105 Pa
pressure, the density of oxygen is 1.43 kg m3
Oxy~en is stored in a container of volume 3.87 m3, at a pressure of2.02
x 10 Pa and at a temperature of 25C.
(i) What volume would the same mass of oxygen occupy at 'standard
temperature and pressure'? Calculate the mass of oxygen in the
container.

(ii) The container is now heated to 45C. What will be the new

pressure? Assume the container does not expand.

(iii) Sketch a graph of pressure against KELVIN temperature for the gas.
Indicate on the graph the original temperature and pressure.
(12 marks)
Total 20 marks
4. (a) Describe the procedure used in the Geiger-Marsden experiment.
State the results of the experiment and outline what these results
suggested about the structure of atoms.
(8 marks)
(b) The common isotope of sodium, Na, is stable, while the sodium
isotope, with a mass number of24, sodium-24, is radioactive.
Sodium-24 decays into an isotope of magnesium (Mg) by the
emission of a ~-particle.
Write a nuclear equation to represent the decay of sodium-24. How many (i)
neutrons and (ii) protons are in the nucleus of magnesium which is formed
and how many electrons are there in a neutral atom of magnesium?
(6 marks)

(c) A radioactive source is found to be emitting, on average, 880


particles per second. If the source has a half life of 25
minutes, how much time will elapse before the average emission rate
becomes 110 per second?
Explain why the measured count rate in one second is unlikely to be exactly
110. What will be the effect on the count rate of (i) gently heating the
radioactive sample and (ii) heating the sample to a very high temperature?
(6 marks)
Total 20 marks
5. (a) Draw a diagram ofa simple moving-coil loudspeaker.
Explain how a changing voltage applied to the, loudspeaker results in
sound being emitted from it.
(8 marks)
(b) A loudspeaker is usually enclosed in a box to improve its efficiency
of converting electrical energy into sound. The loudspeaker is then
connected to the output of an amplifier. A certain loudspeaker has a
resistance of 16 n and operates at an efficiency of 15%. It is
connected to an amplifier which provides voltage of 12 V to the
loudspeaker.
Calculate:
(i) The current flowing in the loudspeaker
(ii) The electrical power being delivered by the amplifier
(iii) The sound energy per second being emitted.
(7 marks)
(c) A second identical loudspeaker Is now connected in a parallel with
the frrst loudspeaker across the output of the same amplifier which
still provides an output voltage of 12 V.
Determine:
(i) The combined resistance of the two loudspeakers
(ii) The total sound energy per second being emitted
(5 marks)
Total 20 marks

END OF TEST

39

40

PHYSICS JUNE 2006 - PAPER 2


General Proficiency - 1 Y:z hrs.
1. A student was asked to investigate the variation in temperature of2 kg of
water initially at 30C, when heat energy was supplied to it by an electric
heater rated at 1400 W. The following measurements were taken and
recorded in Table 1.
Table 1
Temperature of water, afc 30.0 40.0 51.0 62.5 72.0 80.0 89.0
0
60 120 180 240 300 360
Heatin2 time, tis
Temperature change,
0

A2

Figure 1
(i) Identify the type of connection employed in this circuit.

0= (A 6-30)fC

Ener2Ysupplied,(!=1400tJJ

(a) Complete the table above.


(3)
(b) Plot on the graph page opposite a graph of energy supplied against
(1
temperature change.
(c) Calculate the gradient ofthe graph.
(6)
(d) Assuming that the energy loss to the container and the surroundings
is negligible, what physical quantity does this gradient represent?
(1)
(e) DDe~ttee;nnnnj";;in;;;e1ithtiie;-;v~aililu~e~orlf~thiiiissqwmti[ty"iflllk~g:<orlf~w~art;terenwV;e;r;re;Wusesedd
and state the physical quantity this represents.
(2)
On graph page.
(t) Water boils at 100C. Calculate the heat energy which must be
supplied in order to completely convert the 2 kg of water to steam. (3)
(g) Distinguish between boiling and evaporation.

(ii) Give the meaning of the circuit symbols labelled:


C_-.--_ _::---.--_---::_-::------:_
(iii)Name the type of voltage produced at A.
_ _ _ _ _ _--::-_ _--,----:_-,----:-_ _
(c) (i) A fluorescent lamp is rated at 15W, 120V. Calculate the
current which the lamp draws from the main supply.
(ii) Calculate the resistance of this lamp while it is lit.
(iii) When the lamp is lit, 4 W of the energy supplied is lost as
heat. Calculate the efficiency of the lamp.
Total 15 marks

3.

(2)
(1)
(3)
(3)
(3)

Figure 2 shows successive stages in the generation of a transverse wave


on a stretched slinky spring.

(4)

StrtlA:1tod !IIlal<y

Total 30 marks
B

[Specific latent heat of vaporization ofwater = 2.3 x 106 Jkg'I]

~ F"xed ...d("all)

- - - - -'l!llQmOllmmu,-J

2. (a) Explain what is meant by 'electrical resistance'.


(2)

(b) The circuit shown in Figure 1 shows the interconnection between


electrical devices, B and C, and voltage source A.
J)'

Figure'2
Physics June 2006 P2 Page 1 of 3

40

-1

41
(i) Indicate on Figure 2F by appropriately labelling
(a) the position of a crest
(b) the position of a trough

4.

(a) Define 'the moment of a force'.

(2)

(3)'
(b) State the principle of moments.

(ii) Draw labelled arrows on Figure 2E to show


(a) the direction ofthe wave
(b) the direction of hand movement needed to produce this
wave.
(3)

(3)
(c) A cyclist hangs his bicycle with the rear wheel in contact with the
ground and the handle attached to a light string as shown in Figure
4. The entire assembly is in e9uilibrium.

(iii) Indicate on Figure 2A the direction of hand movement needed

to produce a longitudinal wave.


(1)

(b) In a classroom demonstration a student uses a slinky spring to generate


waves of frequency 5 Hz with 0.65 meters separation between
successive crests. Determine the speed ofthese waves.
(3)
(c) (i) Deduce whether the velocity of a sound wave win increase,
decrease or remain constant when the wave travels from a denser
to a less dense medium, given that the frequency remains constant
and the wavelength decreases.

(1)
(ii) Compare the waves produced on the slinky spring in part (b) with
a typical sound wave in terms of the motion ofthe particles.

Transverse wave on the slinky spring


_

Sound wave
(2)

(d) In Figure 3 below, a ship sends out pulses of high frequency sound
(ultra sound) which are reflected from the sea bed and received at the
ship every 1.2 x 10.2 seconds.

O.8m~

I
I

'~--1.7Sm~

Fignre4
TraIlsmltter

7 " <fl?

-..

Cross seclloo
oCsbip

Recdver

(i) Indicate by drawing arrows on Figure 4 the forces acting on the


bicycle.
(3)
(ii) Write TWO equations relating these forces.
_ _ _ _ _ _ _ _ _ _ _ _ _ _ _ _ _ (3)
(iii)The bicycle has a mass of 20kg. Use the result you obtained in
Part (c) (ii) to determine the tension in the string.

'I'ransmitte4 pulse

Received poise

Seabed

(3)

Fignre3
(3)
Calculate the depth ofthe sea bed below the transmitter.
(Speed of sound in sea water - 1 500 m S1)
Total 15 marks
Physics Iune 2006 P2 - Page 2 of 3

Total 15 marks
[Acceleration due to gravity, g- 10 ms2]

41

42
5. (a) Complete Table 2 below showing the International Insulation Colour
Code.
Table 2
Live Wire
Neutral Wire Earth Wire
Code
International
Insulation Colour
--

(b) Fuses are sometimes used to protect electrical equipment from excess
current. Explain how a fuse works.
(3)

(c) A household has the following appliances


Heating iron rated at 1200 W, 110V
Television set rated at 70W, 110 V
Electrical fan
52W, IIOV
(i) Calculate the total power consumed by these devices.
(1)
(ii) These devices are in use for 4 hours per day. Calculate the
energy consumed in kilowatt-hours in a 30-day month.
(2)
(iii) Calculate the bill for the month if each unit of electricity
costs 40 .
(2)
(iv) Assuming that these devices are connected in parallel across the
mains supply, and are switched ON, calculate the TOTAL current
drawn from the mains.
(3)
(v) Fuses are available with the following current ratings: 5A, lOA,
15A, 25A. Select a suitable fuse for protecting these devices.
(1)
Total 15 marks
END OF TEST

Physics June 2006 P2 - Page 3 00

42
<lI

43

PHYSICS JUNE 2006 - PAPER 3


General Proficiency 1hr.
1. (a) State, in words, Newton's three Laws of Motion.
(8)
(b) The Boeing 737-800 aircraft can accelerate from rest to a take off
speed of300 km hr.I in 25 seconds. Calculate in m s2the value ofthis
acceleration, assuming it to be constant.
(5)
(c) (i) The aircraft has a mass of 6xl 04 kg. Two identical jet engines
provide a thrusting force. Calculate the magnitude of the thrusting
force needed to give the aircraft the acceleration calculated in part
(b). Hence fmd the corresponding thrust for EACH engine.
(ii) The thrust calculated in (i) is a theoretical value. Suggest a reason
why this thrust if applied IN PRACTICE would NOT provide the
expected acceleration.
(iii) The mass ofthe aircraft was increased by loading additional
passengers and luggage. How could this affect the time taken to
reach take-offspeed?
(7)
Total 20 marks

2. (a) With the aid of a labelled diagram, explain how you would observe
Brownian motion.
(8)
(b) Use the Kinetic Theory of Matter to explain
(i) the Pressure Law
(ii) how a drop of blue ink placed in a container of pure water
eventually spreads out to occupy the entire volume.
(7)
(c) Mr. Shah decided to do some measurements on one of his motor car
tyres. The temperature of the air in the tyre was found to be 200 C and
the pressure was 200 kPa above atmospheric pressure.
After driving for a few kilometers, he again measured the air pressure
of the tyre and found it to be 240 kPa above atmospheric pressure.
(i) What was the final temperature of the air in the tyre, assuming that
the volume of the tyre remained constant?
(ii) What other assumption is necessary for your calculation to be
valid?
(5)
(Atmospheric pressure =100 kPa)
Total 20 marks
3. (a) (i) Describe FOUR ofthe properties ofelectromagnetic waves.
(ii) X-rays and ultraviolet radiation are both electromagnetic in nature.
For EACH ofthese two types ofradiatiorl, name TWO means of
detection.
(8)
(b) The distance between the earth and the moon has been measured
by transmitting a laser beam from earth to a reflector on the moon and

measuring the time taken for the reflected beam to be received


back on earth.
(i) The laser used had a wavelength of450 nm. What was its
frequency?

(Velocity oflaser light=3.0 xl08 m Sl)

(ii) The transit time was found to be 2.5s.


Calculate the distance between the earth and the moon.
(iii) A prism such as ABC shown in Figure I can be employed as a
reflector for a laser beam as represented as WXYZ.
Determine the relationship between eA AND eB.
B

A'

I' oR 'c

~'"I

Figure 1
(iv) Find the condition which 9B must satisfy in order that total internal
reflection occurs at AB.
(12)
Total 20 marks
4. (a) Compare the characteristics of zinc-carbon primary cells with those of
lead -acid batteries by drawing and completing the comparison table
show below. You should indicate typical numeric values wherever
possible.
(8)
LEAD-ACID

ZINC-CARBON

Terminal voltage
Maximum current
Internal resistance
Rechargeability
(b) (i) Calculate the power output of a lead-acid battery of terminal
voltage 12V when supplying a load of 80A.
(ii) The energy storage capacity of automobile batteries is usually
quoted in Ampere-hours (Ab). Energy storage capacity may also
be expressed in joules. Conversion from Ampere-hours to joules
uses the following formula: ,
Energy (joules) = Terminal voltage x Ampere hours x 3600.

Physics June 2006 P3 - Page lof2

43

44
Calculate the energy in joules stored in a 12 V, 240 Ah battery.
Express your result in megajoules.
(6)
(c) Electric cars employ an array of batteries to drive an electric motor

which in tum drives the road wheels. The array may consist ofN

columns of batteries, two of which are shown in Figure 2. An

equivalent circuit ofthis arrangement is shown in Figure 2.

f"'---" - - 'J:
::c
I--=- I
j

A:fray I :
of I ::::
12V I-=-

batteries I

I --b'L T _

-=""7

::=:::
_I

I
I

I:20Vor

30kW d..c. JIlOmr

_.-

Element

Bi
11
Helium

Atomic mass unit


211.99127 u
207.98201 u
4.002604 u

(iii) Calculate the release in the energy in the first nuclear reaction at
a).

(12)

u= 1.66xl0-27 kg

c=3.0x108 ms- 1

Figure 2
Total 20 marks
(i) Determine the minimum number of 240 Ah batteries which can

supply the POWER REQUIREMENT ofthe motor.


(ii) Determine the number of 12 V batteries which must be connected

across the motor to supply its rated voltage. Hence find N, the

number of columns and the total number of batteries required.(6)

Tota120 marks

END OF TEST

5. (a) Radioactive isotopes have been used in the following area:


(i) Medicine
(ii) Industry and Agriculture
Describe briefly, TWO useful applications in EACH of areas (i) and (ii).

Give an example of the isotope or type of radiation in EACH of your four

applications.
(8)

(b) (i) Represent EACH ofthe following nuclear reactions in the

standard form \X ---+ cdY+ efW:

a) Bismuth (Bi) with a mass number of212 and an atomic

number of 83 emits radiation to form Thallium (11) with a


mass number of208 and an atomic number of 81.
b) Carbon (C) with a mass number of 14 and an atomic number

of 6 emits radiation to form Nitrogen (N) with a mass number

of 14 and an atomic number of 7.

c) More unstable Technetium (mTc) with a mass of99 and an

atomic mass of 43 emits radiation to form more stable

Technetium (Tc) with same mass and atomic numbers.

(ii) Determine the number of protons in Bismuth and calculate the

number of neutrons in that same element.

Physics June 2006 P3 - Page 2 of 2

44

45
MECHANICS

PHYSICS - PAPER 2 - JUNE 2005

General Proficiency Time: 1Y2 hours.

2. (a) (i) State the principle of conservation of energy.

1. You are to spend no more than % hour on this question.

In this question you

find the refractive index of a rectangular block

of glass.
A student was given this experiment as a CXC SBA activity and
produced the following results.
Angle of incidence
i1degrees
10.0
20.0
30.0
40.0
50.0
60.0

Angle of refraction
r/degrees
6.0
12.0
18.0
24.0
30.0
36.0

S'ml
":

r on graph paper.

---------------~-------------------------------------------------------[1]
(iii) Give ONE example of a body possessing potential energy.

(iv) Define the term 'kinetic energy'.

Sin r

------------------------------------------------------------------------ [1]
(v) Give ONE example of a body possessing kinetic energy.

[4]

(a) Complete the table above.


(b) Plot a graph of Sin f against Sin

------------------------------------------------------------------------ [2]
(ii) Defme the term 'potential energy'.

[10]

(c) Find the slope, n, of the graph.

------------------------------------------------------------------------------ [4]

------------------------------------------------------------------------ [1]
(b) At a football match between two college teams, the referee ordered
a free kick. The ball of mass 1.5 kg was placed at rest. The kick
was about to be taken by an eager footbailer. What was the
potential energy of the ball just before the kick was taken?
------------------------------------------------------------------------------ [1]
(c) The footballer kicked the ball and it was caught by the opposing
goalkeeper, 4 metres above the ground.
The ball was travelling at 10 ms -1.
(i) Calculate the potential energy of the ball just before it was
caught. ---------------------------------------------------------------

(d) What does the slope, n, ofthe graph represent?


------------------------------------------------------------------------------ [1]
(e) Draw a labelled diagram identifying the apparatus used and angles
measured by the student to obtain the results.
[6]

-----------------------------------------.------------------------------- [3]
(ii) Calculate the kinetic energy ofthe ball just before it was caught.

(f) A ray of light is incident at an angle of 35. With the aid of dotted
lines use your graph to calculate the angle of refraction produced.
<

------------------------------------------------------------------------- [3]
(iii) After the ball was caught, what was the kinetic energy
converted into? -----------------------------------------------------

------------------------------------------------------------------------------ [5]
Total 30 marks
PHYSICS - June 2005 - Page 1 of 5

-------------------------------------~----------------------------------- [2]
Total 15 marks

45
\0

46
(c) [n Figure 1 below the primary of the transformer is connected to a
battery and a switch while the secondary is connected to a centre
zero galvanometer.

3. (a) Defme the 'heat capacity of a substance' and state its SI unit.

----------------------------------------------------------------------------- [3]
(b) Name the THREE modes of heat transfer.
..L

----------------------------------------------------------------------------- [3]

(c) A well-insulated hot water tank is used to supply hot water to a


residential dwelling house. The immersion heating element inside
the tank has a power rating of2200 W and the tank contains 125 kg
of water at 28C. Calculate
(i) the heat energy supplied by the heating element in 2 hours

Figure 1
------------------------------------------------------------------------ [3]
(ii) the heat energy supplied to the water, given that its temperature
increases to 58C

-----------------------------------------------------------------------[3]
[specific heat capacity of water is 4200 J kg-1K-1]
. (iii) the heat capacity of the tank, assuming that the tank and the
water reach the same final temperature of 58C.

------------------------------------------------------------------------ [3]
Total 15 marks
4. (a) Explain what is meant by the term 'magnetic field'.
----------------------------------------------------------------------------- [2]
(b) TWO bar magnets are close to each other so that their magnetic
fields interact. Sketch the field pattern for the TWO arrangements
shown below.

(i)

(ii)

INS I

IS

(i) Describe what would be observed on the galvanometer when


the switch is closed.

------------------------------------------------------------------------- [2]
(ii) Explain your answer.

------------------------------------------------------------------------ [3]
(iii) Describe what would be observed on the galvanometer if the
switch is subsequently opened.

------------------------------------------------------------------------ [2]
(iv) Deduce what would be observed on the galvanometer if the
battery were replaced by a low frequency a.c. supply.

.----------------------------------------------------------------------- [1]
(v) How would these observations be affected if the number of
turns on the transformer secondary were significantly
increased?
------------------------------------------------------------------------- [1]
Total 15 marks

[4]
PHYSICS - June 2005 - Page 2 of 5

46

..

47
5. (a) Identify the logic gates described in the first column ofthe table
below and hence complete the table by writing in the names and
logic symbols in the appropriate columns.
[6]

SYMBOL

NAME

DESCRIPTION

(c) The logic circuit of Figure 2 is incorporated into the electrical


system of a car as shown in Figure 3 below, and switches 8 I and 82
are operated according to the table below.
~n----..A

Output is always low except


when both inputs are high
Output is always high except
when both inputs are low
Output is always high except
when both inputs are high

81

:2

I\V

(b) Figure 2 shows a logic circuit.


~o
82

.8

Figure 3

ENGINE
ON
SWITCHSl

ENGINE
OFF

ON

OFF

ANY
STATE

ANY
STATE

f--- --------------

SWITCHS2
y

DOOR
OPEN
ANY
STATE

DOOR
CLOSED
ANY
STATE

ON

OFF

-----

Explain what happens if:


(i) the DOOR is OPEN and the ENGINE is OFF.

----------------------------------------------------------------------- [2]
(ii) the DOOR is OPEN and the ENGINE is ON.

Figure 2
Examine the logic circuit shown in Figure 2 and complete the
following TRUTH TABLE:

A
0
0
1
I

B
0
1
0
0

----------------------------------------------------------------------- [1]
Total 15 marks

ENDOFTE8T
---

[6]

PHYSICS - June 2005 - Page 3 of 5

47

48
PHYSICS - PAPER 3 - JUNE 2005

General Proficiency

natural:

Time: 1 hour

1. (a) An acetate rod may be charged positively by rubbing it with a dry cloth,
whereas a polyethylene rod will be negatively charged if similarly
rubbed. Explain why this occurs. Explain, with the aid of diagrams if
necessary, why it is possible to pick up small bits of paper with either
ofthese charged rods.
[8]

artIfiCIal:

[6]

+++

-----Figure 1
(i) In your answer booklet sketch the electric field pattern inside the
cloud, treating the charge distribution as that for TWO parallel
plates.
(ii) If this cloud is directly over a tall building, explain how the
distribution of charges in the building's structure will be affected.
(iii) The air between the cloud and the building breaks down and there
is a flash of lightning. Deduce the direction in which the electrons
will flow in this lightning bolt.
[6]
Total 20 marks

235

92 U

---i>

54

+ He + Energy

Xe +

90
38

Sr + 3( 0 n) + Energy

Nuclide

Atomic mass I u.

235U

235.04393

2i~ Bi

211.99127

20sTl
81

207.98201

51Xe

142.93489

~Sr

89.90730

j He

4.00260

In

1.00867

92

(c) A cloud has the charge distribution shown in Figure 1 below.


++++++++

4
2

Table 1: Showing nuclide and atomic mass

(ii) the number of electrons involved in this current flow.


1.6 x 1O-19C)

1
0n

---i>

208
81 Tl

Table 1 below gives the data for these nuclides where u = 1.66 x 10-27 kg.

(b) The positively charged acetate rod and the negatively charged
polyethylene rod are made to touch ea<;h other, causing 31lA of current
to flow from one rod to the other for a period of 4 ms. Calculate
(i) the amount of charge which flows through the rod

(Charge of electron

212.
83 BI

......

. . . _-----

(i) Calculate the number of neutrons in Bismuth (Bi).

[3]

(ii) Detennine the atomic mass number of Xenon (Xe).

[2]

(iii) Calculate the energy released in EACH nuclear reaction and the
ratio of the larger to the smaller. Deduce the recommendation
[7]
that the engineer will give to the investor.
(b) Radioactive materials emit alpha (a), beta (13) and gamma (y) radiation.
Table 2 summarises some of the properties of these types of radiation.
Copy the table in your answer booklet and complete the missing
entries.
Table 2: Properties of a, IS and 1 radiation

haviourin
ectric field

Rangei
air

Type of tracks in
cloud chamber

a
2. (a) An investor is interested in a nuclear power station project. He has the
option of using natural or artificial radioactive decay processes to
obtain energy. In helping him to decide, he consults an engineer who
compares the energy obtained from the' following TWO nuclear
reactions:

f3

PHYSICS - June 2005 - Page 4 of 5

faint track
LJp()()r i()!1izers)
[8]
Total 20 marks

48

49
3. (a) With the aid of a neat, clearly labelled diagram, describe an experiment
to prove the relationship between the angle of incidence, i, and the
[8]
angle of reflection, r, for a ray of light incident on a plane mirror.

The centre of gravity of the bicycle is vertically above 0, a point on PQ


where PO = 30 em.
I
j<e-c. g.llwng this line

(b) Figure 2 is a side view of ABCD, a prism made from fused quartz (a
type of glass) of refractive index 1.46.
A

pIE

,0

il'IQ

l}_~m

Figure 3

tJ

Given that the normal reactions of the ground on the front and rear
wheels are R 1 and R2 respectively, write TWO equations, in Rl and R 2,
which satisfY the conditions of equilibrium.
[4]
(c) Kenny and Candy decided to sit on a see-saw while visiting a local play
park. Candy, of mass 50 kg, sat 250 cm from the pivot of the seesaw.
(i) Where should Kenny, of 60 kg mass, sit so that a state of stable
equilibrium exists?
[5]
(ii) What should Kenny do if he wanted to elevate Candy?
[3]
(Acceleration due to gravity 10 N kg-I)
Total 20 marks

'c

Figure 2
The ray MN is incident on face AD as shown.
(i) Show that MN will be totally internally reflected from the face
AB.

Redraw the diagram in your answer book.


(ii) Determine, em. the angle which the reflected ray makes with the
face BC at point P.
(iii) Sketch on your diagram the ray from point P, showing clearly the

path from P through to its emergence from the prism.


Indicate any refraction.
Total 20 marks
4. (a) With the aid of a labelled diagram describe an experiment to determine
[8]
the centre of gravity of an irregularly shaped sheet of cardboard.
(b) Figure 3 shows a bicycle of mass IS kg resting in a vertical position,
with the front and back wheels touching the horizontal ground at points
P and Q respectively, where PQ 1 m.

5. (a) (i) Distinguish between EACH of the following pairs of terms:


a) 'Solidification' and 'fusion'
b) 'Condensation' and 'vaporisation'
[4]
(ii) Describe FULLY the process of sublimation.
[2]
(b) For the changes of state mentioned in a) and b) above to take place
energy must be added to or removed from a substance. Give the general
name for this type of energy and state what happens to the temperature
during these processes.
[2]
(c) A student placed 700 g of water at 28C in a freezer. After 6 minutes
and 15 seconds the water was transformed to ice.
Calculate:
(i) the heat energy transferred from the water during the temperature
change
(ii) the latent heat of solidification, given that 235200 J of heat energy
was transferred during the change of state
(iii) the rate of heat energy transfer for the entire ~rocess.
[5]
(Specific heat capacity of water ::::: 4200 J kg- K-1)
Total 20 marks
END OF EXAM

PHYSICS - June 2005 - Page 5 of 5

49

\'1

50
no

2(a)

PHYSICS PAPER - JUNE 2003

GENERAL PROFICIENCY - 1 ~

_..

Compare a common dry cell with a car battery by completing the table
below.
TABLE 2

1. Table 1 shows the relationship between the temperature, T, ofthe

Feature

measuring junction of a thermocouple and the thermocouple emf: E,


that is measured by a millivoltmeter.

CommOll Dry Cell

Large
Heavv

Size I weight

Electrolyte

Table 1

Thermocouple emf ElmV

Ammonium
Chloride paste

Emf

Temperature Tf'C

Lesd
Lesd Dioxide

Electrodes

(a)(i) Use1fle values in the table to plot E against T. Draw your BEST
(10 marks)
straight line.
(ii) Find the slop ofthe graph.
MM

Car Battery

.W.

.-.

~~.

Current

Small to

Capacity

Moderate

...

(6 marks)

~----_-----_----

A1

(5 marks)

(b)(i) Use your straight line graph to determine To, the temperature at which
the thennocouple emfE = O.
--------_._._------------------------_.------------------_.._------------------------
(3 marks)
(ii) Use your straight line graph to determine, Ex the thermocouple emf

when T = 200C.

,_ _J
Figure 1
A generator and a battery of secondary cells are connected through an
ammeter as shown in the circuit of Figure 1.
(i) Indicate the direction of current flow on your diagram.
(l mark)
(ii) Show, on the diagram, the polarity of the ammeter for it to properly
(1 mark)
measure the current.
(iii) Is the battery being charged or is being discharged?

(b)

(1 mark)
(iii) Given that E = S(T - To), use the relationship to find T when E =

0.4mV.

(3 marks)
(iv) Determine the difference between the experimental value as given in

Table 1 and that obtained using the fOIDlula in part (iii), and explain

the significance of this difference.

(1 mark)
A calculator battery of tenninal voltagee 3 V supplies a current of
0.1 mA to the calculator circuit.
(i) Calculate the resistance ofthis circuit.

(c)

(2 marks)

--------------------------------------------_._--------------------------------------
(3 marks)
(ii) Calculate the power being delivered by the battery.

(v) Compare a thennocouple and a laboratory mercury in glass

thennometer in tenns of (i) range, (ii) speed ofresponse, (iii)

sensitivity.

-----------------------------------------------~

..----------------------------------

(6 marks)

Total 30 mark.s
PHYS - P2 + P3 - JU 2003 - Page 1 of 5

(3 marks)

Total 15 marks
50

\;,

51
'Write down the formula that defines the moment of a: force, giving the
meaning of EACH symbol and the unit in which it is usually expressed.

3(a)

(d)

(1 mark)
(i) Use the results ofI(i), (ii) and (d) above to find the mass ofthe
counterweight that must be used ifFA= 2SN is just enough to start
raising the boom.
(g = 10 m S2)
.
(3 marks)
Total 15 marks

(4 marks)

Sate the Principle pf Moments.

(b)

(2 tnaIts)
Secwity guard posts in the Caribbean are usually equipped with a boom
and counterweight arrangement as shown in Figure 2. It allows the
security guard to use a relatively small force to raise the heavy boom.

(c)

~pt

FA

rn

Force

.pplled

t lJ
tw

/Bo9m

"t.

,.c

"liGON

'-PboIB

PoInte ./

(2 marks)
Figure 3 and 4 below are representations of current-canying
conductors. Sketch the magnetic field associated with the conductor in
each of the cases described below.
(i) The conductor is a straight wire carrying a steady current directly out
ofthe paper

'I

3m

0.8",

(i)

Explain what is meant by a 'magnetic field'

!l

here

4(a)

s>.lII flIIJowed.'" \ .

o.:t5 m

What is the value ofRc when the boom first loses contact with its
cradle?

[I
/'

6m

./

1/

./

Figure 2.

Figure 3

In Figure 2:
- Rc is the normal reaction ofthe support on the boom at point C.
- FAin the force applied at point A to raise the boom.
- W is the weight of the counterweight.

(2 marks)
(ii) The conductor is tightly wound into a solenoid carrying current in the
direction indicated.

Write down expressions including W, Rc and FA for


(i) the total counterclockwise moment about pivot B.

Figure 4

(4 marks)
(ii) the total clockwise moment about pivot B.

(d)

(2 marks)
A flat coil is connected to a small bulb and a diod. The assembly fixed
in position near a current-carrying solenoid is shown in FigureS below.

(1 mark)
PHYS - P2 + P3 - JU 2003 - Page 2 of S

51

52

r,

(1 mark)

(iv) A car travelling arowd a bend.


(1 mark)

\J

(v) An electron moving arowd the nucleus in an atom.


(1 mark)
(b)

A small bob ofmass 0.2 kg is attAched to a string oflength 0.5 m and


whirled in a vertical circle with speed of 2.5 mls as in Figure 6.

Figure 5
(i) What will be the effect on the magnetic flux inside the flat coil if the

current through the solenoid is steadily increased?

(1 mark)

(n) Will the bulb glow? Explain.

(5 marks)
(iii) Ifthe diode is removed so that the flat coil is connected to the bulb
alone. indicate by use ofa tick (v) which ofthe following effects will

cause the bulb to glow more brightly.


EFFECT

YES

Figure 6

(i) Indicate on the diagram the forces acting on the bob when it is in
position C.
(2 marks)
(n) Calculate the kinetic energy ofthe bob.

NO

(I) Increasing the number of turnll on the flat ooi1

(3 marks)

(li) Placing a piece ofsoft iron in the core of the solenoid

(iii) At which of the three points A. B. C is the tension in the string:

(iii) Decreasing the rate ofchange of the cummt through the solcmoid.

(3 marks)
Total 15 marks

a) GREATEST --------------------------------------(1 mark)


b) LEAST
-------------------------------(l mark)
(iv) If the string is released when the bob is at C. what is the tension
immediately after the release ofthe string?

5(a)

State the natW'e ofthe unbalanced force which is responsible for


circular motion in the following:
(i) A simple pendulum

(1 mark)
(v) Sketch the path ofthe bob as it travels to the ground if it is released at

C.
(1 mark)

(ii) The earth moving around the SWl.


(2 marks)
Total 15 marks

(l mark)
(iii) Clothes rotating in a spin dryer.

END OF TEST
PHYS - P2 + P3 - JU 2003 - Page 3 of 5

52

\<{

53
PHYSICS PAPER 3 - JUNE 2003
GENERAL PROFICIENCY 1 br

1(a)

(b)

With the aid of a neat clearly-labelled diagram, describe an experiment


to show that when light is reflected from a plane surface the angle of
incidence and angle ofreflection are equal.
(8 marks)
Some Internet Source Providers (companies that provide connection to
the Internet and are known as ISP's) communicate with their customers
via line-of-sight microwave links. A microwave antenna at the
customer's premises should be able to 'see' the ISP's antenna in order
that customer can carry on two-way communication with the Internet.
Consider such a link operating at 2.5 x 109 Hz with 30 km between
customer site and ISP antenna.

- -

~!"'--

- -

(c)

Another transformer is used to further step down 220 V to 110 V. Its


input current is 3A and the output current is 5.6 A. Calculate its
(8 marks)
efficiency.
Total 20 marks

3(a)

Explain in terms of the kinetic theory of gases:


(i) How the air inside a container exerts pressure on the walls of the
container
(ii) Why the pressure of a gas at constant volume, increases when its
temperature is increased.
(8 marks)

.:-:.- :P1sIoDn
..: ..
'

,:

..... t
I

_.

'
_

,,:.

'
I

'
I

_'

'
_

....
I

-------

;-:';;~~-'B';~~;-:~;-'~/'~;:'' .':

'
'_

;-: : ;-: : /-: ;-'.: ;': : ;-:: :-.

" .. '
...

'
'

.. : .. .: .. ,:

... _ ' "


_

'''.'
...

'._

..

,
_

..
I

-:.' -:.' -:". -:". -:. '. -':' -:",-


.

:". :.:'_ . ;\. . :.:: . :~. ;:.... :~


'

SideD

Figure 2
3

A closed insulated cylindrical tube of inner volume 0.8 m is fitted with


a heater and piston as shown in Figure 2. The piston may be assumed to
Figure 1
have
the following properties:
(i) What is the wavelength and period of the microwave signal between
-it is a perfect heat insulator
customer and provider?
-it slides along the tube without friction
(ii) Calculate in microseconds the time it takes for a signal to travel

-its volume is negligible compared with the volume of the tube


between customer and provider and back again to customer.

2
Initially, the piston of area 100 cm is stationary at the centre of
(iii) Explain why the customer may still have communication even ifher
the tube, the heater is OFF and the gas on side A is at 300 K and exerts
antenna is unable to see the ISP's antenna directly.
s
8
1
a pressure ofIO Pa.
(The velocity of light C = 3 xl0 ms )
(12 marks)
(i) Calculate the force on face A of the piston.
Total 20 marks
(ii) Deduce (you do not need to calculate) the pressure, volume and
temperature on side B, giving reasons for your answers.
2(a)
Draw a clearly-labelled diagram of a transformer and use it to explain
(iii) Describe what happens to the piston when the heater is turned ON, and
(8 marks)
the principles of its operation.
also what happens to the pressure of the gas on side B at the same time.
(iv)
After the heater has been on for several minutes the temperature of the
(b) A transformer is used to step down a voltage of 11 KV to 220 V. Given
3
gas
on side B is now 400k and its volume is 0.2 m . Calculate the new
that the transformer has 3200 turns in the primary coil, calculate the
pressure of the gas on side B.
(12 marks)
number of turns in the secondary coil.
(4 marks)
Total 20 marks
PHYS - P2 + P3 - JU 2003 Page 4 of 5

53

54
4(a)
t1

Explain what is meant by the term l pressurel.


With the aid of a labelled diagram, describe a simple experiment using
a tall, cylindrical tank to demonstrate that pressure varies with depth in
(8 marks)
a fluid.

An aquarium has a rectangular base that measures 60 cm by 30 em. Its


height is 45 em and sea water of density 1150 kglm3 is used to fill it.
(i) Calculate the weight ofwater in the aquarium and the pressure on a
fish at the bottom ofthe aquarium due to the water alone.
(n) Calculate the pressure on a fish at the bottom ofthe aquarium if'
atmospheric pressure of 100 kPa acts on the surface of the water.
(Acceleration due to gravity g = 10 m 8,2)
(12 marks)
Total 20 marks

(b)

5(a)

(d)

Three nuclides X. Y, and Z have' half-lives 1 minute, 24 hours and 10


years respectively. Select the nuclide which would be MOST useful as
a radioactive tracer in medical diagnosis. Give TWO reasons for your
choice.
(5 marks)
State TWO other applications ofradioactivity.
Total 20 marks
END OF TEST

Copy and complete the following table in your answer booklet. The
table summarizes the nrooerties of 3 tvDes ofradiation.
Type of ra6UIon I
Charge
I Form of energy I
STOPPED BY

I eo- (negative)
._....

Alpha
Beta
Gamma

KE

--_.

5 mmofaluminium

(6 marks)
(b)

[5.

x
x
x
x

x x
x.x
x x'
x x

x
x
x
x

x
x
x
x

A source S emits alpha,. beta and gamma radiation. Sketch the path of
EACH type ofradiation in the region M which has a uniform magnetic
(3 marks)
field pointing into the plane the paper.
(c)

The following symbols represent four nuclides,


A 58 ,
B 54 ,
CS9 ,
D59

29

27

29

30

(i) Which TWO nuclides are isotopes of each other?


(n) One of these nuclides can be produced from another in the group by

radiative emission of an -particle. IdentifY the parent and daughter


nuclides and give the corresponding nuclear equation,
(iii) Another ofthese nuclides can be produced from another member ofthe
group by the radiation of a beta particle. IdentifY the pair and give the
corresponding equation.
(6 marks)
PHYS - P2 + P3 JU 2003 - Page 5 of 5

54

I'?

55
PHYSICS PAPER 2 - JUNE 2004
GENERAL PROFICIENCY 1 Y2 hrs
1.

[3]

~-----------------T=0-ta-:I--=3-=-0 marks

You are to spend no more than - hour on this question.


A car that cannot start because of a "dead" battery may sometimes be
"push started". This involves applying an external horizontal force to
accelerate it up to a speed at which the engine can operate on its own.
The car will then increase its acceleration in the direction in which it
was being pushed.
The dynamics of a "push start" was investigated by measuring the
distance covered from rest at 5 second intervals. The results obtained
are tabulated below
Timefls
Distance slm
Velocity vIm S1

0
0

5
2.80

10
11.4

15
25.6

PUSHING PHASE

20
45.6

25
75.5
5.88
ENGINE

30
35
122
207
13.4
20.9
DRIVEN PHASE

2(a)(i) Define 'specific heat capacity'.

_ _____:--::-:-:---------::-=-_:__-____:---[2]

(ii) List THREE differences between specific heat capacity and heat

capaCity.

(b)

_ _--:-_ _~------:----:------[3]
A copper cube has side 8 cm. Determine its

(i) volume in SI units

(ii) mass

[3]

(Density of copper

8 890 kg/m3l

[3]

Table 1

(a)

The velocity v and distance s during pushing are related by the equation
V

(b)

(c)

(c)

JO.5s

Use this relationship to complete the table


[2]
During the pushing phase the acceleration of the car is constant. It is
also constant during the engine driven phase. Plot the complete
velocity-time graph of this "push-start" for the first 35 seconds. [10]
Find the acceleration of the car while it is being pushed.

The cube is heated to 90C and then placed in a closed, thermally


insulated container of negligible heat capacity containing 1 kg of water
initially at 3SoC. Calculate the final equilibrium temperature of copper
and water.

[S]
- - - - - - - - - - - - - - - - - - = T : - o t - a : - l::-: :16 marks

(d)

(e)

(f)

_ _ _ _ _ _ _ _....-.,,-
[6]
The mass ofthe car is 1.25 x 103 kg. Determine the NET force needed
to give this mass the acceleration calculated in part (c).
[3]

3(a)

~-

Wave Property
Speed = 3 x ]08 mls
Travels in a vacuum
Transverse wave
Longitudinal wave
Progressive wave
Transfers energy
Can be diffracted
Can be reflected
Can be refracted

One of the forces opposing the motion of the car is friction in the
engine and other moving parts. Explain the nature ofthis friction
_ _ _ _ _ _ _ _ _ _ _ _ _ _ _ _ _ _ [3]
The car in this example experiences constant resistive forces of 14QO N.
Determine the external pushing force that MUST be applied to give the
car the acceleration calculated in part (c).

Microwave

'

SQund

Table 2

[9]
In a track race the starter of the race may be at some considerable
_ _ _ _ _ _ _ _ _ _ _ _ _ _ _ _ _ _ [3]
distance from the runners. Consider the case where the distance
Determine the distance covered by the car during the FIRST 22 seconds
between a runner and the starter is 40m; calculate the time taken for the
of its motion.
.
sound from the starter's gun to reach the runner.
55
PHYS - P2 + P3 - JU 2004 - Page I of 5
(b)

(g)

Complete the table below by placing a tick f/I) in the appropriate


column..\...... f - - - - - - - - - - - - - - t".I._'t"'--- "'" - - ... _...... . -- .

I'

(Speed of sound = 350 mls)

56
(i) Analyse the circuit and complete the truth table in Table 4.

'J

(c)

_ _ _ _ _ _ _ _ _ _ _ _ _ _ _ _ _ [3]
The average speed of a world-class sprinter at the start of the race may
be 6 mls.Travelling at that average speed, what distance could this
sprinter have covered in the time taken for the sound to reach him?

Inout
A
0
0
0
0
1
I
1

_ _ _ _ _ _ _ _ _ _ _ _ _ _ _ _ _ [2]
Total 14 marks

4(a)

Draw circuit symbols for the following logic gates.


NOT
NAND
NOR

_
[3]

C
0
1
0
I
0
1
0
1
Table 4

-
~

[6]

(ii) State, in words, the conditions under which the alarm will sound.

(b) Draw truth tables to show the functions of a


(i) NAND gate

-------~--------~T;;:;-0:::tal:;-:;n18

Sea)
(ii) NOR gate.

(c)

B
0
0
1
I
0
0
1
1

Output
X y Z

[3]

_ _ _ _ _ _ _ _ _ _ _ _ _ _ _ _ _ [3]
The circuit
in Figure I can be used to sound an alarm when
certain conditions occur with an electric kettle.

(b)

[3]
marks

Differentiate between a scalar quantity and a vector quantity. Give


ONE example of EACH.

--,--_---:-_--,--_---:--::--.---:-::--_---:-_----,- [4]
Figure 2 shows the velocity vector, v, of a ball at some instant after it is
thrown in the air.

It

z
y

o
Figure I

A. Band C are logic switches which are connected to the cover, water
level and power switches and their corresponding states of cover. Water
level and Dower switch are shown in Table 3 bel
Switch

Kettle open
Water below heating
element level
Power switch on
Table 3

0
Kettle "closed
Water above heating
element level
Power switch off

Figure 2

Using a protractor and ruler, draw on Figure 2, two other vectors at


right angles to each other whose sum is equivalent to v, and one of
which is along Ox.
[2]
(c) An object is acted upon by force~ of magnitude 6 N and 10 N inclined
at an angle of 45 to each other.
(i) In the space below, draw to scale a diagram showing the forces and
their resultant
'

PHYS - P2 + P3 - JU 2004 - Page 2 of 5

56

tb

57
PHYSICS PAPER 3 JUNE 2004

GENERAL PROFICIENCY -lhr

,
1(a)

Explain with the aid of a diagram the operation of a simple magnetic

00

re~

(b)

Figure 1 shows the starting circuit of a car, in which a low current


through the ignition switch operates a relay allowing a large current to
flow through thick conductors to the starter motor. The equivalent
circuit when the relay is closed is.shown in Figure 2.
Ignition
switdl

(ii) Find the magnitude ofthe resultant, and the angle it makes with the 10
Nforce.

+1'

Figure 1

[6]
Total 12 marks
END OF TEST

lZV

0.0010

Figure 2

(i)
(ii)
(iii)
(iv)

Given that a 12 V battery supplies 400 A to the motor and the


resistances of the conducting wires in this circuit are as shown,
calculate
the p.d. across the starter motor .
the power being supplied by the battery
the power being dissipated in the conducting wires
the power used by the starter motor to crank the car engine.
[12]
Total 20 marks

PHYS - P2 + P3 - JU 2004 - Page 3 of 5

57

....

58
2(a)

(b)

Define pressure and give its units. Describe with the aid of a diagram
an experimentto demonstrate that the pressure at all points on the same
horizontal level in a fluid at rest is the same.
[8]
Figure 3 shows a sketch ofa hydraulic jack which can be used to lift

heavy objects.

3(aXi)

(ii)

(bXi)

Ft

'''I<E-

(ii)

PIston 1

Al

(iii)

(c)

Figure 3

(i) Given Al = 0.002 m2 What pressure is exerted on the fluid if

F, = 1000N?

(ii) What is the area A2 if this applied force raises a van weighing 105N?
(Assume that pressure drops due to the weight ofliquid in the jack are
negligible.)
[6]
(c) The tube shown in Figure 4 is used to siphon water from container A to
container B.
. lUbe

4(a)
(b)

Air is pumped into a car tyre to iriflate it. Use the kinetic theory to
explain how the air inside the tyre exerts the pressure which keeps the
tyre Inflated.
As the car travels, the temperature ofthe air inside the tyre is heated.
Explain, again using the kinetic theory, what happens to the particles
when this occurs.
[8]
The car tyre contains air initially at a pressure of 195 kPa. After
travelling several kilometres, the temperature of the air inside the tyre
rises from 30C to 70C. If the tyre is rigid and does not expand,
calculate the new pressure inside the tyre.
Consider a different tyre under identical conditions to those given in
section (i). This tyre is more elastic and its volume increases by 10%
when the temperature of air inside it increases from 30C to 70C.
Calculate the final pressure in the second tyre.
Based on your results for sections (b) (i) and (b) (ii) above, state
whether it is preferable to have rigid tyres or more elastic tyres for
prolonged high speed driving.
[9]
What is the relationship between the force due to pressure on the inside
ofa tyre and the reaction force of the ground on the tyre? Explain how
you arrived at your answer.
[3]
Total 20 marks
[8] .
Describe an experiment to verify the laws of reflection.
Figure 5 is a magnified view of the cross section of one track of a
compact disk CD) and its read head.
Label

SeCtion
~ )

through
~mpad;

"""L.- Aluminium mirror

disk
(CD)

AI-:

-----

--::~_.~;~~====

_1:

Strengthening acryne layer

..

Direction of
movemel1t

::~

Air

-:::. -=JB

Figure 4

What is the pressures at Points I, 2 and 3 if the end of the tube which is
inserted in Container B is closed just before siphoning begins?
Atmospheric Pressure 1.013 x 105 Pa
Density of water = 1000 kg m')

[6]
Total 20 marks
PHYS - P2 + P3 JU 2004 - Page 4 of 5

Read head

Figure 5

58

t1

59

Information is stored on the CD as a series of 'hills' and 'valleys'


stamped into the aluminium mirror. The information is read by shining
light from a laser source at X through the clear plastic layer so that it is
reflected towards the sensor at Y, which has a very tiny opening.
(i) If the laser light beam follows the path shown in Figure 6, below,
calculate the refractive index of the clear plastic.
[5]
Label

Sedion

throogh
compact
disk
(CD)

.....

r.

StrengtheIllug aeryIie layer

l~AlumlDiummirrw

(iii) Identify the energy transformations that take place when the ball strikes .
the goal post and rebounds.
(iv) Is kinetic energy lost in the collision between the football and the goal
..
post?
Explain your answer.
(v) Explain why, "in reallifell, the ball would not travel at constant speed
but would slow down as it goes farther and farther from the kicker.
(vi) Draw a: simple sketch, without numbers, of a velocity time diagram that
shows the slowing down of the b~ll during its motion through the air.
[12]
Total 20 marks
END OF TEST

Direction cf
mo_t
Air
<

r,y

SENsoii'

7 , '\
A./

Read head

Figure 6

(b)(ii) The track moves to the left (read head does not move) so that EF
becomes the reflecting surface (see Figure 6). With the aid of a diagram
showing the beam reflected from surface EF through the plastic and
into the air, explain why the emerging beam will miss the sensor. [7]
Total 20 marks
Sea)

State Newton's Second Law of motion. Use Newton's Second Law to


explain the existence of the force of gravity on a falling object.
Describe an experiment in which a simple pendulum is used to measure
'g', the acceleration due to gravity.
[8]
(b) Consider the action of kicking an initially stationary soccer ball so that
it travels through the air, hits the goal post and rebounds. We can treat
it as an example ofthe application of Newton's Second Law.
.
(i) The ball has a mass of 0.44 kg and leaves the kicker's foot with an
initial velocity of SO ms-1. Calculate the momentum ofthe ball
immediately after it has been kicked.
(ii) Calculate also the kinetic energy of the ball immediately after it has
been kicked. .
PHYS - P2 + P3 - JU 2004 - Page 5 of 5

59

60
(e)

PHYSICS PAPER 2 - JUNE 2002


GENERAL PROFICIENCY 1 Y:zhrs
1

What would be the temperature ofilie liquid after 12 minutes?

You are to spend no more than % hour on this question.


(4 marks)
A liquid, A, initially at temperature T, = 28.4C, is heated by an
immersion heater in a container. The resulting temperature, T2, is
recorded at 1 minute intervals and the following results obtained.

Time t/mins
Temperature Tzi

C
Temp changes ATI

I 0
28.4

I I
36.0

I 2 I 3
43.1

52.5

I 4
60.5

I 5

I 6

69.0

75.1

Another liquid, B, has a much higher specific heat capacity than


Liquid A. On the same graph paper used in (b) sketch (DO NOT
PLOT) a line showing ilie approximate graph that would be obtained
if Liquid B was the working fluid.
(2 marks)

(g)

State TWO possible sources of error in iliis experiment and in EACH


case describe a method that might be used to minimize the
magnitude of the error.

I 7
84.3

I I

C
(a)

(f)

Complete the table by computing the temperature changes


AT=T2- T l.

(4 marks)

(b)

(4 marks)
Tota130 marks

Plot a graph of Temperature change, AT, against time, t.

(9 marks)

(c)

2(a)(i) Distinguish between density and relative density.

Find the slope, S, ofthe graph.

.__ ._------------~---------------------------------------------------------------.----------

(5 marks)

(d)

Cp the Specific Heat Capacity of the liquid is related to the slope S


by:

Cp 2000 J kg-l K"'. Find the Specific Heat Capacity of the liquid.

(2 marks)
(ii) Show why relative density has NO units.

(2 marks)
(b)(i) A block of wood measures 2em x 4em x 9em. Calculate its volume
in SI units.

(2 marks)

(2 marks)
(ii) Express the result in (bXi) in standard form.

(1 mark)
PHYS - P2 + P3 - JU 2002 - Page 1 of 6

60
\~

61
3

(iii) If the mass ofthe block is 57.6 x 10- kg, calculate its density.

(b)

Two such coils are connected together in the circuit of a hair dryer
by means of switches as shown in Figure 1.

(2 marks)
o

(c)(i) State Archimedes Principle.

..

(l mark)

HOY.

Figure 1

(ii) Use Archimedes Principle to explain why some objects float while
others sink in the same liquid.

This dryer has two power settings LOW and HIGH. A setting is
selected by opening or closing the switches. The table below shows
the correspondence between power setting and the states of the
switches.

(2 marks)

POWER
SETTING
LOW
HIGH

(iii) Explain how it is possible to predict whether a given object will float
or sink in a given liquid.

(2 marks)

(d)

nnTT~-~

0'''-;81t--------4

SI

S2

S3

OPEN
CLOSED

CLOSED
OPEN

OPEN
CLOSED

Draw the equivalent circuit for EACH setting by redrawing the


circuit without the switches. In your equivalent circuit, open switches
are to be left out entirely and closed switches are to be replaced by
an unbroken line.

Would the block in part (b) float or sink in a liquid of density 810 kg

m-3?
(1 mark)
Total 15 marks

(2 marks)
3(a)

Calculate the resistance of an electrical heating coil rated at 275 W,

I1OV.

(c)

Identify the type of connection employed in EACH case.

(2 marks)

(4 II18IkS)

(d)

Compute the effective resistance of EACH connection.

(3 marks)
PHYS-P2 + P3-JU2002-Page 2 of6

61

62
(e)

Write down TWO characteristics of a fuse suitable for use in a

circuit such as that of the hair dryer.

(c)

(2 marks)

(f) Draw a simple sketch showing how a fuse should be connected in a

circuit to protect a load. In addition to the fuse and load your circuit

should include: live and neutral wires, and an on/off switch.

Liquid X

Mercury
Figure 3
The apparatus shown in Figure 3 may be used to determine the
density of an unknown liquid X.
The tube has a unifonn cross-section and the height ofthe water
column is twice that ofliquid X. A and B are at the same horizontal
level. The pressure difference between points A and Cis 2.5 X 103
Pa.

(3 marks)

Total 16 marks
4(a)

Describe an experiment to show that the pressure in a liquid


increases with depth.
(i)

~----- ~-------------------.------------.------------------

Calculate the height ofthe water column.

(4 marks)
(Density ofwater = 1000 kg m3; g = 10m soo,2)
(3 marks)

(b)

(ii)

'"1::

Express Pa -Po in terms ofPA and Pc. [Hint: the answer can be

deduced without resorting to any further calculation.]

(2 marks)

Figure 2
(iii) What is the relationship between Pc and Po?
A container is filled with a liquid of density p as shown in Figure 2.
Express the pressure difference between Pb - Pain terms of p and the
heighth.
.
--~._---~._------------------------------------.._-------..
----------------------

----------

(l mark)

(iv) Determine the density ofliquid X.

---------------.------------------------------._-------_.-------~ _.-------------------------

(1 mmk)

--------------------------------------------------..----------------------------------------
(4 marks)
PHYS - P2 + P3 - JU 2002 - Page 3 of 6
Total 15 marks
62

63

5(a)(i) Draw a diagram showing the structure ofthe carbon atom 12c.
6

(5 marks)
(ii) Carbon -14 is often referred to as an isotope of carbon. Explain what
is meant by the tenn 'isotope'.

(1 mark)

(b)

Define llalf-life' when used in reference to a radio-active nuclide.

(2 marks)
(c)

A radioactive tracer has a half-life of 18 hours. Calculate the fraction

of the initial activity remaining after 90 hours.

(3 marks)
(d)

In a radioactive process the element Radium (Ra) emits gamma


radiation and decays to Radon (Rn) and Helium (He). The equation
of this process is
226Ra -+ ARn + 4He . Determine the values of A and Z.
88

86

(4 marks)
Total IS marks

END OF TEST
PHYS - P2 + P3 - JU 2002 - Page 4 of 6

63

64
2(a)

CXC PHYSICS PAPER 3 - JUNE 2002

GENERAL PROFICIENCY -1hr

1(a)

(b)

State the law ofconservation ofenergy and differentiate


between kinetic and potential energy.

(b)

(3 marks)

Describe an experiment to verify Snell's' law ofreftaction. State wbat


apparatus you would use, give a briefdescription ofthe method you would
(8 marks)
use and state how you would arrive at your conclusion.
An optical device consists of two glass blocks ofdifferent indices of
refraction, arranged as shown in Figure 3. The refractive index from air to
Medium 1 is 1.2. The refractive index from Medium 1 to Medium 2 is 1.25.

A spring-mass system oscillates between X and Y on a


frictionless table as shown in Figure 1.

Spring

+x
~
0;

es

1 .

n-np~

'"

.'

-~~:

Table

Figure 1

Describe what energy transfonnations take place as the mass travels from X
to Y. 0 is the equilibrium position ofthe spring.
(5 marks)
(c)

A 0.5 kg block slides from rest down an inclined plane as shown in Figure
2. The velocity at the bottom ofthe plane is 6 m Sl. The vertical distance

POis2.0m
(g = 10 m sz)

Figure 3
(i) If light is incident at 60 on the block with the smaller refractive index,

determine the angles e1. e2, e3, 9 4 and es


(ii) What is the velocity oflight in Medium 1 ?
8
c = 3 X 10 ms 1

(12 marks)
Total 20 marks

.p
3(a)

Figure 4 shows a simplified diagram of a permanent magnet d. c. motor


(seen from one end) in which a rectangular coil is suspended in a uniform
magnetic field.
N

o
(i)
(ii)
(iii)

(iv)

Figure 2
Calculate the potential energy at the top ofthe incline and the kinetic

energy at the bottom ofthe incline.

Why are these answers in part (i) different?


Ifthe block were to be dropped from P, what would be its velocity at 01
If part (iii) were performed on the moon where 9 '" 2 m S2, explain
whether the velocity at 0 would be greater or lesser than the velocity
obtained in part (ill).

C1IITCillt

Axis

Into paper

of rotation

C1IITCillt
out of paper

Figure ..

(i)

(12 marks)

Total 20 marks
(ii)

Copy this diagram into your answer booklet. On your diagram, draw the
forces, which turn the con. These forces make up a couple that causes
rotation of the coil. Will the coil rotate in a clockwise or anticlockwise
direction? Indicate on your diagram the coil position that gives maximum
value of the couple.
How would the speed ofthe motor be affected by an increase in the supply
voltage? Explain.
(8 marks)
l

PHYS - P2 + P3 - JU 2002 - Page 5 of 6

64
~o

65

(b)

Electricity can be generated by passing a conducting liquid

through a very strong magnetic field. The electrical energy is

extracted by suitably placed electrodes as shown in Figure 5.

LOAD

<Jif

CeDductiDI .....d
A, B-Mquetk poIeI pkeel
A-North pole

(ii)

(iii)

(iv)

4(a)
(b)(i)

(ii)

(cXi)

5(aXi)

Describe an experiment to determine the position ofthe centre ofgravity of


a metal sheet ofuniform thickness. Your account should include the
apparatus you would use and a briefaccount ofa suitable procedure.
(ii) State the condition, which the position ofthe centre of gravity of a body
must satisfy in order that the body be stable. (8 marks)
(b)
Two uniform planks each of mass 10 kg and length 2m are arranged as
shown in Figure 6.
D
P

I_I

c, J)..EleetnNIeII
B-Soadl pole

Figure 5

(i)

(ii) When a drop ofa strongly coloured liquid (such as potassium dichromate'--a
bright yellow liquid) is added to a beaker of water, it is found that after
some time the entire contents of the beaker become a uniform colour. Use
(6 marks)
kinetic theory to explain this phenomenon.
Total 20 marks

By considering an element oftluid, XY, in the shape ofa cylindrical rod,


determine the direction ofcurrent flow through the load RS. Give the name
of the rule you use.
In one version ofthis type ofgenerator the conducting liquid is pumped
through the electrodes in a continuous circulating loop, thereby producing a
d. c. output. Explain why this arrangement produces only a d. c. output.
Suggest TWO possible modifications to the operation of this generator,
which would result in an a. c. output. Select any ONE ofyour suggestions
and explain why it would work.
Suppose the liquid was not pumped and the load RS replaced by a battery.
Would the liquid move? Explain. What would be the overall effect ofthis
change?
(12 marks)
Total 20 marks
What are the TWO principal ideas ofthe kinetic theory ofmatter? Describe
(8 marks)
the kinetic theory model of the three states of matter.
Heat must either be added or removed to convert one state of matter to
another. Calculate the amount ofheat energy, which must be added to
2.5 kg ofwater at its boiling point in order to evaporate it completely.
(Specific Latent Heat of vaporization ofwater = 2 260 000 1 kg-l)
It is observed that the specific latent heat ofvaporization is greater than the
specific latent heat offusion for most substances. Use kinetic theory to
explain this in terms ofthe separation ofthe particles in each state.
.
(6 marks)
If the stearn in part (b) (i) were captured in a closed container at 100C and
then heated., the pressure in the container would be observed to increase.
Use kinetic theory to explain this effect.

x
Figure 6
(i) Find the maximum value ofx for which the top plank will remain in
equilibrium.
(ii) A 15 kg mass is placed at P. Draw a diagram showing the forces acting on
the upper plank when it is just about to topple. Use your diagram to
calculate the value ofx for which the upper plank just begins to topple.
(8 marks)
(c)
Two uniform rods both oflength 2 m are arranged at right angles as shown
in Figure 7 , D and E are the mid-points ofrods AC and AD respectively.

2m
D

2m

Figure 7
(i) In your answer booklet, draw a well labelled diagram of the arrangement,
showing the line ofaction ofthe weight of the arrangement relative to A, B,
C, D and E when it is
a) suspended from A and in equilibrium
b) suspended from D and in equilibrium.
(ii) Hence, labelled diagrams ofthe arrangement as shown in Figure 7 and
clearly indicate the position ofthe centre ofgravity ofthe arrangement
(4 marks)
relative to A, B, C, D and E . '
Total 20 marks

PHYS - P2 + P3 - JU 2002 Page 6 of 6

END OF TEST

65

PHYSICS

nght angles
2. (a) A beam ofp-partic1es enters an area with a magnetic field at66
to its path as shown in Figure I. Draw on Figure 1 the path of the' - ..
p-partic1es.
(2 marks)

- JUNE 2001

~er 2

- lYi brs.

Attempt ALL questions.


I. You are to spend no more than yz hour on this question.
(a) (i) A small block ofwood (mass 0.50 kg) slides from rest down an
inclined track with negligible friction. The distance, x, travelled after
certain times, t, was measured and the graph ofx against t plotted. Use
the graph to complete Table 1.

ft-J ~ j

0.50 [ 0.80

1.20

j 140

1.50

Table I
(7 marks)
(n) State which of the timings is the least accurate and explain why.
.................................................................:
(2 marks)

(iii) Use data in your Table 1 to plot a graph off againstx.

(9 marks)

(iv) Find the slope, S, ofyour graph.

.----.

x
x
x
x

x
x
x
x

x
x
x
x

x
x
x
x

x'x

Magnetic field

into paper

x x x x x
x x x x x

Figurel
(b) Another beam ofp-particIes enters the electric field between two charged
metal plates as shown in Figure 2. Draw on Figure 2 the path of the

p-particles between the plates.

(1 mark )

P- particles
II'
Figure 2

......................................................................................... (4 marks)

(v) Given that


slope, fmd a

P- particles

x
x
x
x

S, where a is the acceleration ofthe block down the


.
......................................................................................... (2 marks)
=

(vi) Calculate the force which must be acting on the block to cause the
acceleration
.
......................................................................................... (3 marks)
(vii) If the block reaches the end of the track after 1.6 s, calculate its final
speed
;
.
......................................................................................... (2 marks)

Total 29 marks

(c) Nitrogen (N-17) is a rare, radioactive isotope of nitrogen with atomic


number 7. N-17 emits a beta (p)-particle to form an isotope of oxygen (0).
(i) Write down the number ofprotons, neutrons and electrons in a neutral
atom ofN-17.
Protons
--------Neutrons - - - - - - - - Electrons
(n) Write the nuclear reaction for the emission of the p-partic1e

~... (3 marks)

.......................................................:

(iii) The p-particle is emitted at high speed. Explain the source of the
(1 mark)
energy of the p-particle
\

(iv) Use the information provided below to find the expected energy of the
P-particle.
Phys June 2001 pg 1 Of 6

66

]..\

67

Mass ofN-17 = 17.0Q845 u


Mass of 0-17 = 16.99913 u
Mass of P-particle = 0.00055 u
Unified mass unit (~) = l.fi6 X 1O-21kg
Speed of light = 3.00 x 108 m s-1

(5 marks)
Total 15 marks

3. (a) Draw clear and identifiable circuit symbols for the logic gates in Table 2
below.
LOGIC GATE
SYMBOL
NOR
NOT
NAND
OR

Figure 3
(i) The logic circuit shown in Figure 3 can be used to warn a driver that
he is about to leave his car with the headlights switched ON. Table 5

represents the truth table for the logic ofthe circuit. Compiete Table 5.
Table 2

US
0
0
0
0
1

(b) Complete the truth tables (Table 3 and Table 4) for the two-input logic

gates below.
---

OUTPUT
A
0

B
0

0
I
I

1
0
1

OUTPUT

B
0

A
0

0
I
I

I
I
I

I
0
-

OUTPUT

INPUT

(4 marks)

IS

0
0
I
I
0
0
I
I

DS
0

Z
-~

0
I

--

0
I

1
(6 marks)

NOR
Table 3

AND
Table 4
(4 marks)

(ii) Examine the result in the Z column and write out in words the conditions
under a which the alann will sound
.
...........................................................................................(2 marks)

Total 16 marks
Phys June 2001 pg2

Of'

67

4. (a) (i) State the laws ofreflection.


"

..

".,

0.":

tLLU. Mirror
"

-0""

"

""

68

o.

..

(2 marks)
(ii) An object is placed in front ofa plane mirror as in Figure 4.

/ / / / LLL./ /

L~/

Kingston 1<
Jamaica

2100 km

>1 Port oeSpain


Trinidad

Figure 5
-"\,

(n) Use the information given on the diagram to determine the angle of

incidence ofthe laser light on the mirror.


Figure 4

p'

Object

On Figure 4 draw the irrlage ofws object.

(3 marks)
Total 14 marks
(2 marks)

(iii) Draw two rays of light leaviI1g point A on the object and received by an
observer at points between P and pI .show how these rays appear to

5. (a) Explain the meaning ofthe following terms and state the SI unit in which
EACH is measured.
(i) Constant speed

come from the image.


(3 marks)
(iv) Explain how real and virtual images are fonned. State whether the
image ofthe object in Figure 4 is real or virtual.

"

".""

"'.""",."

'O

"

".'O ."

'"

(2 marks)
(ii) Acceleration

.............................................
(3 marks)

"

(2 marks)

(b) (i) The beam from a laser in Kingston. Jamaica is reflected from a plane

mirror on a satellite. The reflected beam is received in Port of Spain;


Trinidad. Figure 5 illustrates this situation diagrammatically. On the
diagram draw the nonnal from. the milTor to the earth's surface.
(1 mark)

(b) The graph represents the motion of a cruise ship after its engines have

been turned off.

PItJ's Jnne 2001 pg 3 Of 6

68

69
:~

-~

4++--'--;

400

800

1%00

1600

2000

time Is

Graph of Speed vs Time for Cruise Ship


(i) Calculate the deceleration ofthe ship.

(3 marks)
(n) Calculate the distance the ship travels while decelerating to rest.

(3 marks)
7

(iii) If the ship has a mass of2.5 x 10 kg, calculate the magnitude ofthe

force acting on the ship during deceleration.

.................. """

"

"

~.(L

(3 marks)

(iv) What is the nature ofthe force acting on the ship which leads to the
deceleration and how does it cause the ship to slow down ?
(2 marks)

TIm..

Graph of [)istllnt'e Vs lime

(v) What is the MAIN energy change as the ship slows down?
(1 mark)
Total!6 marks

END OF TEST

Phys .JUDe 2001 PI 4 or6

69

2. (a) State Charles. Law. Describe with the aid of a diagram, an 70


experiment to
test the validity of Charles'Law. Show how you would use the results..
obtained to test the law.
(8 marks)

PHYSICS - JUNE 2001


r.aper 3 - 1 hr.

Answer ANY THREE' questions.

ALL WORKING MUST BE SHOWN since marks will be awarded for correct

steps in calculations.

(b) The cylinder shown in Figure 2 is fitted with a freely-sliding piston


4

containing an ideal gas of mass 13 g and volume 1.0 x 10 em3. The


temperature of the gas is 25C and it is initially at atmospheric pressure.

1. (a) Define the tenn 'moment of a force'. Explain what is meant by the 'centre
ofgravity' of an object. Describe how you would find the centre of
gravity of a thin irregular plane sheet ofcardboard.
(8 marks)

(b) Figure 1 shows a table lamp ofmass 1.5 kg.


X

mm

. .,

_ _

~-

. ._

mm

PIston

'-..1

'

Gu
O~36

Figure 2
(i) The gas is heated to a temperature of 400C What is the new volume
ofthe gas?

(n) How much energy must be supplied to the gas to raise its temperature to
400C? Part ofthis energy becomes the kinetic energy ofthe gas
particles. State the effect ofthe remainder ofthe energy.
A

W
~--O.18

-----7

Figure 1
(i) Calculate its weight and the moment of force of the weight about point B.

(iii) If the volume ofthe gas were kept fixed during the heating what would
be the pressure ofthe gas at 400C?
(Specific heat capacity ofthe gas at constant pressure = 0.7 J g -1 e1
Atmospheric pressure = 1.0 x 105 Pa)
(12 marks)

Total 20 marks
(n) A horizontal force, F, is applied along the line XY to make the lamp
topple. Find the minimum value ofF.

3. (a) Describe how you would detennine the density of

(iii) A heavy metal ring is placed on the base AB. Explain how this increases
the stability ofthe lamp.
.
(Acceleration due to gravity = 10 m s -2)
(12 marks)

Total 20 marks

(i) a billiard (pool) ball

(n) an irregular-shaped rock

Use a different method in each case.

PbysJune2001'150r6

(6 marks)

70

71 in air..
4. (a) Describe a simple experiment for measuring the speed of sound

(b) (i) State Archimedes Principle.

Give ONE method of improving the accuracy ofyour measuremeti.ts-:---- -- .


(8 marks)
(ii)

(b) A loudspeaker is driven by an amplified audio signal so that its cone

oscillates at 500 Hz.

10 enf

(i) Detennine the wavelength and period ofthe sound produced.


(ii) If the distance between the points ofmaximum displacement of the

cone is 0.6 em, sketch a well-labelled displacement/time graph for the


motion of the cone. Indicate on your graph the points of maximum and
minimum cone velocity. (Speed of sound:> 350 m s -1 )

50
em

(iii) If the maximum displacement ofthe cone increased, what is the effect

on the
(a) loudness of the sound?
(b) frequency of the sound?

Figure 3

The solid-cylinder shown in Figure 3 has a cross-sectional area 10 em and


floats upright in water with a length of 50 cm submerged. Use Archimedes
principle to determine the weight ofthe cylinder.
(Acceleration due to gravity g = 10 m S-2, density ofwater :> 1000 kg m-3)

(9 marks)

(c) A column of air is trapped by a 5 em column of mercury (Hg) in a narrow


tube as shown in Figure 4 (i) to (iii). One end is open to the atmosphere.
What is the pressure (in em ofmercury) in the gas in each case?

(12 marks)
Total 20 marks

5. (a) (i) Draw a labelled diagram of a dry cell.


(ii) Draw a labelled circuit diagram showing how you would recharge a car
battery .
(8 marks)

(b) (i) When fully charged a car battery can deliver 5 A at 12 V for 16 hours.

before it becomes fully discharged. Calculate, in kWh, the energy


stored when the battery is fully charged.
(ii) Ifthe battery can undergo 100 complete charge-discharge cycles

before it' dies'l fails, calculate in kWh the total energy it can supply
during its 'lifetime'.

Air

Mercury

Mercury

Mercury
Air

Air

(I)

(II)

(iii) Given that the cost ofthe battery is EC $200, calculate the cost per
kWh for the delivery of the energy in part (ii). Explain why your

answer does not represent the total cost of the energy supplied.
(12 marks)
Total 20 marks
END OF TEST

(iii)

Figure 4
(Ao:nospheric pressure 76 em mercury)

(5 marks)
Tota120 marks

PIg1;JaneZOOl pg6Of6

71

CAR.IBBEAN EXAMINATIONS COUNCIL

72

PHYSICS MAY/.J1JNE 2000

SECONDARY EDV~1I0N CERTIFICATE


.
02 - General Proftdeucy
. "EXA~1rt\nON
A student invead,afa the ~ of a spriDl and obiaiDa abe following resuhs of abe variation (g)
Describe- the proeedUllil the
I

1,11."" "

taper

4.0
LoadIN
Length/rom 18.4

student may have UKd to obtain

the 'resulfS.

Stale lhe

prccautioDs the student should have taken to ensum the accuracy of the readings and to

of Ihe len" of a spriq with JRd.


12.0

16.0

26.0

24.0

20.5 22.4

24.3

26.4

28.S

"8.0

28.0
31.3

prevem: damage to the spring.

32.0
36.4

(a)

u. these: values to plot a graph of IcnPlpiDst load on the Jrid opposile~

(b)

Find the. slope. S. of the paph.

.. _.....

.~.

(10 marks)

.
( 4marks)

Given thlll the spriGg: cousunt

(e)
'<.,

....

( 5 marks)

",!. find the spring cOJlSlaDt.

TotalJO maries
1.

. ..

~. '

This question is com:emed with cbe properties r;>f three different kinds of waves-'
_ _ _ _ _ _.,....--

. (a)

~ terms wIleD used in connection with a sound Wave.

( 2. mub)
(d)

Wavespecd---------------------

U. your gRph roddcnnillC the spring's length when Ihe load is zero.

;0;

( 1 marks)
(e)

Speed. waveleng1hand frequency arepropcrtiesofwaves. Explain thentelllling o(';ach of

Wa\IClength - - - - - - - - - - - - - - - - : . . . - - -_ _

What is the value of the sprinJ's X:IENSION when Ihe IOlId auacbcd is 14 N?

Frequency

---..:-.~------:-
( 4.l'DlU"ks)

( :% mall'b)

(f)

What mass must be placed on tho spring to pn:llkK:e an extension of 9 rom?

(b)

(i)

Flgue 1 repRlSClntaa numberofadjaccnt wavefront! ofa walerwave in a ripple"


tank.

F"lgUte 1 '

( 5 martu)
Use FIgure 1 [0 dctennine the wavelength of the water wave - - - - -
72 ( 2 marks) .
PHYS - P2 + P3 - JU 2000 - Page 1 of 6
":i...U

73

Figure 2 is the displacement vs time graph of a smalliaf floating in the palh ofthe
water wave in (0.

(ii)

(ii)

'1'11efront of the pUlse anives at the position marked. X. at time t =0.1 s. Sketch
on the axes below, a displacement VI. time graph for. small element of rope at X.

Dllpl_ment/m
0.05

TImt/a
0.05<

0.1

0.15

0.2

tis

Figure %
<

0.05

Determine the frequency of the wave by using Flcure 2.

( 5marks)

Total 17 marks

( 2 marlu)
(iii)

Calculau: the speed of the warer wave.

3.

In the Kaieteur Fails in Guyana. which are 230 m high,; the temperature of the water at the bottom
of the falls is consistently slightly higher than the temperature at the t o p . '
(i)

What kind of mechll.!lical energy does the warcr possess


at the top ofthe f . l I s 1 - - - - - - - - - - - - - - - - -
just before it hits the bollomof the f . 1 I 5 1 - - - - - - - - - - - -
( 2marks)

( J marks)

<

(c)

(i)

Figure J shows a wave pulse at time t "" 0, moving from left to right on a suing.

(ii)

Use the princlple ofconservatlon ofenergy to find the speed oftbe water as it hils the bottom
of the falls.

x
I

V .. 2m1s

~I

-o.zm---'
( 3 marks)
(iii)

Figure J

Assuming all theenergy ofthe water goes to raise its temperature,calculate the temperature

rise.
What is the amplitude of this w a v e ? - - - - - - - - - - - - - - -
(lmark)

PHYS-P2 + P3-JU 2000-Page2 of6

73

( 3 marks)

...

(iv)

Suggest why

t.tic lIl:lUal tanperawn:- rise is f~ to be lower I:hao. Ibal: calculated in (iii).

(b)

(i)

74

Fill the bllIlII" in the following table adding either the symbol or name of the logic
gate.

NAME

SYMBOL

j)

(lmark)

The mass of water that moves over the falls per second is 200 kg. What is the rate of
conversion of potential energy?

(y)

D
NOT

( 2marks)
(vi)

State lWO possible advantages of using the energy of the falls to drive a power station
.
rather than using an oil-tired power station.

( 3marks)
(ii)

Complete the truth tables (or the following logic gates:


NOR

AND

INPUTS
B
A

( 2marks)
(acceleration due to gravity

= lO m $-1; specific heat capacity of water =4200 J kg-l IC""I.

0
I
1

I
0
I

INPUTS
B
0
0
0
I
I
0
I
1

OlITPUT
Z

OUTPUT
Z
0

( 4 marks)

Total 13 marks
(iii)

4.

(a)

(i)

Complete the truth table for the combination of two logic gates shown below.

In the space below draw a graph of voltage against time to represent an alternating
voltage of amplitude 5 V and frequency 40 Hz; Label the axes.
Input:

INPUTS
B
A

f)
Y

r>--

Output

OUTPUT
Z

( 2 marks)
. (ii)

Write the name and symbol ofan electrical component that could be used (or half
wave rectitication of the alternating current above.
( 4 marks)

Total IS marks
( 2mark.l)

PHYS - P2 + P3 - JU 2000 - Page 3 of 6

74
'l-S

75

s.

Figure 4 shows a solar waler hearer being UKd to heat a domestic water supply.

(ii)

Sugpst a material that might be used at A. and explain why it would be suitable. '

( :2 marks)
(e)

, The radiation from the sun arrives at a rate of 900 W m- 1. A solar heater has a collection
area of0.4 m2 imd converts the sun's radiation into thermal energy in the hot-water system
with an efficiency of 15%. Calculate the rate ofsupply of energy to the hot-water system.

Cold

Pump

Cold_au

Fipre4
(a)

Identify the processes ofbeat transfer taking place between the following and explain bow
the construction of the healer makes these processcs more efficient:
(i)

TIle sun and the upper surface of the solOlC collector

( 5 marks)
Total IS marks
"

( Z marks)
(ii)

The upper surface to the water in the pipes of the collector

( 2marks)
(iii)

END OF TEST

The collector to the storage (ank

( 2marks)
'(b)

(i)

Give TWO reasons why there is a transparent cover over the collector.,

( 2 marks)

PHYS - P2 + P3 - JU 2000 - Page 4 of 6

75

76
(b)

In a television tube a beam of electrons is acczleralechhrough :l potenlial differ.enceoL


SOOOV.

PUYSICS

(i)

Find the energy gained by a single electron


difference.

Paper 03 - General Proficiency

If the current in the electron beam is Z.O rnA. calculate the number of electrons
pusing a point in the tube in one second.

(iii)

( 09 JUNE 2000 (a.m,) )

it moves through this potential

Calculate its final speed.

(li)

/ hour

llS

. (iv)

,Electrons travel down the tube and strike OJ screen at the filrend causing the screen
to give out light. Hnd the energy converted per second as the electrons strike the
screen.

(Charge on electron'" 1.6 x to-'9 C. mass of electron'" 9.1 x 10- 31 kg.)

In addition to the 1 hour. candidates are allowed a


reading time or 10 minules. Candidates must NOT
....rite in their answer booklets during this time.
1.

3.

(a)

Describe how you would determine the latent heat of fusion of ice. Your account should
include the method and outline lhe calculations which you would mak.e.
(8 marks)

(0)

A glass contains 0.25 kg of waler at oOe. 0.15 kg of ice. also at 0 e. is added to the
water and it is noticed that after II minutes all the ice has melted and all the water is now
at 15C. Assuming thaI the thermal capacity of the glass may be ignored. calculate the
(i)

(ii)
(iii)

la)

. (12

Total 20 marks
&plain with the aid of a diagram. how the human eye forms an image.

(i)

State the characteristics of the image formed and compare the properties of this
( 8 marks)
image with that formed by a diverging lens.

(li)

(b)

----..--.....
-- -

thermal energy absorbed by the ice for it to mell

....-

total thermal energy absorbed by the contents of the glass

--

"

.........

- - water

cz..

average rate of energy absorption by the contents of the glass.

(The specific latent heat of ice is 3.4 X 10' J kg-I and lhe specific heat capacity of water
is 4200 J kg" K-')
( 9 maw)
(c)

marks)

Air

An electrical heater. rated at 7.50 W. is used to raise the temperature of the w::lter to boiling
point. Calculate how long this would take if no heat is lost to the surroundings.
( 3 marlui)

Figure 1

Figure 1 shows a thick-bouomed. glass aqulUium containing water. A beam of light from
a submerged source. S. is incidetlt on the base at point X. The angle of incidence at X is

Total 20 maw

58". If the refractive index for light going from waleC to glass is 1.13. calculate the

2.

(a)

(i)

(ii)

The terms electric field. electrif; charge and potential difference are frequently
encountered in the study ofeleclroslatics. Define the unit in which EACH of these
quantities is usually expressed.

(i)
(ii)

Dislinguish between electric field and electric field strength.

(;ntic;d

angle for light striking the glass to air boundary.

(Refl11Ctive index for air to glass '" 1..50.)

(iii)

Draw a diagram to represent the electric field between two oppositely charged
parallel plates.

(iv)

Write down an equation relating,

me eleCU1c flCkI,. UII'; J!Otc:ntial diff~"mce between

the two plates. and the distance between tbcm.

angle of refraction in the glass.

( 8 marks)

(C)

( 6 marks)

Use the angles you have calf;ulated to draw a diagram showing the path of the re<tmted
beam-after it leaves X. Partia! reflection also occurs at X. Draw the reflected rayon your

diagram.

PHYS _ P2 + P3 _ JU 2000 _ Page S of 6

6 marks)

Total
7620 marks

77
4.

(a)

(i)
(ii)

Describe how you would find the centre of gravity of a piece of cardboard of
irregular shape.

(b)

(b)

Define the moment of a fon:e and state the principle of moments.

(11DIII'b)

Figure 2 represents a tray held horizontally by a waiter. The tray has a weight of6 N and
supports a drink of weight 3 N. The waiter provides a force at P. The tray may pivot about
pointT.

p.

..-

.~.....

.
i
~
-

..

3N

6N

5_ 30

em

-~

\ ......

...
+

Figure 2
Usc the information in Figure 2 to find the force provided by the waiter at P and
the upward force the hand exerts at T.
.

(ii)

The drink is moved along the tray c1osertothe hand. Is the fon::e needed at Ptokeep
the tray horizontal now less. greater or the same? Explain your answer.

(iii)

How much work is done by the waiter in lifting the tray and drink through
a height of 0.60 m1
(11 marks)

........

I
I
/

/
/

(a)

Figure 4

suw .... tube

C1mI1C ..

(i)

Total 20 marks

s.

\
J

-~

.'

A copper rod. connected to a battery. is suspended in the cencm of the magnetic field of a
solenoid in Figure 4~ The copper rod experiences a force.

SolenoId

Cln:alt.

I
!

f'.
t---------l '. S-.,

(i)

Explain thc origin of this Corce and determine itsdilection,staUngany rule you usc.

(il)

Would the magnitude anddircction oflhe force change iflfie cum:ntin the coil were
reversed? Explain your answer.

(iii)

The eunent in the solenoid Is pually increased. Explain the effect of this on the
(12 marks)
tension in the suspending wires.

Iron...

Total 20 marks

Figure 3
(i)

Figure 3 shows two circuitS" A and B. When the switch. S. is dosed the lamp. L.
lights. Explain how this occurs.

(ii)

If the battery in circuit B were to be replaced by.an a.c. supply. would the lamp still
light? Explain your answer.
( 8 marks)

END OF TEST

PHYS-P2 + P3-JU 2000-Page 60f6

77

PHYSICS PAPER 2 - JUNE 1999

78

General Proficiency - 1Yz Hrs

Candidates ar~ advised NOT to slJend mo~ than 30 minutes on this


1.

,mass of !fa,s ONLY at this maximum pressure 12 X 1O~ Pa.

qu~sti()n.

The data in the table below show bow the pressure of compressed gas in a cylinder varies with tht:
total mass of the cylinder of gas.
Mass of cylinder of gas (llll/k!r

9.9

10.2 I 10.8

12.3
(4mark.<;)

Pressure, P. incylinller X 1O~ Pa I 1.5

2.i

5.0

10.9

( 8marks)

At a certain time. the pressure of the gas is 7.8 X1O~ Pa and its temperature is 24"C. What
wUlbe the new pressure if tbe temperature of the cylinder and gas rises to 32"C? (Assume
the cylinder does not expand.)

( 3mark.~)

( 4marks)

(f\

(a)

Plot a graph of P (v-axis I against til (x-axis 1.

(b)

Use your graph to find the mass of the empty cylindtlr.

(c)

(g)

Find the gradient of the graph.

.
(d)

In tenns of the kinetic theory of gases, explain what causes the pressure of a gas and why
the pressure of a gas rises when its temperature rises at a constant volume.

( 4mark.~)
,

When the pressurereaches 12 X lOS Pa. the cyliDder is considered full, Wouldit be possibk
to add more gas at this pressure? Why might this not be advisable?
( 5marks)
Total 30 marks
( 2marksj

(e)

Use your graph to find the


total mass of the cylinder and gas

PHYS-P2 + P3-JU 1999-Page 1 or7

lIo.c

78

?-1

79

2.

(a)

Explain what is meant by the mometll of a force about a point.

'e)

Calculate the magnitude oflhe force. F, which the knife exerts on the lid. (Ignore the mass

ohlle knife.)

( 2marks)

12.c:m

if

Handle

( 3 marks)

Knife

.:.lc.".

Ii'

(f)

Lid

The can exerts an upward force at point, P, on the knife. What is the value of this forCe just
before the lid moves?

;
j
j

CD

30N

i
I

( 2marks)
Figure 1

(g)
Figure 1 above represents a cross section of a table kDife being used to prise off the lid of
a can.
(b)

lfthe force, F, is too small to lift the lid, suggest TWO changes you might make to increase .
the value of F.

The kDife when used in this manner acts as a simple machine. State the name of the simple
machine.

(1mark) .
(c)

On FIgUre 1, (i) indicate the position of the pivot, P, and (ii) draw an arrow to show the '
direction of the force, F, which the knife exertS on the lid.
( 2marks)

(d)

Calculate the moment of the 30 N force about P.

( 2marks)
Total 1$ marks

PHYS-P2+ P3-JU 1999-Page2 of7

79

"i

80

3.

(a)

A transverse. wave on the surface of a liquid ha.' a wavelength of 1.8 em. Explain what is
meant by a transverse wave and the wavelength of the wave.

Figures 3a and 3b below show the same wave striking a plane refle(;ting surface
and incident on a small gap in a pl~. reflecting surface respectively. Complete
the dia!,'TaIDS to show tbe wave (i) afterreflectioD and (n) after passing through tbe
gap.

(ll)

(2marks)

Directioa

(b)

F"IgUre 2 below isa displacement-time graph representing the wave described in part (a)

I Or travel

Direc:tiOD I

of'trllvel

above.

..

Displacement
O~9clD ..... -

Figun3a'

time/I

0JlC:1D~-----

F"IgUre3b

--

( Smarks)

Figure 4 below shows the same wave approaching the gap which has been
increased to five times its original size. Complete F~ 4 to show the wave after
it has passed through the gap.

Figure 2
(i)

I
I

For this wave d.etennine ithe


amplitude - - - - - - - - - - - - - - - - -

frequency - - - - - - - - - - - - - - - - - - -

speed------------------

Directiou

of' lravel

I ..

( 6 marks)

~
Figure 4
( 2marks)

(tcn

L{

I~ ,O'_"'~'-v-:; I

PHYS - P2 + P3 - JU 1999 - Page 3 of7

80

'1'111

II

I: it:

81

:111 il I
,f,

4.

!<I.J

What i~ an elrwrriclidJ';

it)

. Iii i I

How J[\1atly ekctrons

mal~e

up the ,:.:harge ill pan (b) (ii l on page IO'?

------------_.----_.--~

-----------_.

I:

,I

( 1 mark I

How is the dir",ctioll of' an electric field dt~fillletl?

(ii)

1111 illl

ek.t;tron

1.6 X JO-1"Cl

( 3 marks)
]V

fb)

A velocity selector is u:;ed tu separate charged partic'le, and consists of two parillld mewl

plates connected
to a source of e.m.f., 3:; sho"in in I"igure 5 hd6w.

1111 1;i1111,1 II IIi !II 11111 III 111!I!!iillllll I! I iii II I ill III ililllll!llllllillllllll Ililli illllllllIl"illllll I' lilllllllllllllllill illll,J 'ii
111,1

......1'

Me~

Plates

,l
I

-1

Calculate the work dont: o\-th" hattery in chargmg the platt'S.


1111111111

111ll1l1i

ill II

I!I 11111111

III

I III 111111 I!I!IIIIIIIII

111111111'1: Ii 11111 III II I 111111 lIilillllllllll i Ililllllll I

1IIImn"

( 3marks)

-+

t\.',

Olle' of the: plart:, becomes positi\'c'ly charged. Explain how this occurs.

T_

~.

I' I.: I

.L;,

I.. '

t I mark}

Total 15 marks

Figu.J'e 5
i

When the plates are connected, a current of 1.5 A flows for 0,5 x 10- 6 S and the plates

become charged.
On Figure 5. mark the, sign of tbe charge on EACH plate. Draw on the diagram the

electric fieidthll.ttesults from tbe chllrgedplal'es. .


I ,
( 3marks) ,

(il)

Calculate the quantity of charge that flows on to the negatively-cbarged plate.

( 3rnarks)

PHYS - P2 + P3 - JU 1999

4of7

81

...

7'1

82

PHYSICS PAPER 3 - JUNE 1999


General proficiency - 1Hr
1.

(a)

Describe an experiment to determine the specific latent heat of vaporizatiou of water.


( 8marks)

(b)

The body loses thennal energy in several ways. One is by wanning the air taken into tbe
lungs and then breathing it out. A second way is by evaporation of water into air which is
then exhaled from the lungs.
(i)

In one breatb.a girl inhales a volume of 4 X 10-4 m> of air at 15"C. TIus is wamll:d
to 37C in the lungs. What mass of air is inhaled and how much thermal energy is
transferred to the air?
( 6 marks)

(n)

A mass of 1.8 X 10- kg of water vapour is exhaled along with the air for each
breath. Calculate the thermal energy required for the water to evaporate.
( 3 marks)

(al

With the aid ofray diagrams explain the terms 'refractive index' and 'critical angle'. Draw
a ray diagram to sbow bow a right-angled isosceles glass prism can be used to turn a ray of
light through 180, and identify an optical instrument that utilises this property.
( 8marks)

. (b)

A ray ofligbt travelling in water at a speed of 2.25 x 10 m S-I strikes the water'
surface as in J.'igure 1 below. Determine the refractive index ofwater and calculate.
the maximum angle the beam clpl make with the normal if the light is to enter thl!.
air.
(6marks)

iNormal
4

I
I
I

Air

(ill)

In one minute the girl takes 12 breaths.. What is the average total energy toss per
( 3 marks)
minute as a result of breatbiJJg?

Water

(Density of air: 1.2 kg m- 3; Specific heat capacity of the air: 1.0 X 103 J kg-IICI ;
Specific latent heat of evaporation of water at 37C: 2.5 X 106 J kg-I)
Total 20 marks

Point source

Figurel
2.

(a)

(i)

(n)

Define linear momentum and state the law of conservation of linear momentum.
( 3 marks)

(n)

A car ofmass 800 kg collides head-on with a truck ofmass 3200 kg and both come
a stop..The truck was travelling at a speed of 30 km h-I when it collided.

to

A student argues that momentum is not conserved as both vehicles are stationary
after the collision but both were moving before the collision. State whether the
student is correct or inalrrect and explain your answer. Find the speed of the truck
in metres per second and determine the speed of the car as it collided.
( 8marks)
(b)

(i)

(li)

The light source is replaced by a detector of light. and a ray of light is then shon~
on the surface of the water from the air. Is there any angle of incidence at which
.
no light enters the water? Explain.

(SpeedofIight in air
(c)

= 3.00 x

(3marL~)

1O&ms-1 )

FlI'JI'e 2 shows three rays entering a specially shaped piece of glass called a prismatic
window. The angle ofincidence of the two outelrays on the surfaces AB and CD is greater
than the critical angle for glass. Copy and complete the diagram to show the passage of the
rays through the window.

A spacecraft with several small rockets attached is moving at constant speed, in


a straight line, in outer space. Explain how the spacecraft is able to maintain a
constant speed for many mon~ without turning on the rockets. State and explain
the effect on the motion of the spacecraft if the rockets are fIred for a short time.
( Smarks)

The spacecraft bas a mass of 1.2 X 104 kg and the rockets provide a force
of 4.0 X 103 N for 3 s. Find the pin in velocity as a result of the firing of
the rockets.
( 4marks)
(~eW ;a:; ~)

J..C

i.,L:V.~)

PHYS - P2 + P3 - JU 1999 - Page 6 of 7

82
';IV

83

4.

(a)

Describe, with the aid ofa diagram. an experiment thaI demonst.raJ:es that
(i)

an induced e.JIi.f. is caused when the maguetic flux cutting a wire changes

(li)

the size of the e.m.f. depends on the rate of change of cuning oftlle magnetic flux.
( 8marks)

(a)

Sketch voltage-time graphs showing how the rectified a.c. voltage differs from that
produced by a battery.
( 8 marks)
(b)

(b)

Draw a circuit diagram of a half-wave rectifier coonected (0 a resistor aud an a.c. supply.'
Indicate on your diagram the direction of ctuTent flow.

(i)

An electric guitar has six steel strings and a 'pick-Up' made of bar magnets with a coil ofl
copper wire around them. Figure 3a and 3b. When a player plucks the strings they vibrate.

An air conditioner consists of a fan-coil unit and a com:p;essor unit. When the air
conditioner is switched on. the fan-coil unit. rated at 250 W, operates continuously. .
while the compressor. rated at l00e W, operares only 40% of the time.

The air conditioner is switchedoti for 12 hours per day. Find the total electrical
energy, in kwh, consumed in a 3O-day month.
( 9marks)

Steel strings
(li)

If the air conditioner is operated in a Caribbean country where electricity is billed


( 3marks)
at $5.00 per kwh, find the cost of this energy.

Total 20 ma.rks

Figure3a

END OF TEST

CoIl of

ropper wire

li!-fl-/-I-l-----}-
Figure3b
The purpose of the pick-up is to convert the vibration of the strings to an elel;tric current
which the copper wire then conducts from the guitar to an ampli1:ier. Explain how the
( 5 marks)
Vibration of the steel strings causes this electric current
(c)

Another example of the application of electromagnetic induction is the transformer. An


ideal transformer is used to operate a 12 V lamp using a 240 V mains supply. There are
1800 tumll on the primary coiL Calculate the number of tumll on the secondary coil. What
is the current being drawn from the mains when the lamp has a current of 2.0 A flowing

PHYS-P2 + PJ-JU 1999-Page 7 of7'

83

:"

TIle gr.lph on page 2 was PIOUCl.l uu............._ .. w.w_

PHYSICS PAPER 2 - JUNE 1998


General Proficiency - 1~ fIrs

How woullJ you make:. liure Ih:U the remperlltUllr ot tbe g;1S'WUGOJlli1lIm84
durinlplle

{;I}

cxpcrlmenl. '1

+..

Dc7a1r

:1[:

.._ _

_
u

_._

_
n

k. x

where Us II con:uanL

To leSt whClber Ibe rela1ionship appli&:s in Lhis case. read on v<l1ul:S orv from tM graph
10 compk:~ Table 1 below. Ca.lcul:lle the corresponding .v<llues of W.

PreSllUePllle

.J~C_ctlcHt'OI'

_ _

_."

ltissu33CS1Cd that if the temperaure is eonstlUlt.tbc pressure.P. of" fixed masso( (l:lS
~ invClSCly ptoportio/lll.llo UlC volume.V.l.bat is

(b)

Glaa'lube

1+-11

__

__.._------_

( 1 mark.)

1.
I''; fol.

You should NOT spend more than 30 minutes on this question.

'1);.! ~I

- -..u

Volume. went

PICliSUlC. p/kPa

,-,

<VWcm-)

100
011

I~O

. 200
FJ&urel
230

TheapparatusshownUiFIgUIIl 1abovemaybeu.scdlDinvtatiptehowtllevolumeoCaoxedmass
ofgas 21: COBStlIDt lemper.ltlUe va.rie& when tbe pmsu:e II ebaIIpd. AIr II trapped In a uniform
g1llss tube which has a scale behind It calJbraIcd ill em'. Aa lbo paIUll:: iIllho oil dwnbcr is
inCJeasedusing tile puDip.. ol1l.sCon::ediDto theaIass tube.~ tllevOWlIIlIUJlLbl ale. The
pressure. In kl'a.1s ind~<:ilCd by a pup filled to LbI oU cllanibet.

. T"'l"
,,-,l J!,..., ',.
- .....-,t
" t

"~l

..p.,
+! r
t1 ..t~'!~~I-;'I.-i .
iF'"

'n'

m.

'~'.~

.,~.

:ql'
Ibt'1" ,
, , If,
I'll

40

oW'

;.. . .+

=t l F .

l i t ;'"

: ~H 'ndFH

-1+"

'!d~ f

;'.

300
Tablei

..
'-I
1 1-,
I.

..;..~

tt!
'" r !~

( 911W'ksl
( 8 mllncs)

On page 5. plat a iI'lPh of W againstP.

(c)

1""1""t";''' .~"Jl

'

+!-,-+""

for

Whuconc:llI$.ioncanyou draw,(romthe graph you h:lve plotted? Give .. reason for your

.(d)

lIIlSWet.

!:l~;
.ttl
f l,i"

'"

~,j:
:

11"~;'r

l~ . 'lt~j~l.tt
t. J . . . .

If :j :;
;:
~ '!'
, .. .,..
45!il':"'1

.r..tll; !.;.
..f.;:.i: I:: : :

"

t-

110

I l~.. ""

.' ++i4

.... I~~ ...

...
1
:.

351' ~ , ~ I:: ::Ir 11 ~': %r::nr!'

_
"._ _._ _ __ _._
.................................................._---_
_

f :

~"

_.

..._

__ _ _

" u._._ _.,.


_.._
.
( Z marks),

.'.1

~ '.

.1 . ;

.101 ; ; ::

(e)

.The padlen!. oC lbe papIIls equal 10 the value or i'.. Find k.

'" ~ l'
.. ' j !

151:

1',';

:I,

11)1':: :

50

15

.__

:~I! ;~~Ii.i'\

I
Hr

i: r: ;:1;

%.50 175

300

315

J50

"'
-

~ureIkP.

PHYS - P2 + P3 - JU 1998 - Page 1 0(7

_
-

.._.._

_ -_

. -- _

---:- _

_--.-.....- -

__

__

_._- - __._._

..}.._ . _ - _.. _

_._ _

_ .._._:...-....

__

_.11'

_._ _w

~.

...

_.__

( 5 marks)

84
'y

85
.:

WblU would betl1evolunle ofthc alrilltlietube ICtllie~could be1ni:Rla:SCd


to SSO kPa7 (AssuIl1ll the eqUlltioll ill (b) on page 3 appt1es.)

(i)

(0

1.

__

(3)

_-_.._.__ _ __. _._---_. _-_ _. _--


-------,. _--._--_. _. _ __..

_ _.

__

__
_ _.....-----_._---
_ __-_
__._-_._.__ _---._.__

...

....

.......--

__ .

---

..... ...-...

__

~ \VlItchgl:l$StS

cin'. Caleulate the prtSS\Il'e of .

_--_._ _--_._----_.._--_..----._._._-
_-..--- _----_._-----_--.. ....
-_. ---_._-- ...-.._....... - - "

... __........

Figu~1

".

'

Figure Z llbove shows a palytltcllC rod. n=::ung on twO w:ltch glanel ....hlch :let 1IS:t low-l'rietion
piVOt. A ncg3tively ch:lrgc:d rod is brought neM to the polythcne rod.

..--...

'!em..,

..............--......~.-.- ..--.-.........---._...
. ....,1..---....__--

( 2 marks)

(i)

If the potj'thcnc rod is 3ttr:u::ted. what TWO pOSSible cOllClusions migbt be


drawn?

ToClIIJO IUrb
d:,

. . . . . . ._

_
; I

,,'

n;; :i

1M'" !
I"'~

..

. , "

:;

i:-; ; iT'
rr ,i:~ q 11 : q! t t f ~

i; "7, lli~~ . ; I

t ; ~ I' i;;' it 1-

f .. ~, t '
t ''' f .
0,:. f r~ t:~f ~;~~ j,:j!
I ! t ; ~ t ; . ~: ~:;~. 1:-!
t

; ,-;.

l'

":'

..

; ..

fl',;.

,I.

,'.

,It

,.

".':

to

tr'l .

t"!

,.

!
J

~, 'It:. l"~'

'f

f'

lit
tr'

'

.t

(il)

.1

'

in

;:::

'1"

i,

.t

..

,0'

':'"

I}

.:.

~!';:,
'f" , . . :.;.
:.,

,,

:;l.
,.,: ",'
:j:l
.,.,

;:!'
':~: ;':'
'.! . , n t

i,j~: :::: :::: ::.! i:~: :;11 :j~! i~l! j~i!

Hot

.t

!~ : : ~ ~ ;:
t ~.,
...

"

. '.

; .. , I : "

ri.: It:!

1.1~

".! it'i IIp "11

::;:

-..

,.

;~:' ~i f

i:t:

_..

.. ..

( Z marks)

._

_-:.._

-.-

".--~.

( I nmrk)

AneptlYelyc.hargedSPbereisCOllflCetCdbyacopperwlrcloapositivelYcllargcdsphc;rc.
causing a cummt to flow. as sJlowrt in Figure 3 below.

<: : :::' ~
,i,

." t '

-f ,.
I I ~

,I"
,f.
1'1'
. " ........ ,,'
!i1.i
i1~' i~:; l~:;
I,:,

- ..-

- _. _

__ __ ."_ _

"

II

'",

.'"

C""
I.
I'

",'
',"
",1

;i!: ~:t: :t:~ i~it

1.

"f

..

Figure 3

t,:

;:;
:
;+

"~

_._._ _

i!ii
;~i~

. ':,;; ,;:i ~::: ::;,,':: ::.: ;,': :::; 'W !!i! :!:: !::: ::!! :;:; :,:: ChI

.
'! " . ,

,,_

'"

f.;'

HH'i": :;:1 il:i:;; ;i; ,:i:,:;!! :!ji,;!!Li!H


t

(b)

t"

, i . ..

__

...........--=

i ,

':":.1 :l:-rr:;
, : ; ;f
.,
oOt ... , ....
. ... ''';\~i

.. -

tt,1:

'

__

.._

"'_

If

.',ri fff . ;~ ~fI

.'

..-. __

WhlU. cOllCtusion could be drawn if the rods repelled each other?

_..

"

:;,;11:; ;~~ ; ; :;~ ::~ ::.~


.::; ; H ~m
iiI i'i!
;: .. :;:11:;.,...
o.ot I . : I,.. , I.. ; . . ' . "
: ' " "r' " , , HI; , ,
d I' !r..
; : i l, :,. ;
.. 1
, ...
r .. 1.... :
~;.;
~,-,
I '1''
'1
I f ; .~
I::
:
:';'
.... .,,; : ~!4 !:i;
; .: ~:: . ; :: ':t
~::~ , ~t i:t: :JJ ;!:~ I l l ; ! l::: Hd
.. :
~d
if
t' ~ ; ,
,"
;: i
;:;i :;ii ;:: f:;' .:.: ::: ;:::
. ;: ;;I; Ii!;: F
f: ..... ,.i :, ..
&.03 ,
1", II:! /"
1';' .. " ,jl . / p ' li'i'
,.
j' I;::.
-,. t

~._

: ...

:;;: ; ;:;
,

,;

__

,~

.._

-;-;j-;- , ,. , I ' \ , : , : ' I; "\ j I


i! 4 1 ~ .. :
1; ~:!; : ~ ;: : j ~ I ;:; ~

l~; "tt
tlil~tf~ t r
ft. It'" s" . !:

: ; ! ~ :r ~
~ l J ! i l' I 'Ii ! ; ~ l l ~ :: I' I !.,. ~ f ;; 1;;.; :-,
H.:;~
ti,. t ~l . :.,
t~tl IiI dt j;.~ 1-"!1 ~",'q t t, r
~ r ! [:: r t! :';"1 t t: ~ : '., ...
,~;';
f: J ~ f r t: ~ i
t;;!
~ :--j': rr~:
~~~
.. .., ..... I f " ,. f ... I t 4
,
... r i
't.l
. t , . , .. t 1.. t t" r .. t j J
OOJ .. i: /.! .r Hf . ' ' .. I.' II': .:!.!flt
, ....
+1." .,!. i i ... ','
, .... 1. : ': ,;,
;". .' ,
t i i ' I i ;. !., I.'
!!', \' 'I . " ! , ' . ' j' r
;~t~ :;!~ .i! ~:l
, .. , !:~, ;l; =:.; :": :. ~ d . ,!;: ~~f: I;J! !~,: ~:I: ~
.i

Pr:ilrlhtllt rod

~---

( 3 IIItIrks)

The mulmum volllll1ll ofthe air inlbo tube Is 6S


the air for this :"olvme.

(il)

. _._...

-~

....

"I"WTYVC"

ft1 _I 'ft"'2

nT

'IftACJ

'ft__. . . .

_I""

85

86

b)

(i)

J.

Mark. on Figure 3 above, lhe direction of conventional current flow.

(a)

( tmark)

,What kind o(cbarge aelually moves in the wire?

(il)

.""""."~_ _ . " " . " . " " " " , , , , , , , ,. . ~n_..

n_"

."""""'"'''''''''' '''''''''

Type of Wave

__

longitudinal1
Infra-red l'lIdiation

In which direction do the charges move? ,

PrOIlL............................. 10
(iv)

Sound waves

Loud-speaker

( 1 mark)

TV transmitter

Name the S1 unit of clecl.ric charge and give its defmition.

__

.........."

",

--

.................................... _..

Table 2

( 5 marks)

Tnmsver.sc or

Source

..

( 1 mark)
(iii)

Table 2 below lists Iypesof waves and thcirsources:.Complete the table by filling in the
blank spaces. In Ihe thirdcolumn. state whether the waves aretr.lJlSYClSC or longitudinal

(b)

_.-

.-

Figure 4 below shows a wave SjlIl:ading out Crom the point S. The wave hilS the barrier
and is lI':f1cctcd. On the diagram. draw TWO zeOected wavefronlS and mark. with an X
the point from which !hey appear 10 come.
( 3 marks)

( 1 marks)

Barritt'

If the curreRI in Ihe wire is 3 tLA and it flows for 2 ms. how much charge is
lmIISCem:d through Lbe wire?

(v)

-..__._---

__ _ _ _- ..

.......... _

.......... _

__

.................................................

__...-_-

--_ .

"

io

..

__ _-

_ __

-......

( 4 marks)

(c)

The cbarge stored In aCar balIery is lISUlI1Iy stated using the non-SI unit8ITIp-OOllIS. One
anip-hour is derll1Cd as the dwge srored In a battery when a Cllmr. of one ampere Is
a:sed 10 cbarge it for a period one hour.

or

_-_

__ __ --

(c)

Calculate the cfIarie Sl:ored in a 40 8ITIp-hour battery using the SI unit Cor tharp.

....... ,;;..
_

.. ..

__.

~~~

...............................................

............__

__._

_-..

- _
_ _

--

_-_
" _

_._

_..
_

_ _ _ _-

_-_

_ _

__

Fipre4

Figure5belowisapaphwhichreprescnadlovariadolloftbedisplacement.y, withl1me,
10 as a wave pIS'ICS a ccrtaia point..

..

1/mm

--.. --ri;;;;;.ki),

Total IS marks

-151' '1\ 'II"" 1\' !I"

1\ . 'II

.$
.7.5~

PHYS - P2 + P3 - JU 1998 - Page 3, of7

1.1

0.1

IU

l::
U

tis

YllUre5

...

GA

,W

86
-;7-

87

~)

Use the graph to find the amplilude of the wave.

(i)

( 1 mark)

(c)

The radioactive isotope of iodinc,l;~ 1. is used to study blood circulation. This isolope
has a half-tlfe or 8 days.

How lolli does one complete C)'C1e of the wave take?

(Ii)

Explain whal, is mc:uu. by the HALF-LIFE oCthe isotope.

(i)

(lmark)

CaIcu1ale thc frequency of I.be wave.

(iii)

(Imark)

iii)

Give the symbol for another possible isolOpe of iodine,

( 2 marks)
( Z marks)

What other information would be needed to cakulll1e the wavelcnfPb of the


wave?

(iv)

......... _

h _ _

Other isotopes arc available wilh half-lives ono seconds and 2 years. Explain
why 8 days is a suitable halC-Ufe for medica.l uses

(iii)

..

( IlDlU"k)

On Fipre 5 above, draw aoothcr wavc which has thc SAME amplitude bul
HAu: thc frequency.
( 1 marks)

(v)

Tolal15 maru
4.

(a)

( 1 maru)

Radioacuvcll'lICCrsarcsomctimes used in mcdicinc. Describe how aradioactivcisolope


is used as II tr:lCeI'in the human body.

._

~_

_.-- _._

A sample of I:~ I is introduced iDlO a person's body_

..."
...

Nuclei whichernit only 0: - particles are NOT nonnally used as rndioactive tr.ICC1'S in
the human body. Give TWO rca50ns why this is so.

I (d)

"

-_

_-

.._ _

-.....

""

_
_..--_

- _

__

" .."

_ _ _. _

_
_

-.._--_

"_

,,

u,

u_

.
( 3maru)

I:

.............................

__

( :z marks)
TOlaJ 14 marks

................................ u

!""

Wben a nucleus of
I decays, it emiU a B- panicle and Y- radiation and becomes.
lIenon. for whlclt the symbol is Xc. Wriu: III equation for a nuclear reaction to represcll1
this decay.

__-_

( 2 marks)
(b)

After bow many days

will';' of thc sample have decayed?

_-_.

_ --_ __

(iv)

....

( 1 marks)

PHYS - P2 + P3 - JU 1998 - Page 4of7

87

88
5.

(a)

(i)

Staff: Archimedes' principle.

Use lJ1e infonnatiun given in Figure 6 to find the volume oftbc rock in em).

( J marks)
(ii)

( 2marks)
(b)

Swe the volume of the rock in m1


(l ml

A tree is many times heavier 1JIlUJ a naU. In leImll of IIJc for:ccs aaing on the twO objects.
explain why a tree fiOalS in waler but a nail sinks.
""

--- _ - _ _..

"'

_.

_ __

-..

(iii)

= 1 x lO'cm

1 mark)
)

If the roct has a mass of 0.12 kg, findilS density in leg ml .

.
( ) marks)
(iv)

( 2 marks)
(c)

W~uld the rock OO:lt or sink. in the liquid bromofonn whiCh bas a density
of 4000 leg m'l '! Exphlin your answer.

The apparatus shown in Figu.ne 6 below Is used in IIJc detenninalion of the density of a
rock.

emJ

t:mJ

100

100

!It

70

70

( 2 marks)
90

18

"
50

(<1)

On Earth,the gravitational field slrengtll is mUClllarger than thai: on the Moon. If a piece
ofrocE were t:lkcn from the Moan 10 the E4U1h. Stale whlltcbange. if any. there would
be in the mass, weight and density ofthe rocle,
.
Mass

"

Weight

50

Density

.............._
~

"--- " .

,,

"

"'

""".~

()

( J marks)

40

Total 16 marks
JO

zJ

_H J-Rotk

It

Figure'

PHYS - P2 + P3 - JU 1998 - Page 5 of7

88

...

89

PHYSICS PAPER 3 - JUNE 1998


General Proficiency - IHr
1.

2.

(8)

(a)

~LC.OIItpllt

Sometimes tight is In::w:d as wave.1l and at other times it is represented by.suaiSbllints


arrays.
(i)

Staticmv, coil

. Soli iroll core

Describe, wilhthe aid of a diasratn. an experimcnllo show lhat liglu does consist
of waves.
( 5 marks)

( 1 mark)

(ii)

Eltplain tbo term 'diffraction'.

(ill)

Uglu can be trelUed as mys traveWni in suaiglu lines becallSl: diffraction is


usually negligible, Explain w by t..his is so. and live an example of aD observation
or demonstration which suppons the view Lbat Ught travcli in straigllliines.
( 1 marks)

Soll iron (ore

J-:=:1

"a.c. output

(b)

The diagram above shows the maincomponenlS in an oil-fwd powerslalion. The boiler
produa:s lIleam wbicb is used to spin the turbines. The turbines are connected to a shaft
whkh rotates III cJcctromq;nel
(i)

(ii)

Eltplain how clearieit)' is genenu.e6 when the turbines spin. Stale the main
energy changes. in thc conect order. rhaI o<x;ur in this power SWion.
Hydro-elearie po-e stalions are IIsc:d in several tenitories In thc Caribbean.
The water in a rim'. racher than steam, is used CO spin the turbines. Stale TWO
advantaps of using this ~ ramer than usilJf oil
( 8 maries)

(b)

Glass fibre

't

The outpUt of the generalOf in a power Stalion is at I vol. . of 25 ltV and a step-up
ftlUlSfonner lben raises the voltage to 70 kV far dJstribalfon.
(I)

(ii)

(iii)

Calculate the r.Iliu of the numberoflums on the reeondary coil to the number on
the primary coil of the Jransfonner. If the power outpUt IS 54 MW. find the
current in the wiles of the secondary coil.
( 7 marks)
Aner distributJon, the VOltage is stepped down to I I0 V, A toaster connecred to
LIte 110 V supply convens energy at rate of 440 W. What is the worlcing
resi.'ltlll1Ce of the tOlSl.er?
'
( 3 marks)

DUe to a fault on the disDibulion line. the VOltage supplied 10 the toaster rises to
( 1 marb)
140 V. Suggest what might occur and explain why,

Data:
Speed of Iighl in ail
Speed of lillu in glass
Wavclenath of liglu in air

3.0 X 10' m 5"'


;; 1.9lt 10' mil"
.. 1.2 x IO-;;m

The diagram above iIIWillall:.s the way Iillu tr:\velli through II glass fibre. The ray onigbt
stays willtin the glass because ofrepealed 10lal insemal relleclion. Use !he data provided
to

answer !he foUowing questions.

(i)

CalculUlc Ibc refractive index OWIC glass and fimhlle limallest possible valudor
angle. i. if the lighl is to undergo lotal internal rcfkCLion.
( 5 marksl

(ii)

Calculate Ihe frequency of the light in air and 100 frequcncy and wavelength of
Ule Ughl in Lhe glass fibre.
( 7 marks)

Tol8no marks

Total 10 marks

PHYS - P2 + P3 - JU 1998 - Page 6 of7

89

II..

90

"'.

J~

(i)

(a)

(il)

Define the terms 'speed' and accelmm~'. Exp. ~Ih aneQS'llple. bow it
ispossibleroranobjecitObeaa:elcrating.wtdleitiS'uavellingllloo~rpoed.

4.

(a)

The diagnm below shows a type oC flashing. lamp colll2i.ninll a bimetallic strip.

Arunnerinam:eaccelcra.tc:sUllifonnlyf'romn:st.l1lI:nl1'llvc1sllaCOl'lSllUllspced

filamellt

and only slows down near Ihe end oC the race. SQ;JCh Ihe speedAime pph COl'
Ihls motion. Whal is meant by Ihe AVERAOE_EO oC the nmncr?
(I marks)

.... . .

Bimtlllilic ;---

In 1991. Cad Lewis ran the [00 m race in 9.87 s. The lime tala:n to COVei' cadt [0 m
SealOR Is shown in Ihe table be[ow.

(b)

Tune/'

1..81

l.0ll

0.91

0.89

Dislanl:dm

0-10

101J)

2030

30-40

0.84
40-50

0.&4
50-60

Il.I4
6Q.70

0.84

0.B6

0.81

7Q.l1O

ao.90

90-100

suip

He acc:e!C:riu.cd Cor Ihe rust 40 m and ran at an approximately eonsmnt speed Cor Ihe next
40 iii.
(I)

(i)

Find the value oCIIis speed in thescction 4O-SOm. and QSC Ihls value to rmd his
acceleration over Ihe rust 40 m.
( 7 marks)

(i1)

If Lewis had I mass oC 83


accelerating.

(ill)

~ was his avemge speed

le,. fll'ld the

aVCl8ge

(il)

Distinguish betweenlhe terms 'specific hculcapacity' and Ileatcapacity. SjaIe


( .~ ~ilFU)

Ihe S1 unit Cor he:u capacily.

(iii) . The lamp convedl Slit to lhennal energy and Slays on for 2 secoriiif; bc{~
switeh.inr; oCf. Durina this lime Ihe tempcrll1we oC the birnetallil: suiP'risc.s by
80 Ie. Calculate the heat capacity oC the bimetallic strip. '.
. ( 3 marks)

Corce thll he cxel1Cd wbi1c


( J marks)

Cor the complete rau7

Explainbowmelampisabletotlashwhenil.isplllggedilltolheclecuicalsupply.
Suggest slliWllc malCrials for Side A wuI Side B of the bimet.allic strip :md
expl. your c:bolcc. .
( S marlu;)

( 2 marks)

(b)

TolaIlO. marks

Each oCme liimps in the circuit belowUlkcs a ClUlCnt oC 0,(17 A from the 120 V mains
supply. The lamps are alll.lllllthe same time.

FUll
5.

(a)

DesCribe how the specific latent he1lt or \/lIPOrizatlon or WlllCt.t... can be detennlned in
the labomtory. usin, an electrical. method. include a circuil diagram in your answer.
S141e clcllI"ly the measurements which are needed. :md show how the value ofty is .
caJcuhned Crom Ihcse mc::lSurcmenlS.
.
.

~uov

( 8lDl1rks)
(b)

In a RUclcllr power SUltion. the energy n:leascd by the fISSion oC unuUum-2JS Is used to
boil waler to make steam whic:lJ drives the turbines.
0) . One ofIhe possible flSSlon n:acdons IsrcpI'CllCnted below. The IlOmic number
and mass number of Ihe Uypl.On nucleus (Kr) MVC been omilled.

~n +- mU
n

-t I"" Sa
56

Jet

+-

In

+-

(i)

How much ClUlCnt Dows in lhe wire. PQ?

( :I marks)

(Ii)

Wbll.t is II slliLliblc current l1l1ing for the fuse in this cucuit?

( Jmai'ts)

(iii)

AD a.c. voltmeter is COIUlCCtCd. in tum. across

QandR

0n

PandS

How many protons wuI how mMy neutrons docs Ihe Iaypton nucleus conl.1lin?
.
. .
(5markl)

PandQ

( J marks)

What would be the voltmeter reOlding ,in EACH case?


(ii)

(iii)

In this lission =lion. Ihe tow mass of Ihe products Is less than Ihe OriJlnal .
Determii1c Ihe chaniC in mass necdi:d to produce 5.0 x 1 ()I J oC Cl'lCrgy.
( Jmarks)

mass.:

(iv)

u.mp X bums alit. How much current will now flow in Lamp Y?

( Imllrk)
TOlal:lO maru

. Uonly40%ofthisS x 10' J oCcnefIYiSUllnSCerred to boiling water. whatmw

oC Willer would be converled to SLearn?

( 4 marks)

Data: F(lr water tv


.. 2:3 x 10' Ski l
Speed or Uglu: 3.0 x 10' m S'l

. t

END OF TEST

Total 20 fJliU'U

PHYS - P2 + P3 - JU 1998 - Page 7 of 7

90

q.lI

91

J'

PHYSICS PAPER 2 - JUNE 1997


General Proficiency - 1Y2 Hrs

(t-

(e)

Figure I below sbows a SIlI.:Ill. positively charged Ob~t ne:ll' to

:I

small. negatively

Figure 3

ch:ll'ged object. Indic3tC. with:l short arrow,l:!belled F. the direction of the forceexcncd
on the negatively chatged object.

(i)

magnet. Explain why the coil begins tQ rotate when a cunent is passed through

Dr.lW the elect:ric field in the region of the two charges. Use arrows to show the direction
of the flCld.
( 3 marks)

r~

A coil ofwire is mounted on an axle andpIaced between the poles ofapermanent


it.

.1

(3 marks)
(ii)

Figure 1
(b)

What extra device is needed to ensure that the coil continues to rowe?

______._,.

_M_'

A conductor. canyi.ng current into the paper. is repteSented by


in FiJUre 2 below.
The "conduCtor is at right angles to the uniform magnetic flCld sflown.
(d)
(i)

(I)

On Figure 2 below. indl.cate, wilh an amlW,lhe direction oCthefon:e exerted by


the magnetic f:e1d en ':.t'is condaCtor..

MlIll1letie

Reid

( 1 mark)
F1JUre 4 below shows a conductor.@ carrying cum:nt into the paper. Dr.lw
the magn;:.tic field 1lC::lt to tile conductor, using arrows to indicate tile direction
of the rlCld.'

Q9
Figure 2

How could the force on this conductor be increased?

p
- - _.._ - -

(ii)

*"'t'

......- - - - - - - - - , , - - -....- ..- - .

(1marb)
Draw a labelled dj:lgram to show bow you couid"produce a uniform magnetic
rlCld like the OM on page 2.
( :% marks!

Flgure4
( 2.marb)
(Ii)

PBYS-P2+ P3-~ 1997 -Page lof6

. Anodler wU,e, parallel t~

the ~L, ~i.ftg cum:nt in'the SAME: dim:lion. is


posi~ aPointP.: Figure 4 above, show the dlreetion of the force on this
conductor and clearly label it with an F.
"
(Zmarks)
91 "

On

".

Total 15 marks

,;S

Distinguish between v~Gr ana sc:lbr qlllllUities.

')}:)

..

_ _..---_.

__

__

(iii)

_--------_.
-- - _- ...
- .._ - ..__ _ _.._ _..- -

92the
Explain wby the driel' ot the CU' must push harder on the gas ~ to k.ce;s
C:lJ"

moyill.l: at SO kin trl

---._------------
_--_

---_ _----_..
__ _--.__

............ _ _ _

"

..-

'

( :: m:Jru)

"._-..;-_.....

..

-------_.._--_.
_.----.,----.

(dJ

Yipn!
(b)

A small CU' II'llvels ~ a collSWlt sp:ed. uound the cirl:u.1lIr race tI'lll:k shown in Figun: S
above. swting :md rmishing at Point A. The:rxk b.as a radius oC70 metreS and it tms
20 Set;onds to complete tile joll.t'lle'Y- (Takc:lt:lS ~ andc:m:umfen:no: =2ltT, whe."l: lis
(\)

FUId the diswlce nvelled in the 20 So

------------_.

__-------
~.~u

(1 marks)

(iii)

(e)

What is the average vdodty for tile allllpice journey?

-------------'-~--_.

._----

o_rt)

A -.;:It is U1lvcllll1g along a level road at ~O Jan Ir'. Tbe c:ar speedS lip to &0 km lr'. TIle
driver fInds th:lt she b.as to leeep her foot pressed Iw:der on the gas ped:d if sbe wiles to
suy at trois spet!li
(i)

marks;

The pilot nles the plane 111 80 Jan lr'" duc$OUuud:uive to the lIir. Use a se:lle di:lgram
to ruxt ..viLb wb.:lI velocity il.o"!'Hn wlwdit'er.:Uon!be plane will nvel n:l.aive to the E.anb.

( 5 marks)

ToW 16 IBlIrkJI

6.1.13.
(bl

Two spe:lkers, Yibr:l.ting i:lllh:lse. are positioned some dis~nee ap::\.\'1 in::,,'1 open Sp:lCl:.
The Co"eStS of the waves [rom t.'le spe3k..'"l'S arc sbown on Figure 9 bela",',

How woulI1 Aristo!.le's 'law of motion' explain lltis observation?

------_._._--_. _.--------
---_..-

Spe:lku X.

.--------

--'
(jj)

....

w+

( 1 mark)

--------_.._--

--_ -_._-_...
(Z

FI~\JRf

Wind

....

(li) Ftndlbe avenge speed oC!beC1t.


.

.........

...

St. Luci3 is 80 km due south ofMartiPique. '" sm31l aeroplane rues from M3.lti.nique to
St. Lucia on a day wl1l:n a Strong wind of SO kin trl is biowing from the:ast. as sllllOlltl
in Figure 6 below.

-...0

tlle radiU$.)

_--._-~_

...

( 1111SrtsJ
Spc:>ur y.

Complete the following sutelnem of ~ewtotl'S fU'St law of motion:

It the resll1unt feme on a body is ::r.:o it will rcm:W1 at rest or


Figure 9

-----------

--------

(i)

,-------------------_._
(1
mark)

PHYS - P2 + P3 - JU 1997 - Page 2 of6


Il,

ExpWn t..'le t.::=

'COl:lSlrUCUve

imcrferenl\C',

.'

---------.-

._--_ _-----_.......92-..

..

(1 mark)

(ti)

93

Wh:lt TWO factors le:ld ll) the pleSSllle being lower :It gre:uer aIlitudes?

( 1 mark)

-----.....- - - . - - - - - - - . - - - - - -.....-----.------- I
( 2 marks)
(b)

(e)

_-_

In'side a pressure cooker, t..'le pressure is higllerthan normaI. Figure 10 below shows the
pressure cooker being used to cook: some food.
WeiJ:ht

Holuna l0:Jm

Aini3ht Ud

_
_- - _._-----_
_._._-_.__ _--_

__

_._~_._.

__..----

_-
------_ ---_ _.-----_ _ _--_.. ..-_ - - _---_

jaf<lt wive
~ t:_~".

,;._

In terms of the kinetic theory of gases, why does the pressure of lhe gas in a sealed
conWner rise when tbe temperatw:e rises?

..

..

'

_-

..
( 3 marks)

PHYSICS PMER 3 - JUNE 1997


General Proficiency" 1&

12
1~1l)

~~

The gr:spb below shows !he current through u'le resistor in the :Le. circuit shown in
Figure 1.

Figure 10

The lid is ainightexcepl for a small hole. which bas an area ofl x 10""" m I , at the lOp.
A weight is placed on top of the hole 10 close it.

(i)

If the weight used is 0..5 N, what is the preSSlIle due to \llis weight? Conyert your

answer 10 kPa.

.............

-~

(ii)

Supply

_._ _

__ ---_ -

"--".,,

_.~-_

..-._.._._.__....-.._..-...-.-_.---._--..

010

_.-_.- - _-
(i)

-.---.---.-'(4~)

(ll)

At what lemper.uure would the water in the pressure cooker boil?

-("Jil')

What effect might the Il;mper:LtllIe identified in (iii) above have on lhe limA; it
takes for the foOO ll) coolt?
.......

......._

__

" .._

..

_ _ .~.;...._ _

" _ _. ., _ _. _

__

~ "

,,

"."

~.n

(b)

..

(i)

Ofa240and lZ0 burner. which one takes me gTClltercurrent and which delivers
!he greater power?
( 2 marks)

(ll)

Draw adillgnW to show how the bu.nietS are cdnneeted, and lind the equiva.leOl
resistailte of the cin:uit when"they llie all switched on :at the same time. (The
symbol for :l he:uing elemeOl is tbe ~e as the symbol for a resistor.)
(4 marks)

..

To prevent accidents. tbe safety valve opens if the pressure inside tbe p.ressure
cookereltceeds twice &.e pressuze of the 3tmospbere. At what ternpenuure does
this occur?

PHYS - P:Z + P3 - JU 1997 - Page 4 of 6

...

A cassette recorder is normally powered by four 1..5 V cells. It can also be


operated from the 3.t'. malns supply if a suitable adaptor is used. What
Cl:lmponents are required in such an adaptor?'
( 3 marks)

An eleatic Sll)Ye, connected to a 120 V 3.c.-supply:has TIVO 'bwners', EACH wilh a


1'leSista.nee24Q.,andTIVO oIhers. EACH wiIh a resistance ofl2!1. ThcFOURelements
C3!'l be oper:Lted independently

( 1 mark)
(v)

With tbe ald of a circuit diagram, describe how the current through tl".e resistor
could be reCtified. Draw a graph to show bow the rectified current would v3Z'f
What is lhe diIference between this direct current and the direct curreOl from a
battery?
( 5 marks)

( 1 mark)
(iv)

( 3mllrks)

( IlDllrk)

(iii)

Find lhe period and frequency of the altem:lting supply.

wi1h lime.

----_....._--------_ _..-_. _---_._----


..

ResiRor

Figure 1

What is the touI press= acting on !he contents if atmospheric pressure is


100 kPa?
,

tlms

93
~b

Mark,withthe !eUcrC. TWO places on Figure 9 whcfeconstrllCtivcimerfer-..nce


( .1 markJ)

(ti)

(a) . n.e gnphbeJow shows how the boiling poilu of Willer vanes with the pressure of
the air above the water. Use 'the graph to answer 'the- quesUoos on Pages 11 and94
12.

4.

is taking place.

J'ra5m'eJkP.

Expillin the temI destructive Interference'.

(iii)

-_._----_._-----_._-

....

._----.-----------
( 1 mane)
Marie, with a leuer D. TWO places on Figure 9 where <Iesuuctive Interference
( .1 marks)

(iv)

isoccurring.

.-."1 :....

(lvh)l .:.

-:z~

I :.. .
..
"'/

. ......

--:,':f-="': : r=:..:: .

1JOIGjl~~ti~~j~?li:~~~fi.i;k~:E~~h::: -:~'I~~:h4 ~I~::

- . _.-_ ...
' .,_. c}'
--ri-..- '.

~ . ~w;..""

Describe what you would expect to hear as you-walk along the line. PQ.

(v)

l~"""

..
....

~,~

_.

..

..

-.-..

~.

..

mJ -
.- .... -1 ....
.-.,::..:..
V. '.--....

- -_.-._ __ {_
_t~!
--~-

-"

~ :~~=..

~.

~-

( .1 marks)
(vi)

If the two speake:s were moved closer together. what effect would this have on
what you hear as you walk along the line PQ?

---....-......_----------_. _. _.......
au.

. .........._.._._.

a..

3
( l marld

~
r-:-;
The SClLie dbgram. Figwe 7 below, shows pl:me wavefroms
incidelu on aburrier wbicll
lllll .
(i)

Determine the wavelength of the waves before they I'C:ICh the burrier.
......_

_ _

__

__

(iii)

t-tf"'!"(

__

Wh:U will be the wavelength of the .....aves lifter they pass throuib the gap?

---_._--_.._----_. _-_ _--_ _._-_

Complete Figure 7 below to show whlll happens to the waves.

__

.~.

.i:+t++:

; ..

hf :"

~B'
, ." .

.:~

-H-.

.. .

'"'i..,.

,..
.

;.'

ttim

..

'.r--.

(1 mark)

( 2 marks)

Figure 7

(iv)

:.

_
.- .._

( 1 mark)
(il)

':~""'--""""
".
.

I::::z-tl:

has a ll:l.lTOW gap in it.

:zo

On Figure 8 below, show what happens when the gap In the barrier is wider.

( 1 marla)
(i)

Figure 8

A school in Jamaica is sitvared at an altitude of 1000 m. Atl.hC'SChl:loLair


pressure is usulL1ly about 90 kPll. At wlw temperature would yOlleX.pC;ct waicr
to boil'!
..._.

...

,,

...._ ...__.._..__...l.__....._..._._._........

l!'_ ... ~_.~..- . -............

'" (lmark)

PHYS - P2 + P3 - JU 1997 - Page 3 of 6

94

'

(iii)

95

The stove is connected. via a circuit bre::1ker. to a special high cunent cln:uit in
the kitchen. Wbal would be the minimum cumnt raLillg for the cireuit brca.I:er?

(ii)

(3 marks)
Total 20 marks
2.(:1)

The !:Ible below shows some of the properties of the emissions from radioactive
subst1ncc:s.
Type of lQdi;u.ion

R:mgeinAir

Penelr.ltion

Greater than 1 m

Passes through thin

6cm

Stopped by sheet oC
paper

70cm

Stopped by sheet of
aluminium 3 mm thick

Identify .0\. B and C.


(b)

The diagram below shows

5.0

10.0

15.0

20.0

:.s.a

30.0

35.0

Count ratelmilr l

460

267

156

104

60

50

32

34

In lo1e 3bsence of the radioactive source. the ratemetcr records 20 counts per
minute.

Why is it necessary to lind the count rate without the source being
present?

( 1 mark)
Plot a graph of conected count l':ltc ag3inst time 3Ild use it to find the
average value oC the IulC-life.
( 8 marks)
Explain why the graph does not necessarily pass through all the plotted
( 1 mark )

points.

Total 20 marks
(3 marks)

It

method that is used in

soUrce

Time/min

sheet of le:u1
B

In tile labor:ltory. a scientist :ur.empts to lind the half-life of this ~otQpe of


is placed near a deleaor 3Ild the count rate recorded
magnesium. The
every live minuteS.. Her results are shown below.

gSa)

some factories to check 011 the

(i)
(ii)

thickness of the polythene being produced. The Geiger-Muller tube is about IS em from
the IlIdioactive soun:e. and is used to detect radialion that has passed throup the

Explain. with the aid oC a diagram. what is meant by the moment of a fon:e.
Wbal do you understalld by the term 'centre oC gnvity'?

Explain why. in the absence of other forces. an object may be balanced lit its
( "' marks)

polythene.

centre of gravity.
(b)

The diagram below represents a wheelbarrow containing a load of sand.

!.f~

!o!!th~~_

Handles

Roll

(c)

or polylhene

f--O..5m-l

(i)

State., giving TWO reasons. which type of radialion would be most suitable for
this purpose.
( :z marks)

(Ii)

State :mel explain the effects on the mtemeterreading of vari.'ltion in the thickness
of the polythene.
( 1 marks)

The container and load are pivoted at P and have a mass oC 40.0 kg. Their centre of
JI'll.vity. O. is 0.5 m from P and the handles oC the wheelbarrow are 1.4 m from P. The

handles are being held so thaLthe wheelbarrow is in equilibrium as shown in the diagram
above.
(i)

Amagnesium (Mg) nucleus contains IS neutrOns and 12 protons. It emits a beta-panicle


and becomes a nucleus of aluminium (Ai).
(i)

Write an equation for this de,cay.

I(

( 3 marks)

(ii)

Name the principle tbal must be satisfied for the wheelbarrow to be balanced.
Explain how this principle, applies in this case.
( 2 marks)
Calcul:w: tbe upward Corce.which is being provided at the handles.
( 3 marks)

( 3 marks)

(ui)

C;ucul:lle the upward fon:e at the :utle. P. of the wheelbarrow.

(iv) .

Explain the :ldvanuge of redesigning the wheelb3trow so thllt the centre of


gravity is much closer to the point. P.
( ~ marks/
t

PHYS - P2 + P3 - JU 1997 - Page 5 of 6

"(v)

More s:md is :u1dcd to the wheelbarrow. Explain why this II1lIkes the wheel
barrow more likely to tip over to one side.
{ :2 marks'

95

96

(c)

~~!,m

(Iv)

What value does this experiment give Cor the specific latent heat of fusion?
Express your answer in 1 kg-'.
.
( J marks)

(v)

[nher list of precautions. the experimenter stated L.'!.atshe dried the ice wiLh filler
paper :lIld she st:lrted wil.h the w:ue: temperature 10 degrees above the room
temperature 0(29 C. Explain why L'1ese prec:Iutions would m::tke herresultmore
accUl':!te.

The wheelb:lmlw (mass 40 kg) is now wheeled up a r.unp :md pins :! m in height. 3S
shown in the diagnm above. The work done is I 400 1.

(! marks)

C:Jlculate the change in the gravitation:Jl poumti:Jl energy or the ""heelbarrow. Tre:l.ting
the r.unp 3S a machine. c:Jlcu!.alc its efficiency.

'J..

::>

Total 20 marks

The diagram below 5hows a bo)' doing his homework. He has difficulty reaiiing his book under

(g .. IONk~')

( 4 marks)

the sma1lligllt bulb.

"''>. '" \ \ \.1 I I / / /

TOI:lI::O marks

~U,4
(a)

(i)

~
When you dri11 a hOle in meta.! the drill bit becomes very nOL
How could this obseNation be explained using
the eighteenth century c:Jloric theory of heat?
the modern kinetic theory of he.?t?

(b)

( .&marks)

(ii)

How did th;: evidence Crom Count RumConi's experiments in drilling the barrels .
ofcannons convince scielllisa that the kinetic theory was better tl'..an the c:lloric
theory?
( 1 marks)

(iii)

Usc the kinetic theory of !le:lt to uplain the melting of a solid. at consunt
temper.u.ure. wben it is he:ued.
( 1 marks)

A studcm is asked to determine the specific l31ent he:lt of fusion of wat.::r. She StaltS by
warming some Wlller :lIld ~fully pouring it into a st)'T'Ofoam cup. (This cup is used
beC:lUSC it has negligible he.at c:l~ity). She finds the mass of tIle w::.:er :lCd rr.e.asures
ilS initi:Jl tempcr:ir.ure. She then drtes some sm:l!l pieces of ice :md slowly ::dds them to
the w;ucr, stirring until the ice is completely melted. The finallemper.:n:re :md mass of
the water :IlC noted.

The student's uble of results is shown below:


lnitia1 mass of warer
Fina1 mass of water
inilia1 temperature of water
Final temperature of water
Specific heat capacity of water

(i)
(li)

(iii)

(a)

In your answer booklet, dmw a ray diagr:un to show the shadow produced by his he:ld
on the table. What difference would it mate to tJ-.e shadow if the' light bulb were repbced
by a long fluorescent tube? Draw a dias:r..m to i!lustl'::te this C:lSC.
( ! m~rks)

(b)

Light is a form of eleetrom:lgnetic (e.m.) waves. Name THREE other types of e.m .
waves and state which of the three has the longest wavelength.
( J marks)

(c)

(I)

Low-frequency e.m. radialion is used to communiCate with submarines. Tile


frequency used is 30 Hz. lftlle speed of e.m. waves in air is 3.0 x 1'0' ms- l , what
is the wavelength of the 10w-fIequency radialion in air?
( J marks}

(il)

A beam of low-frequency r.ldialion is incident upon the surfilCe of the sea.at :lIl

angle of IS~ lIS shown in the diagr:un below.

:: 250 g.

.. 79 9 g
'" 39C
u
19C
'" 4 200 1 kg-'
4.2 J g-'

How much therm:i1energy did the origina1 ~s of water lose?

\~:
Air
Sea

If the speed of the radi:;tion is reduced to 2.5 x Ifl m S-I in water. determine the
angle of refraction. What is tl'.e wavelength and frequency of the radiation in
Wlller:
( 6 marks}

( J marks)

\vhillWas the g:tin:in cnergy of L'lc MELTED icc as its ternper:aure.rose Crom

o0c:

. ....;

. (3 mar.ks)

Using your lUlSWeIS' to (i) and (il) above. find the llm0l!nl of. thermal energy
required to melt r.,ie ice at ac.
_
. (1'mark f .

(iii)

If radiation of the same frequency is emiued by

asubmarine which is under

Water. at wh.at angle ofincidence would total inlem~

PHYS - P2 + P3 - JU 1997
- Page 6 of 6

~ i\J}'
~

--\..._
tt:;-...). .
f

reflection acetlr,in the sea?


( 3 marks)
Total 20 marks

96

Calculate the density of the glass used to make the marbles.

(vi)

PHYSICS .... JUllU.l996,,-PAPER 1


. General Proficiency - 1 Hour
1.

(a)

97

........................"

(2 marks)

A l.eaCher asks her studcnlS to measure the density of the glass from which marbles are
made. Each group of students is provided with 20 identical marbles,

Figure 2 below shows the. outline of a girl's shoe heel. drawn on graph paper.

(b)

The students are toid thai i.hc volume!lf asphere is!1tl'). where r is the radius. Thus.l!I'et
volume may be calculated if the radius (= diameter) is knowlL

They measure the diameter of one of the marbles WiLh a micrometer, The reading
obtained is shown in the magnified diagram below.

F~4=F;t::::::;~F:::t=.~: an

Figure:%

Figure 1

Estimate the area of the heel of her shoe.

(i)
(i)

What is the reading shown in Figure I above?


Diameter =

(ii)

Express this reading in metres using, standard scientific form.


Diameter

(iii)

Area of heel '"

(1 mark)

mrIL

is standing on one heel'? (g

(lmark)

Find thevolurne of ONE marble.


.~:"

~.~

"

"n

..

.. ..

_._

-:-

..
......................................................... h

( 2f!larks)

The girl's mass Is 4S kg. What pressure does she exert on the ground when she
10 N leg-I)

(ii)

In.

..

:""'

;',

..

..

...................................................................................................................... u

(3 marks)

(2 marks)

Total 14 marks
(iv)

lfthe only available balance is calibrated 10 measure 10 the nearest gram. suggest
a method. which students could use. to accurately find the mass of ONE marble.

..........

......"'

.,

-_

"'

"'

. 2.

(a)

It an object is allowed to fall in the laboratory. it accelerates


(i)

..

The mass of one marble was found LO be 1.1 g. Express the mass in kilOgrams.
Mass '"

kg.

_.-

_-_.._ _. _.._. _

..._

...................................................................................................................................................................
( :% mark..;)
(v)

Define the term accei4ralion..

_ _ _

- __

_.

...:

(1 mark)

(il)

( 1 mark)

PHYS -P2 + P3-JU 1996-Page 1 of6

_ _

Explain why the object. accelerares.,

_
.............................. _ _

.........................

..-.

__. _

__

97

( l mark)

"'i>

(iii)

Why is the aeceleration of free fall the same for all objects at the same place. if air
resiswlce is neglible?
.......

_._

_--_

_ _

__

_ __

In the space relow. sketch the velocity-time graph ior lhe motio:n:lJf
98 t!le tl;Y!"-._,,

_.

_""

(iii)

( 2 marks)
flec:.tromapet

II'OlI ba
Ekc:uie IUlp-CIock

Using the graph. or otherwise. calculate the acce lerationof the ball during itS fall.
. . . . . . . . . . . . . . . . . . . . . . . . . . . . ._

4~

Trapdoor

c::aas=

.............u

"

....

F1gure.l
(b)

The appanlWs shown in Figun: 3 above. may be used 10 determine the acceleration due to
gravity. ofthe smalliton balL The height orraJl., h. and the time offall. t. need to be measured.

I .. marks)

Explain how llle apparaws worts.

(I)

Total IS marks

................................,.

...: _

uu

3.

{a)

Figure 4 below shOws the structure of a neutral atom of carbon. The nucleus has six
protons. Complete lhe diagram to show the correct number of electrons in the outersbell
oflhe atom.

... -:.

(3 markS)';
(ii)

Using this appanllus. a student obtains a value of 0.490 s for t. when h is 1.16 m.
Ca!cul31e the avcroge speed of the ba!1 during the fa!1.
n

__ n

..

figure 4
. . . . . . . . . . . . . . . . . . . . . . . . . . . . . . . . .,

40.

............_

..................... "

""

""

"

" h".""

nn

" "

"".U h

u . . ,,.u

"'n"

"

12 marks)

Hence. calculate the final speed of the ba!1 just before its impact with the
trap door.
.
..........................................................._

"

"

Fill in the blanks in the following sentences.

0)

."."

"

"

""

"."

"

The other panicles in the nucleus are known as

..
( 1 mark)

The nuclide can be represented by the symbol ~ C.


(ii)

A is known as the

and its value is

.
( 2 marks)

{iii}
........ h

( 1 mark)

"'''''

"

Z is -kfI9WI1 as tllen~m ..'

and its value is

.
12 marksl

PHYS - P2 + P3 - JU 1996 - Page 2 of 6

98

.
( 2 marks)

(b)

99

Three isotopes of carbon are r:ldiO:lCti',le. with half lives as shown in Table 1 below.
(ii)
IsotoPe

[Xtermine the energy relc.:I.'icd when one carbon - 14 nUCleus t1c.::.tys.


(c = lOO x 10" 10 S ')- .

"H:lJfGfe

Carbon -10

10 seconds

Carbon -14

5130 years

Carbon - 15

2.5 seconds
h.~ H

""'"

( J ImlrkJil

Table 1

Tola' IS marks

In Figure 5 opposite. tbC decay cur>c for ~ g of the carbon - 10 isotope has been drawn. On the
same axes. draw graphs to show
li)

lhe decay of 4 g of carbon;.. 14

lii)

the decay of ~ g of carbon - 15.

4.

Label the graphs CLEARLY.

'fwi~

Battery
I.sV

3LL~~..J--+~":""':-'t-i-+--l--+---i-r-r-ll-r-:j
10
...

... .' .
.

. .... .... .
__ .. __ . _ . .

:l

..

"

~...

'. .

figure 6

(al

- Car'bO~':lii

'-"'-..: .. 1':. ::

Thccircuit shown in Figure 6 abOve. may be used to investigate the relationship between
appiicd potential difference:llld currenl. for:t 10 ncarbon resistor. The ballery has an
e.m.f. of 3.0 V and neglible intcm:u resi~tana:. The ammeter has negligible resislant'C.
A 20 n rheostat is used to control the current.
(i)

5.0

2.5

20m

-+

Rheoslar
'

7.5

11.5

10.0

15.0

17.5

~o.o

Draw a circUit diagram to represent this ammgemcm of I:omponcms.

Timfh

figure 5
IS mar,kst
(C)

When the carbon - 14 isotope. decays by 13 emission, it becomes an IsotOpe pf Ol1m!,!cn.


The mass of a carboo - 14 nucleus' is 1. 324538 ,~ 10: ;"g. :LIU.l the m:~s uf ;j
nnrogen - 14 nucleus is 2.324510 x 10- 26 kg.
-

(i)

Calculate the dirrerence in mass between nuclei of (arbon - 1~ anu


nitrogen - 14.
. * .. ~

. ...

hu.d" U~.,.,

. .

H u

~u+

..

_.u on 'U'" on u ~.~ 40 . . '

u _ h

..

..

( 1 mark I

I J mark.'il

PHYS - P2 + P3 - JU .1996 - Page 3 of 6

99

.-z,.q

00

Explain why it is neccssary for thc voltmeicr to h:lvC


......~

* h . H

" . ' " ono'

u.~

on

:l

(0

high resistance.
~

_ ,

Find the resistance of the rcsistanre wire.


n

................... H

'."

100
u

(3 marksl

( 1 mark)

(iii)

Complcte Table 2 below, to show how thc meter readings change when the
sliding COntact on thc rheostat is m.Dved from cnd X to the other end Y.

(ii)

Find the resistance of the lamp when thc current flowing is 0.20 A.
..'"

'"

'"

~'"'"

'"

'" u

'"

(You may use the space below the table for your calcu1:ltiortS.)
Rhcostat selling

Rcsistlll1Cc of
whole circuit/O

Ammetcr
rC:lding/A.

VoltmclCr
rcJdingIY

(2 marks)

X
(iii)

The lamp. resistance wire and a bll1tery are now connected in Series.
If the current in the circuit is 0.20 A. dctermine the total potential difference across
the lamp and resistance wire.

Table 1
(5 marks)

............................................................ u

.......................................................................................................................u

(h)

Thc graph in Figurc 7 below. shows how the currents through a small lamp and a piece of
resistance wire change as the potenli:ll difference across each is varied.

..

..

(2 marks)

IA

~Lam,p

Total 16 marks

I'

15Ll-i--+-+-++m~TIttr:r:

~J>

OF TesT:

:zoW--+-+++WH:HrtTtuiJII
'.'

15

....

lot

OSl--t'f---+--t--=:!'~-1--+-+-+--I--+-+-+-+--I--+-+-+-

i/,'l?'
-.
Jl/T' ,

"

0.5

1.0

1.5

1.0

figure 7

25

3.0

-3.5

4.0

Vivo

PHYS - P2 + P3 - JU 1996 - Page 4 of 6

100

PHYSICS - JUNE 1996 - PAPER 3


General Profidency - 1BQ11!"~ __
1.

3.

(al

She lined up three idcntical steel balls (A, Band C) on.a tr:lCk. and rolled Ball A towards
the othertwO (sec figure I). After the collision. Ball A c;une to rest next to Ball B. but
Ball e moved off with. apparently. the s;une velocity that Ball A originally had. .

This question is about some of the physical principles involved in scuba diving.
Scubadivers breathe:lie from steel tanksc:mied on their backs. The tankscontain air at a pressure
of 2.0 x 10' Pa. At atmospheric pressure. this :lie would occupy a volume of 2.5 m l ,
(a)

(b)

A te:lCher used the following experiment to demonstrate the law 101


of conservation of
momentum:

8~OO

Explain. in terms of the kinetic theory of gases:


(i)

How the :lie exerts a pressure on the inside of the steel tank

(ii)

Why the pressure in the tank increases when the tank is left out in the sun

(iii)

Why the pressure inside the tank decreases as air is released

Dctenmne the volume of the air when inside the tank at a pressure of 20

Figure 1
(i)

Explain the term 'linear momentum',

x 10' Pa.

(ii)

State the law of conservation of linear momentum and explain how it applies in
the experiment above.
( 3 marks)

(iii)

In terms of the forces acting. explain how Ball A C:lffie to rest and Why Ball B
remained stationary.
( 4 marks)

When the tank is in the Sun. the temperature of the air inside reaches 40 0 e and its
pressure.is 2.1 x 10' Pa. Underthe water. the temperature of this air goes down to wOe.
(i)

Oi)
(d)

Assuming that the volume of the tank does not change when it is underwater.
detennine the pressure of the :lie in the tank. when it is underwater.
(4 marks)
Express the change in air pressure. when thetaiLk is underwater. asa percentage
cf the air pressure when the tank is in the sun.
( I mark)

(b)

(i)

State the magnitude of the total momentum possessed by the twO vehicles before
( 2 marks)
the collision and explain how you arrived at your answer.

Oi)

The van was travelling at 12 m sol. Determine the speed at which the car was
travelling.
( 3 marks)

(iii)

== 1.0 x I cY kg m- J
= 10 N kg-I

..1m 5- t

Atmospheric pressure =, 1
10~Pa
oe :: 273 K

,1..

(a)

(i)

( 1 mark)

A car of mass 800 kg. and a small van of mass 1800 kg. travelling in opposite directions.
collided head-on and both carne completely to rest.

Because of incrcasing pressure with dcpth underwater. there is a limit to the depth to
which a scuba diver can safely go. ea1culate the tota.! pressure exencd on a diver who
is 24 m under the surface.
( 4 marks)
DATA: Density of water

8~

00

( 8 marks)
(3 marks)

(e)

After the collision

Before the collision

12

TOlal20 nrnrks

Draw a ray diagram to show the formation of an image by a diverging lens. State.
with ONE reason, whether you think: the image is real or vinual. (4 marks)
0.1

0.2

0.3

0,4

tis

One defect of the eye is 'shan-sight'. Explain what this term me:.utS.

Figure 2

Draw" a ray diagram to show how a diverging lens m:lY be used to correct this
dcfcct.
( of marks,
(b)

When a convcrging lens Conns a real im:lge. the magnification of the image can be
c:llculatcd from the equation
"J".
image distance
magm Icatlon::
"
.
.
object distance
With the aid of a ray diagr:lffi, show lhatthis relationship is true.

(c)

The graph in Figure 2 above. shows the change in the velocity of the van during
the crash. Delennine the magnitude of the force acting on the van during the
crash.
( 6 marks)
(iv)

What was the magnitude of the force acting on the car during lhe.crash?

(lmark )

( 3 marksl

A converging lens is used to form a re:d image of an object placed 15 em away from it.
Given that the image is twice as high as the object. by means ofascale drawing. delermme
the focallcngl.h of the lens.
( 9 marks)
Tolal 20 marks

Total 20 markS

PHYS-PZ + P3-JU 1996-Page 50f6

101
1.1;0

Most of the electricity used in the Caribl:x:an comes from oitor natura.! gas. This q ueslion is about
4 generar.ing
electricity from alternative. renewable. sources ofenergy instead of using fossil fuels.
(a)

Discuss the possible use of TWOalternative sooree5-efenergy. ethel'- than wind or


nuclellr. to generate electricity in your country.

5.

(a)

Explain. in terms of electron fiow. the difference I:x:tween a direct current and an
102 ( 2 marks)
alternating current

(b)

Figure 5 below shows an a.c. generar.or. and the graph. in Figure 6 below. shows the
variation. wim time.. of the cum:nt from the generator.

Include in your answer. a description of the energy changes whkh take place in eadl
process.
( 6 marks)

1IA.

(b)

s
t1millisecon

Horizontal axis windmill

Figure 3

A wind-powered generator is an example of the utilisation of a renewable energy source.

Large blades are turned by the wind. and the rotation of the blades is used to run an
electrical generatOr.

Figure S

The average speed oCthe wind is 10 m S-l and the cireul:ll' area swept out by the blades
is 1300 mZ Consider the cylinder of air passing this area in one second. (Sec Figure 4
below.)
-;10-

Figure 6

(i)

Use the graph 10 find the frequency of the a.c. supplied by the generator.
(3 marks)

(ii)

Using the same scales as were used in Figure 6. draw a graph to show the effect
ofturning lhe generar.or at TWICE the ORIGINAL speed.
( 2 marks)

(iii)

With the aidofa d ~ describe a modification that would enable direct


cum:nt to be obLained from the genern1or.

Wind-1lo-

10 ms- I
~

Sketch a graph of the output of the modified generator when it is rowed atthe
ORIGINAL speed.
( 4 marks)

_IOm_

Figure 4

(c)

(i)

Calcula.te the volume of air passing the blades in EACH second.

I. 3 marks)

(ij)

What is thc mass of this air?

( 3 marksl

(iii)

Calculate the kinetic energy of the air passing the blades in each second.
13 markst

(iv)

The ..:ffidcncy of Ihe gener.nor is 15%. Calculau: the power output of the
generator.
( 3 marks)

(v)

Deseri be w hat happens to the other 85% oCtile energy which is not convened into
elcctrical energy.
( 2 markS}
(Density of air

=:

(d)

Alapower'staliun. the output power from a 10 kV generator is 4 x l<r w.


(i)

Calculate.the CUlTent in the transmission cables.

(ii)

If the transmission cables have a lotal resistance of 20 n. how much power is


available to the consumers at the end of the line?
( 4 marks)

(3 marks)

Explain why electricity is dilotribuled over long distances as a.c. at high voltage. even
1ll0ugh consumers use low voltages. such as 110 V or 220 V.
( 2 marks I

1.3 kg m- l )

Total 20 markJ

Total 20 marks

EN.D

OF E'XA-tvl

PHYS - P2 + P3 - JU 1996 - Page 6 of 6

102

2.

PHYSICS - January 2007

(a) Define the 'potential difference between the ends ofa conductor'.

103

......... - ..... - ..... - .. ",,- . _.

........

Paper 2 - GENERAL PROFICIENCY -1 % hours

Answer ALL THREE questions in this sectio~.

1.

.. ::: ..

(3~~~ks)

(b) Explain why voltmeters used to measure potential difference MUST

A student was asked to fmd the acceleration due to gravity, g, using


the pendulum. She varied the length ofthe pendulum and found the
time for 20 oscillations. Her results are presented in the table below.

have a very high internal resistance.

..... . ...

. ."

"

"

- . - ..

. ..

"

""

"

-._..

."

Length of
pendulum,
11m

I)

2)
3)

4)
5)

6)

0.1
0.2
0.3
0.4
0.5
0.6

Time for 20
oscillations,
tis
12.7
18.0
22.0
25.4
28.4
31.2

Time for 1
oscillation,

........
-

Time for 1
oscillation, rls

TIs

(c) A bulb is rated at 7.5 V. Dry cells ofe.m.f. 1.5 V are used to supply
current to this bulb.
..
(i) Detennine how many cells are needed
................................................................................<1 mark)

(ii)

(a) Complete the table above


(6 marks)
(b) Plot on the page opposite, a graph oflength ofpendulum (1) versus

Cr'\

time squared
(c) Find the slope ofthe graph.

(10 marks)

marks)
(d) Using the slope, calculate the acceleration due to gravity, g, using

Calculate the electrical power supplied, ifthe bulb draws 1.2 A at


its rated voltage
(3 marks)
(3 marks)

(iii) Calculate the resistance of the bulb.

(d) Two of these bulbs are connected'in parallel to the same voltage source.
Draw the circuit diagram showing this arrangement.
(2 marks)
Total 15 marks
3. (a) Define the following terms:

(i) Half-life..
......... n

..

nnn

(ii) a particle

=4~ (-!r)

(X"" 3.14)

(2 marks)
(e) When ~ 2.6 s2, use your graph to determine the length of the
pendulum associated with this result
................................................................................\,l; marks)

....

...(3 marks)

nnn

1'" marks)
.
marks)
..\.'"

(b) The uranium isotope~"U decays into stable Lead by successive

emission of8 a - particles and 613- particles.

(i) Calculate the mass number ofthe Lead isotope


(3 marks)

(t) Describe the procedure the student would have used to obtain the
results in the table.

(iii) Represent the nuclear decay in the standard form for a decay

equation
... -..
~.

........., " "

~"..

. . ..

.lb marks)
Total 30 marks

(3 marks)

103

...

UtI

104
238
9
(c) The 92 U isotope has a half-life of4.5 x 10 years. How long
would it take for the mass of uranium in the sample t~ decay to
its .original value?

"' ' _

~'~'~"M

' ,

_M _ _.,

, ""

,_.~

, ..

A.' _

."......_._ "...._. __ ..""" ...... "... ". __.. "__ '"" ...."..__ "...

'

Express your result in standard form.


I mark)
Total IS marks

.. " , . ,

" __ '-~ marks)


Total IS marks

"

(iii)

5. (a) The Figure 2 shows an object in front ofa converging lens.


Converging Lens

4.

(a)

(i) Define the 'upper fixed point' on the Celsius Scale.


~

TO

__

~~...

M_

__ ."

..

",~

_. _

,~

__ """"." .__

"

11 mark)

(ii) Give the values ofEACH ofthe following on the Celsius


Scale.
Lower fixed point
"
" " ..
"
"._
(2 marks)
Upper fixed point
(b)

As the temperature of a substance changes. some ofits physical


properties may vary. Identify THREE physical properties which
vary with temperature.
..... ,

~-,,_

.......
(c)

~._

'~'~"

.. "

~~

. __ . ,. _

_._

~.

~,,-

.v

~'"

"'

.. - .. --

" ' . _ " . "'

:>=-..n..............

.I

Figure2

(i) Identify the following by using appropriate letters.


(1) Principal axis
" " "
(2) Principal fOcus
(3) Optical centre "
(4) Focallength
m

_" m

_.." .

"" m " " m m

"

"

"

"

"

"

(4 marks)

..

Figure 1 shows a football pump with a gas tight piston placed


alongside a ruler calibrated in em. The piston when positioned
at the 4 em mark traps a volume, V, ofgas inside the pump at
27C and 100 kPa.

(ii) Write TWO formulae for the magnification ofan object.


(1)
(2)

(2 marl{S)
(b)

A ray of light enters water from air at an angle of incidence whose sine is
0.50. The refractive index ofwater =1.3.
(i) Calculate the angle of refraction.
(4 marks)

(ii) Calculate the critical angle for light rays travelling from

water to air.

Figure 1

(4 marks)

Calculate the volume V, given that the cross-sectional area of the


.
. 15 em.
:1
pIston
IS

(iii) It was observed that a ray of light travelling from water to

air at an angle of 53 0 was totally internally reflected. Why

did this occur?

(3 marks)

The end of the pump is sealed and the temperature of the gas
increased to 122C while the pressure is increased BY 150 kPa.
Calculate the NEW volume ofthe gas.
(5 marks)

.....,

(ii)

'"

(i)

A B C

~.~

(3 marks)

~ 1111:;1111111
~
I

F
~

~ .. _

__

., . . .

,...
__ "

"

..
_.'-A

.,

. , ..

104
- END OF TEST

mark)

lI..

PHYSICS - January 2007

(b)

Paper 3 - GENERAL PROFICIENCY -1 ~ hours

Answer ANY THREE questions in this section.

1.

(Specific heat capacity of water == 4.2 x 103, J kg"l, Kt,.


6
Specific latent heat of vaporization and water = 2.200 x 10 J kg I).
(5 marks)

(b) A model submarine lies on a sea floor 325 m below the surface as

shown in Figure 1. The submarine has a mass of480 kg and a


volume ofO.2m3
.

(c) A brass cube ofmass 250 g was heated for a few minutes then quickly
red into a copper calorimeter ofmass 0.065kg which contained 0.12 kg of
water at 25.0 C. The final equilibrium temperature ofthe calorimeter and
its' contents was 80.0Q C.
(i) Calculate the temperature ofthe brass cube just before it was
transferred to the calorimeter.
(5 marks)
(li) Ifan identical lead cube was used with the same temperature as
the brass, would the temperature of the brass be higher or lower
than the lead after heating for the same interval? Justify your
answer.

------------~-------~-.
-------~~~-~----

325m'
tomrfaee

-_- _-_- _
-_
- _
- _
- _
- . -- -.1.... - -- -- - ~- - - - - -- -- -.
..~1-

: l'iprd.

WeJaht

(i) Calculate the upthrust on the submarine.


(ii) The normal reaction force of the sea floor on the submarine
is 1800 N. Find the resultant downward force acting on the
submarine.
(iii) In order to rise to the surface the submarine must expel
water from its ballast tanks thereby reducing its weight.
Calculate the mass of water which must be expelled.

(Specific heat capacity of brass == 380 J kg' l K"I.


Specific heat capacity of water = 4200 J kg' l K I.
400 J kg' l K- I .
Specific heat capacity ofcopper
Specific heat capacity oflead = 130 J kg-I K I).
(2 marks)
Total 20 marks

[ density of sea water == 1150 kg m-3, g == 10 m S-2]


(12 marks)
Total 20 marks
2.

(a) You are provided with the apparatus shown in the labelled
diagram (Figure 2). Describe how you would use this apparatus to
detennine the specific latent heat of vaporization ofa liquid.
Your account should include the fonnula (e) and calculation you
would use to arrive at a result and the units in which it would be
expressed.
. _..V Slliipji-_d."
(8 marks)

q..-:
\;

sWitcla

105

Calculate the time needed to convert 0.25 kg of water initially at 10 Q C


completely into steam. Assume that heat gained by the beaker and the
environment is ignored.

(a) Stat~ Archimedes Principle. Use this Principle to explain how a


submarine is able to float and also to sink in water. (8 marks)

::.. -~-..: ~ ~ :..:..-..;...-.:.:..~= ~ :.:.:::-~- J:':~.1."~

The heating coil in Figure 2 has a power rating of2.0kW.

3.

(a) Describe a simple activity to estimate the speed of sound in air. (8 marks)
(b) During a recent thunderstonn in the Caribbean, an observer noticed that 8
seconds elapsed between a flash oflightning and the corresponding sound of
thunder. Detennine the distance between the observer and the thunderstorm.

(Speed ofsound = 300 m S-I.)

(3 marks)

(c) Calculate the ratio ofthe sine of the angle of incidence to the sine ofthe angle of
refraction for light travelling from air to transportation materiaL The speed of
light in that material is two-fifths its speed in air.
(Speed ofLIGHT IN AIR, C = 3.0 X lOs m SI).

(3 Marks)

(d) A stamp was placed 25 cm from a converging lens of focal length 15 em.
;"'-~"

'-~
. JI .:.

. ...,

".~

,...

" ...

'

...

BIJliDee

- ----~
.

li'I:J:are ~

Calculate (i) The distance between ~e lens and the image


(ii)The magnification of the image

(6marks)
Total 20 marks

105
~2-

k.

106

......-

,wtj':",E;.;. ,

CoIl

~:'!(;1,
l

-""""~' ! . '
~~

...

~,

:..t

.,

(a) Explain the action ofthe simple alternating current (a.c.) generator
shown in Figure 3.
(8 marks)
An a.c. generator supplies 8 A at 240 V to a transfonner whose
secondary coil is connected to a resistive load. The primary coil
has 9000 turns and the secondary has 3500 turns.
Calculate:

the power input of the transformer

(ii) the e.m.f. (voltage) induced across the secondary coil.


(6 marks)

Given the transfonner is 95% efficient, calculate


the power loss in the transformer

(ii) the current that flows in the secondary coil.


(6 marks)
Total 20 marks
5.

1'

0.55

z0
Figure3. Al~am'ent~F.

(i)

....

tuJ)m,

(c)

ltim.

J1Ie:dbJe_

,Englne ~;
,rotation .~

(i)

,.

-X-W.lY

(b)

,+,.",. - "..

Figure 4 shows a mercury (Hg) V-tube manometer which can be


connected to a source of pressure and used to measure its value.

','.".

:..

~~.

Descnoe CLEARLY how you would carry out such a measurement. State the
readings you would take and show how you would convert them to pressure.
(8 marks)
(b) In one particular measurement of pressure the heights of the mercury columns
above Z, WZ and XZ are 1.3 m and 0.55 m.
Calculate the TOTAL pressure being measured.
(2 marks)

(a)

(c) ,The diameter ofthe manometer is 0.5 cm


(i) detennine the volume ofmecury. VM the right arm ofthe manometer tube
(4 marks)
between the levels W and Y.
(ii)

(d) (i)

(3 marks)

Compute the weight, W, of the volume of mercury, VM

Write down an expression for PM, the pressure in the tube due to the
(1 mark)
weight W if A is the cross-sectional area ofthe column.

(ii) Hence. express Px. the pressure being measured in tenns ofW, A, and
atmospheric pressure Pa
(2 marks)

(density ofmercury = 13,600 kg m3 )


(atmospheric pressure "" 100kPa)
(g= 10ms2 .7t=3.14)

- END OF TEST-

Total 20

106

107
PHYSICS PAPER 2 -- JANUARY 2006

General Proficiency - I Y2 Hrs.

Answer ALL Questions


1. Three hundred and fifty grams (350 g) of a solid substance, X, is heated
in an insulated oven at a steady rate. A record of the variation of
temperature as time varies is tabulated below in Table 1.

Table 1
Temperature

T/oC
,--

Time
tis

20

50

80

80

100

200

300

80

105

147

400

550

800

2. (a) State the TWO laws ofreflection.


(i) -----------------------------------------------------------------------------
--------------------------------------------------------------------(1)
(ii) -----------------------------------------------------------------------------
-------------------------------------------------------------------(2)
(b) Figure 1 shows a ray of light AB, incident on the surface of a

triangular prism.

(i) . Draw the path taken by the ray of light AB as it travels through
41l
the transparent triangular prism shown in Figure 1.
(ii) Indicate by labelling, the angle of

a) incidence ~ on the fIrSt boundary

b) refraction r, on the first boundary

___ c_

(a) Using the observation from the Table 1, plot a graph oftemperature
vs time on the graph paper. (your graph should have THREE
straight line sections.)
(11)
(b) Detennine the slope of the graph during the frrst 200 s.

------------------------------------------------------------------------- (5)
(c) Given that the specific heat capacity of substance X is 1720 J kg -1
K"I, calculate the rate at which heat is supplied to substance X.
------------------------------------------------------------------------------ (5)
(d) How much energy is needed to melt substance X completely from its
original temperature of 20 OC?
-------------------------------------------.---------------------------------- (3)
(e) Explain the behaviour of substance X between t = 200s and t = 800
s.

(2)

Figure 1

(c) Light is incident on a glass block ofindex of refraction of 1.5, at


angle of incidence of 45. Calculate the angle of refraction of the .
light.

--------------------------------------------------------------------------- (4)
(d) Figure 2 shows a waterproof spotlight, spotted! placed underwater so
that the light rays it produces are allowed to strike the water's
surface at varying angles of incidence, i.

-------------------------------------------------------------------------(3)
(f) Sketch the cooling curve of substance X if the oven is turned off at
(3)
t = 800 s
Total 30 marks

Air
\Vater

:~

;:..;.;'"
4
I

Fig1lft2
PHYS - P2 - JAN 2006 - Page 1 of 3

107
It'>

....

108
(c) When the fan and the heater are both in operation, the dryer draws
lOA from the 240 V mains. The fan is using 2A.
(i) Calculate the power used by the fan.

(i) Calculate the critical angle in this situation.

------------------------------------------------------------------- (3)
(ii) Explain what happens to a ray oflight at angle of incidence 68,

----------------------------------------------------------------------- (3)
(ii) Determine the resistance of the heater.

--------------------------------------------------------------------- (2)
[Refractive index of water = 4/3]
Total 14 marks

3. (a) An electric hand dryer located in the washroom of an airport has the
specifications: 240 Va.c.;
50 Hz;
lOA
(i) Fill in the blanks in Table 2 below with the physical quantities
associated with the numbers and units given in the
specifications.

Table 2
Specifications

Physical Quantities

50Hz

lOA
240 V a.c.

------------------------------------------------------------------------ (4)
Total 15 marks
4.

(a) (i) Define the term 'displacement'.

----------------------------------------------------------------------- (2)
(ii) State the SI unit for displacement.
-------------------------------------------~---------------------------(1)
(b) Hurricane Hurry leaves Point A and travels 12 kIn due west at
5 km h,l to Point B. At B, Hurry changes course and speed., and
travels a further 16 km due north at 3 km h'] to reach Point C.
(i) Draw in the space below, a vector diagram showing this
movement.
(2)

(3)
(ii) What does the abbreviation a.c. stand for?

----------------------------------------------------------------------- (1)
(b) The only electrical components in the dryer are a motor driven fan
and a heater. Draw a circuit diagram showing how you would connect
these components to a 240 V a.c. source so that they can work
INDEPENDENTLY.
(4)

(ii) Calculate the displacement of Hurricane Hurry from Point A to


Point C.

---------------------------------------------------------------------- (5)'
(iii) Determine the distance Hurricane Hurry travels in moving from
Point A to Point C.
------------------------------------------------------------------------ (1)
(iv) Calculate the average speed of Hurricane Hurry in travelling
from Point A to Point C, in km h,l.

----------------------------------------------------------------------- (4)
Total 15 marks
PHYS - P2 - JAN 2006 ..,. Page 2 of 3

108

109
(e) What would be the force necessary to give the truck the acceleration
you calculated in (c)?

5. (a) State 'Newton's Third Law of Motion' .


-.--------------------------------------------------------------------' (2)
(b) A large truck which has a mass of 9500 kg is parked on a straight
level road.
(i) What is the weight of the truck?
---------------------------------------------------------------------- (2)
(ii) What is the reaction force exerted by the road on the truck?
(g= ION kg-I)
(c) Figure 3 shows the velocity-time graph for ajoumey completed by
the truck mentioned in (b).

----------------------------------------------------------------------------- (2)
(f) (i) Explain how distance travelled is obtained from a velocity-time
graph.

----------------------------------------------------------------------(ii) Determine the total distance covered by the truck.

(I)

----------------------------------------------------------------------- (2)
Total 16 marks
END OF TEST

Velocity!
ms-1

20
10
50

100

200

300

Ficure 3

400

500

600

o->u

100

SOO

Time { s

Calculate the acceleration of the truck during the fIrst 50 seconds.

-------------------------------------------------------------------------(d) State 'Newton's Second Law of Motion'


(i) in words

----------------------------------------------------------------------(ii) in the form of an equation.

----------------------------------------------------------------------

(3)

(2)

(I)

PHYS - P2 - JAN 2006 - Page 3 of3

109

u.-~

110
PHYSICS PAPER 3 - JANUARY 2006
General Proficiency - 1 Hour
Answer ANY THREE Questions
1. (a) Give a brief account of the Kinetic Theory of Matter and use it to
[8]
explain the macroscopic properties of a gas.
(b) Figure 1 shows a stopper wide-necked flask in which 5 litres of a gas
are trapped. A pressure gauge and thermometer pass through the
stopper via airtight seals to monitor pressure and temperature of the
gas in the flask. The initial pressure of the gas is 1 atmosphere and
the temperature is 27C
1bennoml'ter

II ..

Wide-Necked
FIllsk

011 Bath

[Atmospheric pressure = 1.0 x 1O'Pa]


(iv) What would be the effect on a if the initial pressure were 2
atmospheres while the temperature remained the same as
before?
[12]
Total 20 marks
2. (a) (i)

State TWO differences between the real and virtual images


formed by lenses.
[2]
(ii) Describe an experiment to measure the focal length of a
converging lens.
. [6]
(b) Figure 2, drawn to a scale of Icm = 10cm, shows an, object, X - X',
Y', along with the lens, L - 0 - L' which
and its image, Y
produces the image.

Figure 1

The flask is placed in an oil bath at 127C and the gas in it


allowed to come to equilibrium with its surroundings. What
will be the reading of the pressure gauge?
(ii) The mass of gas in the beaker is 8 grams and the specific heat
of the gas, (at constant volume) is 300 J kg'l K 1 Determine
the amount of energy absorbed by the gas.
(iii) Apparatus such as that shown in Figure 1 forms the basis of an
instrument used to measure temperature in some laboratories
because the pressure indicated is directly proportional to the
temperature,

Figure 2

(i)

that is, P

aT

Vsing the diagram in Figure 2, determine


(i) the object distance
(ii) the image distance
[5]
(iii) the magnification
(c) A ray of light in air is incident on a water surface at an angle of 40.
Calculate
(i) the angle ofreflection of this ray
(ii) the refractive index from air to water.
(The velocity of light in air = 3.00 x108 m Sl)
(The velocity of light in water = 2.25 x 108 m Sl)
[7]
Total 20 marks

Calculate the value of the constant a for the given initial


conditions.
PHYS - P3 - JAN 2006 - Page 1 of 3

110

111
3. (a) The battery charger for a cellphone consists of a transformer and a
diode rectifier. Figure 3 shows a simplified circuit diagram of this
charger connected to a cellphone battery. Explain the operation of
this step-down transformer and sketch detailed graphs showing the
voltage wavefonns across terminals X Y and A - B.
Step.-Down

-Trlmsformer

xr - - ..,

e;t"? "I

1I....J

r -

:}II

-I
j

T _.J

4. (a) (i)

Define
a) 'potential energy'
b) 'kinetic energy'
(ii) Give ONE example of a situation in which EACH of these
[6]
types of energy exists.
[2]
(b) State the principle of the conservation of energy.
(c) (i) A pole vaulter of mass 70kg can run at a maximum speed of
10 m Sl as shown in Figure 5.
Calculate his MAXIMUM kinetic energy.

Cell pboae
battery

I
yl- __

Figure 3
(b) (i)

(ii)

[8]
The transformer in part (a) steps down the mains voltage of
120 V to 9 V. Given that the primary coil has 920 turns,
calculate the number ofturns in the secondary coil.
A current of 30mA flows in the primary coil. Given that the
transformer is 85% efficient, calculate the current in the
~~ary~.

(d) Figure 4 shows a diode testing circuit connected to a test diode.

,..----I
I
I
I

Lamp

Figure 5

~ tJ_D~'
_

(ii)

Dioden.ter

Figure 4

The diode is first connected in the forward-biased direction as shown and


then in the reverse biased direction. The lamp glows in both cases.
Explain this observation and deduce whether the diode is defective- or
not.
Discuss whether this circuit would be useful as a diode tester if the
battery were replaced by an a.c. source.
[4]
Total 20 marks

He performs his vault with perfect efficiency so that his


kinetic energy is completely converted to potential energy.
Show that he will be able to vault a height of 5 metres.
(iii) a) The world record for the pole vault is 6.14 metres. Identify
the factor which could account for this discrepancy
between the 6.14 metre world record and the height of 5
metres shown in (ii) above.
b) Show how the result is improved when it is taken into
account.
(iv) Is there a limit to the height which a human being can achieve
by pole vaulting? Explain your answer. Compare high
jumping and pole vaulting in terms of energy transformation
and hence suggest a reason why high jumpers do not achieve
the same range of heights as pole vaulters.
[12]

PHYS - P3 - JAN 2006 - Page 2 of 3

....

111

*>

112
[g acceleration due to gravity = 10m
[Assume that the average height of the centre of gravity of a person is
approximately 1 metre.]
Total 20 marks

5. (a) The standard notation for specifying a nuclide X, may be written


~ X . Explain the meaning of A and Z and write an equation which

(b)

(c)
(d)

[3]
explains their relationship to the neutron number N.
Define the terms (i) 'isotope' (ii) 'radioisotope'. Identify TWO
properties of radioisotopes which contribute to their usefulness in
[5]
medicine and agriculture.
Determine the distribution of electrons in the atom of Si and
[4]
draw a labelled diagram showing this distribution.
Develop the nuclear equation representing the fusion of two
deuterium ( ~ H) nuclei to form a helium nucleus He ). The
atomic masses ofdeuterium and helium are as given below.
Determine the energy released in this nuclear fusion.

i:

(i

,-

Substance

Symbol

Atomic Mass

Deuterium

2H
1

2.014102 u

Helium

4H

4.00263 u
(8)

( Velocity oflight c

3 x 108 m Sl

Total 20 marks
END OF TEST

PHYS - P3 - JAN 2006 - Page 3 of 3

112

113
PHYSICS - PAPER 2 - JANUARY 2005
GENERAL PROFICIENCY 1 Yz hours

MASS l~Fl
1\:1
F2
Very
small
angle

Answer all questions.


] . Figure] below shows a hunting slingshot which is used to shoot lead balls
over considerable distances.

Figure 2
Given that M 5 x 10- 2 kg, estimate the force in EACH elastic band and
justify your answer.
_ _ _ _ _ _ _ _ _ _ _ _ _ _ _n n

Figure 1
(a) The dynamics of such a slingshot were investigated by using a high speed
camera to photograph the motion involved in a typical shot Some
measurements of distance s travelled by the shot, and t, the time elapsed
since release of the shot, were obtained from these photographs and
recorded in Table 1.
Table 1
Distance
s (cm) .

Time t(ms)
0.0
2.0
4.0
6.0
8.0
10.0

r(ms)2

------------------------------

[3 J

(d) In one particular model of slingshot a pull of 70 N will extend the elastic
band from an initial length of 0.3 metres to 0.6 metres. Calculate the
constant of proportionality of the elastic band.
---------------------------------------------------------------------------------- [3 J
(e) Identify the energy transformations involved in the following phases of
firing the slingshot.
(i) Pulling the elastic band.
-------------------------------------------------------------------- [1 J
(ii) Immediately after release of the shot.
[1 J

_____ u
--------------------------

0.00
i

(f) After release, the lead shot travels freely through the air.
What TWO kinds of energy does it have?
(i) ------------------------------------------------------------------
(ii) ----------------------n________________________________

lJO
3.80
8.22
14.40
22.00

[1 J

Tota130 marks

[3J
(i) Calculate corresponding values of (2 and complete Table 1.
(ii) Use the data in the table to plot a graph of s against i on graph
paper.
[10J
(b) (i) Determine A, the slope of your graph, in ms- 2

2. (a) Sound is produced by vibrating systems.


Complete Table 2 below by identifying the part of the instrument that
vibrates to produce its sound.

[5J
(ii) The slope A represents the acceleration of the lead shot. Calculate
the force required to give a lead shot this acceleration given that the
shot has a mass of 5 x 10- 2 kg.
.

u
--------------------

_ _ _ _ _ _ _ _ _ _ _ _ _ _ _ _ u
--------------------

[3J

(c) Figure 2 shows a simplified model ofthe slingshot in which it is assumed


that the forces exerted by the elastic bands are constant.
PHYSJAN 200S-Page 1 of6

Table 2
Musical
Instrument
Steelpan
Tuning fork

Vibrating System

Drum

Flute
Guitar

[5]

113
t,+\.

114
(b) The following is a brief description of how a student estimated the speed
of sound in air.
Standing at a distance d, 100 m from a large wall, I clapped my
hands, increasing in frequency until each clap just coincided with
each echo heard.
The time for 50 claps was measured and used to compute the round
trip time t. The speed of sound was then calculated using distance d
and the round trip time t.
(i) Why did he stand 100 m away from the wall instead of 10 m?
--------------------------------------------------------------------[1 ]
(ii) Why were 50 claps used instead of 1 clap?
[1 ]

(iii) What other device could be used in its place?

(b) (i)

Determine the current flowing through component X when BOTH


the electric iron AND the bulb are in operation.

-----------------------------------------------------------------------[4]
(ii) Three samples of component X, rated at 2.0A, 5.0A, 7.5A are
available. Which one is BEST suited for use in the given circuit?
------------------------------------------------------------------[1 ]
(iii) The iron and the bulb were both in operation for 3 hours and the
cost of electricity was $0.45 per unit. Calculate the cost of the
electricity consumed.

_________________________________________________ n
-----------

(iii) Write an expression for speed of sound, in terms of d and t.


-------------------------------------------------------------------------- [4]
Total 15 marks

(c) During a recent thunderstorm, a clap ofthunder was heard 12.5 s after the
lightning flashed. The speed of sound is 340 ms- 1 in air. How many
kilometres away was the lightning strike?

4. (a)

Complete the following table relating fundamental quantities, their base


units and their standard SI symbols.

-----------------------------------------------------------------------------[4]
Total 15 marks

Fundamental Quantity

.~~~

(b) A regular solid has uniform cross-sectional area A, height H, and mass M.
In terms of A, H, and M, write expressions for
its volume -------------------------------------------..,---------------- [1]
(ii) its density _m________________________________________________________
[1]

x
150 W bulb

(c) Figure 4 (page 6) shows a plan of a plot of land, drawn to scale, on 1cm
graph paper. The shaded area represents a dwelling house.
(i) On the plan, what distance does I cm represent?
[1]
2
(ii) On the plan, what area in m2 does I cm represent?
------------------------------------------------------------------------ [1 ]
(iii) Use the given dimensions in Figure 4 to detennine, in m 2, the land
area outside the house.

O.75kW
electric
iron

Figure 3

(a) (i)

SI symbol

Current
Temperature
Length
Mass

3. Figure 3 shows a simplified circuit diagram of part ofthe electrical wiring in a


home. The electric iron is connected into the circuit through a plug.

Base Unit

Identify the component labelled X.

(ii) Explain the operation of the component X'and give the reason why
it is used.

-----------------------------------------------------------------------

[4]

-------------------------------------------------------------------------- [5]
(d) What volume of topsoil, in cubic metres, would be required to uniformly
cover the area outside the house to a depth of 15 cm?
---------------------------------------------"t----------------------------------- . [2]
Total 15 marks

PHYS JAN 2005 - Page 2 of 6

114

115
5.

(a) Complete the following table based on the structure of an atom.


Particle
Electron Proton
Neutron
Alpha

Symbol

PHYSICS - PAPER 3 - JANUARY 2005

GENERAL PROFICIENCY - 1 hour

Polarity of Charge

Answer any THREE questions.

~~-

(b) A radioisotope Iodine - 131, has a half life of 8 days. If administered to a


patient, what fraction of it will remain in the patient's body after 24 days?
[3]
(c) (i) If the sun loses mass at a rate of 4.0 x 109 kg S-I, calculate the power
of the sun.

------------------------------------------------------u_________________________
_ _ _ _ _ _ _ _ _ _ _ _ _ _ _ _ _ _ _ _ _ _ _ u

uu_u_____

1. (a) (i) State Charles' law and the general gas law.
(ii) Explain, using the kinetic theory, why the pressure in a gas
increases if it is compressed at constant temperature.
[8]
(b) Figure 1 below shows part of the apparatus in which dry air is trapped
in a container of uniform bore, fitted with a thermometer and an
electric heater. A pressure gauge, metre rule, pump and a valve are
also shown.

[4]

16

(ii) The total power received by the earth's surface is 1.79 x 10 W.

What portion ofthe total power of the sun does this represent?
(Speedoflight
_ _ _ _ _ _ _ _ _ _ _ _ _ _ _ _ _ _ _ _ _ _ _ _ _ _ _ _ _ _ _ _ _ u

3.0 x 108ms- l )
----------------

[2]

Total 15 marks

Figure 1

(i)

Given that the electric heater is rated at 1600 watts, calculate the
heat energy it will deliver to the dry air in half an hour.
(ii) The dry air is initially at a temperature of 19C and a pressure of
105kPa. The heater is turned on for a certain period and the
temperature rises to 95C while the volume is increased to one
and a quarter times its original value. Calculate the final pressure.
(iii) Explain how you would attempt to keep pressure constant while
investigating Charles' law.
[12]
Total 20 marks
PHYS JAN 2005 - Page3 of 6

...

115
'-\-7

116
(i) In your answer booklet copy Table 1 and complete it showin-gthe
displacements of the waves at common points and the sum of the
displacements.

2. (a) Figure 2 below shows the apparatus for a Young's double slit
experiment.

Filter

vertical vertical
Single Double
Slit
Slit

Points

Screen

=:]Lamp

Table 1
Displacement Displacement
of Wave A
ofWaveB

Sum of
Displacements of
Wave A and Wave
B
--

t[

t2

-~

13
14
t5

Figure 2
(i) Explain the function of the filter, the vertical single slit and the
vertical double slit, stating why EACH of these is necessary.
(8)
Describe the pattern that would be seen on the screen.
(b) The wave patterns of two waves are shown in the graphs below.

--

(ii) In your answer booklet draw a graph of displacement against


time, showing the sum ofthe displacements of points tl to ts
against time.
(8)
(iii) What type of interference does this waveform depict?

(c) In certain medium, light has a wavelength of5.0 x 10-7 and a speed of
1.8 x 108 m S-l. Calculate the wavelength and frequency of this same
light when passing through another medium where its speed is 1.5 x
108 ms l
[4]
Total 20 marks

Displacement
fcm

1.0

0.5

oI

..'

\"

~ timets

".

-0.5
-1.0
Displacement
fern
1.0

0.5

I
-0.5
-1.0

timets

3. (a) Define
(i) 'kinetic energy'
'linear momentum'
(2)
(b) State the law of conservation of
(i) linear momentum
(3)
(ii) energy
(c) An object is suspended at rest some distance above the surface of the
earth and then allowed to free-fall to the ground under gravity. Identify
the forms of energy it possesses
(i) when it is at rest above the surface of the earth

. when it is in the middle of its fall

(iii) at the instant it touches the ground.


(3)
(d) In the process of pile driving, a heavy weight falls from a great height
onto a concrete or steel column, called a 'pile'. The pile is driven into
the earth where it can become part of the foundation of a very tall
building.
To model the process, a short steel fylinder of mass 100 kg, and whose
length can be neglected, is made to fall freely from a height ]5 m
above the top of a steel rod of mass 15 kg which is sharpened at the

PHYS JAN 2005 - Page 4 of 6

116

III..

117
bottom end. Initially the rod is held in place so that it just makes
contact with the thick block of wood. After the steel cylinder falls on
the rod, the rod is driven into the block of wood.

oE--- Steel cylinder

mass = 100 kg

(ii) the heating power deliveredto the loud speaker.


.. [6]
(c) If the loudspeaker is operating at an efficiency of 0.5%, calculate the
sound power produced.
[4]
(d) Another identicalloudspeak.er is placed parallel with the first.
Deduce (no calculation required) the effect which this change would
have on the total power delivered by the amplifier.
[2]
Total 20 marks
5. The graph below shows the voltage wavefonn of an a.c. power supply.
Voltage (V)

156
15m

.il

..../

.....\

.J

iJ'

Time (ms)

-156
(

In a laboratory demonstration a teacher placed this power supply in series


with a resistor and a semiconductor diode.

Wooden block

(i)

Calculate the potential energy of the steel cylinder with respect to


the top of then steel rod. Hence compute the speed of the
cylinder just before it hits the rod.
(ii) Given that the rod and cylinder move together after the impact,
calculate the instantaneous velocity of the combined body
immediately after impact.
(iii) Frictional force brings the rod and cylinder to rest after the rod
has been driven 0.05m into the block of wood.
Calculate the
magnitude
of the
frictional
force.
(Neglect any changes in potential energy occurring after the
collision.)
(12]
Total 20 marks
4. (a) (i) Draw a clear, fully labelled diagram of a moving-coil loudspeaker.
(ii) Using your diagram to give a brief e>;.planation of how the
loudspeaker converts electrical signals to sound.
(8]
(b) An amplifier supplies a signal of 40 V to a loudspeaker coil of
resistance 8Q. Calculate
(i) the current drawn by the loudspeaker coil

(a) (i) Draw the circuit diagram of this arrangement and describe the
operation of the diode.
(ii) Draw a graph showing how the voltage across the resistor varies
with time.
(iii) What is the difference between this type of d.c. and that from a
battery?
(8]
(b) (i) Detennine the period and frequency of the a.c. power supply.
(ii) If the voltage across the diode is O.7V and the resistor has a value
of 3.8 kQ, calculate the MAXIMUM voltage across the resistor
and the corresponding MAXIMUM value of the current flowing in
the circuit.
[7]
(c) Many modem electronic appliances require a low d.c. voltage to
operate, and to satisfy this requirement the input a.c. voltage of 156 V
may be transformed before rectification. A transfonner to be used for
this purpose has 648 turns on the primary. If the secondary voltage
required from the transformer is 6V, calculate the number of turns of
the secondary circuit, assuming the transformer to be ideal.
If the transformer is non-ideal, explain how this would affect the
output voltage.
[5]
Total 20 marks

PHYS JAN 2005 - Page 5 of 6

117

*K

118

Figure 4

118

(f)

PHYSICS PAPER 2 - JANUARY 2004

GENERAL PROFICIENCY - 1 'h hrs

119

Find the value of the forces on the bob when it is in theequiU~Iium

You are to spend no more than 'h hour on this question.


The time for 20 oscillations of a simple pendulum was measured as its length,
I. was varied. The results are tabulated below

I.

Time for 20 oscillations tis


Length Vm

18.8
0.2

24.4
0.4

31.0
0.6

36.4
0.8

40.0
1.0

43.2
1.2

._
2(a)

Period TIs
-v'1 In 'f,

Solar radiation at the earth's surface consists mainly of electromagnetic wave


in the wavelength range 0.3 J.lID to 3.0 J.lID. Name the THREE types of
electromagnetic radiation whose wavelengths lie within this range.

-_ __............_-_ ..._------------------------........_------------ ........... _------_


......_-------_ ...........

(a)(i) Complete Table I above.


(4 marks)
(ii) Explain why it is better to measure twenty timed oscillations instead of one

oscillation.

------------...---------------------------------------...----_...---- ..-------------
(3 marks)
(b)
Name ONE type of electromagnetic radiation whose wavelength is
(i) less than 0.3 J.lID

(1 mark)

Use the data in the table to plot a graph of;7 against T.


Find the slope, S, ofyour graph.

(9 marks)

(1 mark)

(ii) greater than 3.0 JIm.

(c)

(1 mark)
Name ONE property which these three types of radiation have in common.

(d)

(1 mark)
Calculate the frequency of electromagnetic radiation of wavelength 1.5 JIm.

(5 marks)

Jif
(d)

(3 marks)
Total 3 I marks

. ......

Table I

(b)
(c)

po~ij:LQ!h_~

Given that 6.28

S, where g is the acceleration due to gravity, find g.


(2 marks)
(e)
The mass ofthe bob is 130g and the length of the pendulum is 1.2 m. The
maximum vertical displacement of the bob above its equilibrium position is
1.8 cm.
(i) Calculate its potential energy at this point.

(Velocity of electromagnetic waves in vacuum = 3 x 10' mls)

(e)

(3 marks)
A monochromatic (single wavelength) beam of light enters a glass block

whose refractive index is 1.5. Calculate the velocity of light in the block.

(3 marks)
(ii) a) On the diagram of a simple pendulum shown in Figure 1, indicate, by
drawing arrows, the forces acting on the bob.
(2 marks)

(3 marks)
Total 12 marks

/
3(a)(i) Draw a circuit diagram showing two resistors R 1 and Rz in series with a
battery.

~:

- - - "fUcm

,. ,,/

(1 mark)
(ii) Write down an expression for the equivalent resistance ofthis combination
Figure 1
----------------------------...------------------
b) Name the forces by labelling them on the diagram in Figure 1_
(2 marks)
(1 mark)
PHYS - P2 + P3 - JAN 2004 Page 1 of 4
119

........ _-_-""",..".

-------

t-\-q

120
(b)(i) Draw a circuit diagram showing two resistors R 1 and ~ in parallel with a

(ii)

Name TWO metals that could be used to make a bimetallic strip:

battery

(2 marks)
Total 19 marks

4(a)

State Newton's Third Law of Motion.


(2 marks)

(1 mark)
(ii)

(c)(i)

(ii)

(b)

Define the moment of a force and state its SI unit.

(c)

(3 marks)
Figure 3 shows a wheeled suitcase, of weight W, supported in the. stationary
position shown by applying a vertical force Y at the towing handle.

Write down an expression for the equivalent resistance of this combination.


(I mark)
A set of Christmas lights, consisting of 100 identical bulbs rated at 1.0 W
arranged in series, is connected to the 110 V mains. Calculate the voltage
across EACH bulb.

(2 marks)
If all the bulbs are ON at the same time, calculate the current through EACH
bulb.

(4 marks)
(iii) Calculate the resistance ofone of these bulbs.

Figure 3
The mass of the suitcase is 25 kg. By taking moments about the wheel,
calculate the magnitude of the force Y.
(d)

(3 marks)
A bimetallic strip may be used to make the lights switch on and offrepeatedly.
A schematic of such an arrangement is shown in Figure 2.
(6 marks)
An additional horizontal force X is applied a- the towing handle so that the
suitcase begins to move in a horizontal direction.
Deduce the relationship between the force X and the horizontal frictional force
at the point of contact with the ground when the suitcase
(i) is moving at constant speed

r----~..!!f!~----.,

D-Cl~

E~--------------

(d)

,I

I...--..B

Figure 2

(i) Identity the components labelled A, B, C and D.

I,.,l

mv maIDs

(I mark)
(ii) is accelerating.

A-----------------------------------------------------------------------------------

B..;-----------------------------------------------------------------------------
C------------------------------------------....:--------..-------------.----------.
D-----------------------------------------------------------------.--------------

(l mark)
Total 13 marks

(4 marks)

I
PHYS - P2 + P3 - JAN 2004 - Page 2 of 4

120

5(a)

(b)

Define specific heat capacity and state the relationship between heat capacity
and specific heat capacity.

(3 marks)
1.5 kg ofwater initially at 19C is heated at 45C Calculate the amount of heat
energy supplied to the water.
(Specific heat capacity of water = 4200 J kg010CI)

PHYSICS PAPER 3 - JANUARY 2004

GENERAL PROFICIENCY - Ihr

1(a)(i) Derme the following terms.


a) Kinetic energy
b) Gravitational potential energy
(ii) Write the equations used to calculate these quantities.
[4]
(b)
Describe and explain a physical situation when kinetic energy is transferred to
potential energy or the reverse.
[4]
Vertical Wall(c)
1

(4 marks)
(c)

Derme specific latent heat of vaporisation

If'

(d)

121

(2 marks)
An electric kettle rated at 2.5kW contains l.4kg of water. The kettle is
switched on and brings the water to boil. Calculate the time taken to convert
the entire amount of water in the kettle to steam after it begins to boil.
(Specific latent heat of vaporisation = 2.26 x 106 Jlkg)

4 -

- - f-----

:1

IA

:1

10

m-+

Figure 1

(6 marks)
Total 15 marks
END OF TEST

(i)
(ii)
(iii)
(iv)

Starting at 0, an insect of mass 0.01 kg crawls along the base and then up a
ol
tiled wall as shown in the diagram at a constant speed of 0.02 ms
What is the total distance travelled by the insect?
How long does the insect take to crawl from 0 to C?
Calculate the magnitude ofthe displacement of the insect and its kinetic

energy.

[12]
Calculate the change in potential energy between 0 and C.
Total 20 marks

2(a)(i) Draw a labelled diagram of a thermocouple thermometer and explain how it


works.
(ii) List TWO advantages of a thermocoup1e thermometer over a mercury-in-glass
[8]
thermometer.
(b)
Water initially at 30C enters a shower head and is heated by.a coil rated at"

1200 Was shown in Figure 2.___ __-..;:-e: 3d'c

Shower

bead

Hot

".ter~,'
, ,:
,, -,,
,

PHYS - P2 + P3 JAN 2004 - Page 3 of 4

Figure 2

121

50

122

(c)

A kilogram of water flows through the shower each minute. Calculate the exit

temperature of the water assuming that all the heat is transferred from the

heating coil to the water.

[8]

(Specific heat capacity of water = 4200 J kgloC I )


List TWO advantages and TWO disadvantages of this type of heater over a
solar water heater.
[4]
Total 20 marks

3(a)

Describe with the aid of a sketch the Young's double slit experiment. Sketch
OR describe the interference pattern seen. What did Young deduce from this
experiment?
[6]
(b)
State and explain the effect on the interference pattern of the following:
(i) White light is used as the source
(ii) One slit is covered with an opaque material
(iii) Apiece of glass is placed in front of one slit
(iv) The slit separation is reduced
[6]
(c)
Explain the difference between the interference pattern produced by fed light
[3]
and that produced by green light.
(d)

Figure 4
(i) Determine the value ofthe current which flows in the circuit.
(ii) Suppose the resistor R were replaced by a short thick piece of conductor. What
current flow would you expect? Hence deduce a reason for including R in the
circuit.
(iii) What current would you expect if the diode were reverse biased?
(iv) The d.c. source in the circuit above is replaced by an alternating voltage source
of amplitude 9V and frequency 50 Hz. Sketch a labelled diagram of the current
through the circuit.
[12]
Total 20 marks
5(a)

(b)
(c)

Describe the Geiger-Marsden experiment and state the conclusions about the
structure of the atom which can be deduced from the results of this
experiment.
[8]
Sketch the shell model of a Carbon -14 atom. Carbon -14 decays by emitting a
beta particle. Write the nuclear equation for this process.
[4]
One ofthe simplest fusion reactions involves the production of deuterium
?

(HI) from a neutron and proton.

Dispiaumebt

0.1 ill

HI +no --HI +0

41:\

\.~

/..

) Time
<IllS)

How much energy is released in this reaction?


1

HI = 1.007825 u n o = 1.008665 u
?

Hi
Figure 3
(i) What is the frequency of this wave?
(ii) Sketch the curve of another wave of the same frequency and phase, but twice
the amplitude.
[5]
Total 20 marks

=2.014102u

lu= 1.6605 x 10.27 kg


[8]
Total 20 marks

END OF TEST

4(a)(i) State TWO differences between alternating current and direct current.
(ii) Most. electronic circuits require low voltage d.c. power for proper operation. If
they are to utilise an alternating current,(a.c.) source then conversion from a.c.
to d.c. must be done. State the THREE steps involved in this conversion and in
[8]
each case give an example of a" device which may be used.
(b)
The diagram below shows a circuit used to investigate a diode.
R

9V

PHYS - P2 + P3 - JAN 2004 - Page 4 of 4

122

123
(n) The results ofthis experiment can be used to verify Boyle'5
Boyle's Law as clearly and as completely as you can.

CKC PHYSICS PAPER 2 - JANUARY 2003


GENERAL PROFICIENCY - 1 Y2 brs
1.

10

\10

...

Flat plate

I.

l'tUftle1'

(ill) State TWO precautions that Boyle's Law suggests for this f

o
__....

..._M

an

L::':~H---- Piston

'''; II

l==il ~

(c)(i) The piston has a radius of2.5 em. Calculate its area in m2

Trappm ps

'M

. . .

Sealed bottom of pump

Figure 1
A bicycle pump is modified by replacing the handle with a flat plate

(n) Calculate the pressure applied to the trapped mass ofgas w

onto which masses can be placed. The plate carries an attached pointer
that is next to a vertical scale as shown in Figure 1. The bottom of the
pump is sealed so that a fixed mass of gas is trapped inside. Initially
masses, M, are added to the flat plate wtil the pointer reaches the 5 em
mark. The masses are then removed in steps, the trapped gas expands
and drives up the piston. increasing the upward displacement L.
.Corresponding values ofL and M are recorded in the table below.

of 100 kg rests on the flat plate.

To
2(a)

Explain what is meant by the term lelectric current' and give


for an electric current.

(b)

Figure 2 shows the internallayollt of a torchlight (flashligh1


electrical circuit.

(Assume the temperature of the gas is kept constant tbroughou.t the experiment)

TABLE 1
MusM(lgr\
Dill
Lfcm)
!(cm-1)

118

48.0

5.00

10.0

25.0
15.0

13.0
20.0

2.00
30.0

6.00
25.0

(a)(i) Calculate corresponding values of

and complete Table 1.

(4 mks)

Cell

(n) Plot M vs 1 on graph paper.

(9 marks)

(ill) ____
Determine
the slope, S, ofthe~ graph and
give its unit.

~_w
~

._._. . . . . .

1.5V

Copper strip

Copper strip

(4 marks)
(b)(i) What is the value ofL when M =::: O?
_ _ _ M

WM.

. .- . - _ .

(2 marks)

PHYS - P2 + P3 - JAN 2003 - Page 1 of 5

Reslstanco or bulb

-~

supply [-:

Figure 2

123

S\

124

(i) In the space below draw the circuit symbol used to represent one of
these cells.

(1 mark)

(b)

P- particles move at much greater speeds than a - particles but cause--_~


much less ionization when passing through air. Give TWO reasons
why this is so.

(2 marks)
Radon is a naturally occurring gas which has a halflife of 54 seconds.
(i) What fraction of a sample of this gas is left after 108 seconds.

(c)
(ii) State the type of energy conversion which takes place in these cells.

(1 mark)

(2 marks)

(iii) The component labelled Q is used to connect the copper strips thereby

(ii) Calculate the time that will elapse before only

t. ofthe original sample

remains.

completing a closed circuit. Name this component.


(l mark)

(3 marks)

(c)
The voltage ofeach cell is 1.5 V and the bulb has a resistance of 6.0.
(i) Calculate the current flowing in the circuit.
(3 marks)
(ii) How much energy is delivered to the bulb in 12 minutes?

222

(d)(i) A nucleus ofthe radioactive isotope 86 Rn emits an a - particle when it


decays into a nucleus ofthe element Po. Complete the equation
representing this event by writing in the nucleon and proton numbers.
~ Rn _ Po + He
-------------------------------------------------------------------------~

-----------

(2 marks)
Total 15 marks

Total ~5::-~
3(a)
Describe the nature and origin of the following particles:
(i) a - particle:

14(a)(i) ~~_:~~~_~~~:_~~_~~~~~!~ of:.~:.

._.

_
(1 mark)

(ii) List TWO features that are characteristic of a phase change.


(3 marks)
(2 marks)

(ii)

fJ - particle:

(iii) Ice initially at _5C is heated until steam is formed. Sketch a graph

------..---------------------------------------------._--~------------------ --------

showing how the temperature ofthe ice would vary with time.

(3 marks)

PHYS - P2 + P3 - JAN 2003 - Page 2 of 5

(4 marks)

124

125
(b)(i) 200g ofcrushed ice at -5C is heated until it JUST reaches OC.
Calculate the heat required for this process.

(4 marks)

Suppose the supply voltage were 220 V dc. and the electric stove and
light bulb are rated at 1500 W and 150 W respectively.
(i) Select ONE fuse of correct rating for the electric stove from the list
below and explain your choice.
l.0A, 2A, lOA, 15A

(b)

(li) More heat is added at ooC until it is completely converted to water.


Calculate-the amount ofheat that is added during the heating process.

(5 marks)
(Specific Heat Capacity ofIce = 2100 J kg }("l)
(Specific Latent Heat of Ice = 3.34 x lOs J kg-l)

(3 marks)
Total 14 marks

(ii) Calculate the TOTAL current drawn from the supply ifBOTH

appliances are being operated.

5.

(4 marks)
Electric\ly
supply

Electric
stove

Light
bulb

(ill) Explain TWO reasons why alternating current is MORE suitable than
direct current for country-wide distribution of electricity.

Figure 3.
(a) Figure 3 shows two household appliances connected in parallel to the
electricity supply.
(i) Give TWO advantages of connecting electrical appliances in parallel.

(2 marks)
Total 16 marks

END OF TEST

(2 marks)
(li) Explain the pmpose ofthe fuses.

(2 marks)
(ill) In connecting the stove, an earth wire is also used. State its function.

(I mark)
PHYS P2 + P3 - JAN 2003 - Page 3 of 5

125

7)

126
CXC PHYSICS PAPER 3 - JANUARY 2003

GENERAL PROFICIENCY - I hr

1(a)
(i)
(ii)
(iii)

(iv)
(b)

2(a)(i) State Archimedes' principle.


(ii) Describe with the aid of a diagram a laboratory experiment to verify
(8nuuk~
thisprincipre.

Explain what is meant by EACH ofthe following terms when applied


to a wave in a medium:
A progressive wave
A pulse
A longitudinal wave
(8 marks)
A transverse wave
Some ships employ sound waves for determining the depth ofwater

below them. (see Figure I)

_ _ _ _ ShIp

Hull (bottom of ship)

----::j.

(b)

A block of volume 2 x 10-4 m3 and density 104 kg m3 is suspended


froma spring balance and fully immersed in a liquid of density 103 kg
3
m.

(i) a) Calculate the weight of the block and the upthrust on it when it
is fully immersed in the liquid.
b) Use your results to
the reading of the spring balance
when the block is fully immersed in the liquid.
(ii) What is the density ofthe resulting liquid if 1 litre of the above liquid
is completely mixed with 2 litres of another liquid of density 900 kg
o3
m ?
(12 marks)
(g, acceleration of gravity = 10 ms2)
Tota120 marks

determine

3(aXi) Define 'convection' and list THREE differences between convection


and radiation.
(ii) Draw a simple diagram showing how the direction of sea breezes is
(8 marks)
determined by convection.

UJar-Die tnmsmit1er/ruelver

__ - _ ....".......-- Reflected WlIVe

-----

-----

- - - - - - Tnmsmllted waye

(b)

?/////~~~~//////

Figure 2 shows across section of a solar air heater. The principles of


its operation are similar to those of a solar water heater.

if1
C.. \

Figure 1

These sound waves have frequencies above the range of the human
ear and are therefore called ultrasonic. A short pulse of such a sound
is emitted by the transmitter. It travels to the ocean floor and is
'echoed' back to a receiver close to the transmitter. The depth ofthe
ocean can then be determined from the time that elapses between the
transmission and reception of the pulse.
(i) What difficulty will be encountered ifthe transmitter were to send a
continnous sound wave?
(3 marks)
(ii) Why it is necessary for ~ transmitter to use pulses of sound?
(2 marks)
(c)(i) The ultrasonic transmitter has a frequency of40 kHz and the wave
velocity is 1500 ms-I . What is its wavelength?
(3 marks)
(ii) The time taken for the echo to return is 200 ms, calculate the depth of
~o~

~m~~

[_

(i) What are the functions of

a) the glass?

b) the coconut fibre'?

c) the blackened aluminium sheet?

(il) Explain how the physical properties of these materials make them
especially suitable for these functions.
(7 marks)

Total 20 marks .,
PHYS - P2 + P3 - JAN 2003 - Page 4 of 5

126
.

...~.

(c)

Air enters the heater at 30C and leaves at 100C. Calculate the
energy absorbed by 0.1 kg of air as it passes through the heater.

(d)

List TWO methods by which the temperature ofthe air leaving the
(2 marks)
heater can be increased.
Total 20 marks

(Specific heat capacity of air = 1040 J kg'l 1("1)

127

(li) The d.c. supply is replaced by an a.c. supply what would be the
motion ofthe bar magnet.
""
(12 markS)' ---

Total 20 marks

(3 marks)

5(a)(i) Define the terms 'work', 'energy' and 'power'.


(li) Distinguish between kinetic and gravitational potential energy.
(8 marks)

4(a)

Figure 3 below shows a simple d.c. motor connected in a circuit.


(b)

Coli \ ~rl
'. -'--)1-:11.

_....- - - . 0 3

Brush -"

Figure 5 below shows a body of mass 6 kg moving at constant


velocity along a horizontal, frictionless plane. When it reaches point
A, a constant horizontal force is applied until it reaches point B. The
force is then removed and the body travels up a smooth inclined plane
BC until it comes to rest at C.

Brush

s
Figure 3

Use the figure to explain how the motor is able to rotate continuously.
(8 marks)
(b)

Suppose a bar magnet is mounted so that it can rotate between the


poles ofan electromagnet as shown in Figure 4. The switch S is
initially open.
Figure 5
.... I

Pivot

D,C. Supply

11---.,1

Given that the velocity at A is 15 mls and that the distance AB is


110 m and the force applied between A and B is 37.5 N
Calculate
(i) the work done by the force and hence the average power exerted over
this distance ifthe force acts for 4 seconds
(li) the kinetic energy at B
(12 marks)
(iii) the vertical height of C above B.
Total 20 marks

END OF TEST

Figure 4

(i) Describe and explain in detail the motion ofthe magnet when the
switch S is closed.
PHYS - P2 + P3 - JAN 2003 Page 5 of 5

127

$;'

128
(e)

CXC PHYSICS PAPER 2 - JANUARY 2002


GENERALPROmCmNCY-l%hn

By how much would the ball compress the spring if it were

placed (NOT DROPPED) on the platform?

You should not spend more than 30 minutes on this question.

1 A 10kg steel ball falls H metres from rest on to a light platform


fastened to a spring below it. The maximum displacement, x, of
the platform from its equilibrium position is measured and
recorded. The procedme is repeated for different values ofH
and the results tabulated below.
Height H f metres
Displacement i!fcm"
Impact velocity v"t m" s-z
(a)

(b)

(c)

6.0
324

5.0
250

4.0
196

3.0
157

2.0
112

(4 marks)

(t)

1.0
49.0

The graph you plotted in part.(b) is obtained by assuming that


the kinetic energy of the ball at the moment of impact is totally
converted to potential energy stored in the spring.
Identify the error involved in this assumption.

(1 mark)

By equating the kinetic energy at the moment of impact with the


change in potential energy of the ball we find that the velocity,
v, at impact, is related to the initial height by

(g)

v=~20H :. Y%=20H

Use this relationship to complete the table

--------..--------------- .

(2 marks)

On a graph page, plot a graph of the square of the displacement,


x2, against the square ofthe impact velocity, Y.
(9 marks)
Find the slope, S ofthe graph.

What height should the ball fall from in order to displace the .
platform by 10 cm?

(4 marks)

(h)

Calculate the potential energy lost by the ball before coming to


rest after the impact in part (g).

(3 marks)
Total 30 marks

(5 marks)
(d)

Given that the spring constant k =

2(a)(i) State TWO differences between sound waves and


electromagnetic waves.

!!! N m-1, fmd k.


S

(2 narks)

(2 marks)
PHYS P2 + P3 - JAN 2002 - Page 1 of 6

128

129

(ii) Is this frequency in the audible range? Justify your answer.

(ii) Relate the terms "pitch" and "loudness" to the physical

properties of a sound wave.

(l mark)

(speed ofsound in air = 330 mls)


(2 marks)

(iii} Describe an experiment to show that sound does not travel in a


vacuum.

Total 16 marks
3(a)
,

CA)

FJ
1)010<),

J1V

-=
_uIa.-=IacI-IICid

]Ow

z,cw

60W

(7 marks)

(b)

..

...

w-
:;

we
. : . '!

110

. : ; .. , . .

.:....
."

":!'. ... ..::. ... .. .;.


.'.
.". . ".. . .;.,....
.,
. .." ";.~.....
. .}~
-60

.10

.:...

..

:;.

III

..

:;

:~.... .:~ ~
.;.... .;",.... .~ V .331 uris
i..
:,...
:11."" ,_,.... :,
,.,

:,.;.......:.:......

..

>

JO

60

90

DllCaan

caw

Figure 1
Figure 1 shows how a sound wave travelling through a long,
air-filled tube sets up a periodic pattern of rarefraction and
compression.

(i) Determine for this sound wave the


a) wavelength

SIde

HncI

lip...

liP'"

Park

........

Figure 2
(i) Figure 2 shows a very simplified wiring diagram of part of an
automobile electrical system. Calculate the effective resistance
ofthe headlights.
-------------------------------------------------------------------------------~.-----

(3 marks)

(ii) Compute the magnitude ofthe current, I, when all three sets of
lights are on.
---------------------------------(5 marks)

(1 mark)

b) frequency,
(iii) Explain the function ofthe fuse, FI, in Figure 2.
(3 marks)
(l mark)
PHYS - P2 + P3 - JAN 2002 - Page 2 of 6

129

5"\

130

(iv) Iffuses are available in only SA, IDA, and ISA denominations,
select a fuse rating for Fl. Explain your selection.

--....---.-----------_..-----------------------------------------------------~

(2 marks)

(bXi) State TWO advantages of connecting all electrical devices in


parallel with the battery.

------

. . . -. . . . . .__

-----w

____

-----------------------------------------------------------------------,_.------..

-----_.------------------------------------------------------------------------------
(2 marks)
(ii) Give TWO reasons why a dry cell may NOT be used in place of

the lead accumulator battery used in cars

---------...

------_.-._-------.-._------------------------------------------------~

----..-----------------'!"'---------------------_._
----------------------~--------------------------------.-----------------------------

(2 marks)
Total IS marks

(li) Why is energy released in the nuclear fission process?

-----------------------------------------------------. ---------------------------------..._---

(1 mark)

(c) The graph in Figure 3 shows the activity ofIodine -131 et31 I)

observed over a 36-day period.


53

.......,/llot

--I_.'
,.

I.
,.

,.

J.

1111

.4

.1

II : !\ 'I j 1'1 !l,tlll I 11 ~


. I I';.1
II' IIj!ll.!li
iH1 i ,..
II
1

II 11111
I

(ii) The half-life of a radioactive nuclide


------------._-------------~--------------------------------------------------------

------------------------------------------------------------------------------------(2 marks)

(bXi) What is nuclear fission?

-_ _---------------------------------------_ _----------
....

lli,~ m!~Illl/if ~ii ~II i i rITllilii


I!III Ii ~r1411 J!UH! IIllIi llii!
rllB I Ill! I! i Hili\! i lit It Ii ; Iii; ;ITt1
U t i l i " d l.lfl:'llli I.,: '.11 I.JlII
1.~111 .IJ.~
II , .. I .! 1111111 UINnJoI.......".,.

I.

,.
4(a) Explain the meaning ofBACH ofthe following:
(i) Radioactivity
----------------------------------------------------------------------------_.....-.-
---...---_..------------------------------------------------_.-----------------~
(2 marks)

...

------------------------------------------------------------------------------------(2 marks)
PHYS - P2 + P3 - JAN 2002 Page 3 of 6

ililli

tI

I.

ilPHii

1=,Ill:
'::1

iI
II IIIld!

II I III Ulli

liliidl III ill i 1111 til If


IT,mn II ! iii! 1I1l1111l11 I II IlllUHlI!1
11111111111111111 Jfilii' 1/11/111111 nmHIlIlIIIIII fIIiI
lflll\lli ! III . I t
1 II!

Ii I!lil
I n rn
II 111 I II I
HI II
1lllllliffimHlIIHlil!rIlllillUmmmrnmUIIUIT
Figure]
130

131

(ii)

(i) What is the initial activity?


-----.---.----~-.------------------------------------------------------------~

..--~

(1 mark)

(ii) Determine the half-life of iodine -131.

(1

(3 marks)
(iii) Calculate the activity on day 48.
Figure 4
Figure 4 shows two plane mirrors A and B. The mirrors touch
each other along one edge and are oriented so that their surfaces
make an angle of 50 with each other. A beam of light is
directed towards A at an angle ofincidence of30. Calculate the
unknown angles a. b. c and d in Figure 4 Hence deduce the
angle of reflection (e) from B?

(3 marks)
Total 14 marks
5(a)(i) Arrange the following in order ofDECREASING wavelength.
X-rays, Radio waves. Visible light rays.
(3 marks)
(ii) Name TWO properties which these waves have in common.

(5 marks)

(2 marks)
(b)(i) State the laws of reflection.

(c)

Zircon, a material used for manufacturing imitation diamonds,


has a refractive index of 1.9. Calculate the speed ofIight in
Zircon.

--._-----------~-------------------------_.----------------.------- ---------------_ _ _ _ _ _ _ _ _ _ _ _ _ _ _ _ _ _ _ _ _ _ _ _ _ _ _ _ _ _ _ _ _ _ _ _ _ _ _ _ _ _ _ _ _ MW

(2 marks)

__

(speed of light in a vacuum 3 x lOS m Sl)


Total 15 marks
END OF TEST

PHYS - P2 + P3 - JAN 2002 - Page 4 of 6

....

131

55

132

CXC PHYSICS PAPER 3 - JANUARY 2002

GENERAL PROFICIENCY -1 hr

1(a)(i) State Newton's second law of motion.


(ii) Use Newton's second law of motion to
(a)
distinguish between mass and weight
(b)
define the unit offorce.
(iii) IdentifY the type of unbalanced force which acts on

(a)
(b)

an electron moving around a nucleus


car going around a bend in a flat road.

Shallow water

(b) The elevator in a high-rise building accelerates uniformly from rest to a


speed of 4m S-1 in the first 2 seconds ofits upward motion. It then continues

at constant speed for 4 seconds and thereafter decelerates uniformly to a


complete stop in 3 seconds.
(i) Sketch a labelled velocity-:time graph of this motion. (Do Dot nse graph
paper)
(ii) Calculate the acceleration in the first 2 seconds.
(iii) Determine the distance travelled by the elevator while it is decelerating.
(iv) Determine the total distance travelled by the elevator.
(12 marks)

Total 20 marks
2(a)
Explain the meaning ofthe following terms as they relate to a wave:
(i) Amplitude

(ii) Period

Gluiliwet
water

(8 marks)
Deepwater

Figure I
(i) If the period ofthe incident wave is 0.1 seconds, the wavelength ofthe
incident wave is 0.5 cm and the wavelength of the refracted wave is 0.3 em,.
calculate the
a)
speed ofthe incident and refracted wave.
b)
refractive index at the deep to shallow water boundary.
(ii) If the angle ofincidence ofthe incident wave is 25, determine the angle of
refraction ofthe refracted wave.
(12 marks)
Total 20 marks
3(a)

(b)
(3 marks)

Describe an experiment to show that a metal expands when heated. State the
apparatus you would use, briefly describe your procedure and say how you
would arrive at your conclusions.
(8 marks)
Figure 2 shows a diagram representing the heating system or a domestic
electric iron. The temperature of the iron is regulated by a bimetallic strip. At
point C the bimetallic Strip makes contact with the live Wire of the supply.

(b)(i) Draw a displacement-line graph to represent the movement of a floating

buoy which has an amplitude ofO.5m as a water wave passes with a period

of3 seconds.

(ii) Can the graph you drew in part (bXi) be used to tell whether the wave is
transverse or longitudinal?
(5 marks)

(c)

The diagram in Figure 1 shows the wavefronts in a ripple tank as a water

wave moves from deep water to water made shallower by a sheet ofglass .

placed in the water.

,:::J.

Electrical
--",I,

Figure 2
(i) Use your knowledge ofthe thermal characteristics ofbimetallic strips to
explain how this system regulates the temperature ofthe iron.
(ll) Sketch (no numbers required just labelled axes and a shape) a graph showing
how the temperature of the iron would,vaIY with time. Indicate on your
graph the times when the heating coil is ON and OFF.

I
PHYS - P2 + P3 - JAN 2002 - Page 5 of 6

132

..

133
Figure 4 shows an apparatus which may be used to demonstrate the
phenomenon of magnetic levitation. A copper ring placed around a solenoid
can be suspended in space without apparent support by adjusting the a.c.
supply to the coil

Coil_lid 011
IIUIpetie

u. .
....,.,.

;.

~ .~.

Figure 4

Explain why a varying supply causes a current to flow in the ring.

At a given instant the a.c. supply voltage is increasing and the base ofthe

coil is a north pole. Figure 5 shows a sketch ofthe ring and the magnetic

field around it at that instant.

.....---Coil

(".
:t

...

FIlled COIIItac:t A
lYlUTaut_ ~Ntt;. I I

~AI'1IIl :unC

J::=:I

"-

~
U

l.e.

h
E=

( OlIladY

priaJD

Figun: 5
Redraw this diagram in your answer book. Show the direction of the induced
current on your diagram. Explain how you arrived at your answer.
(iii) On your diagram indicate the vertical forces acting on the ring when it is
suspended and in equilibrium. Give the nature and origin ofthese forces.

CoDE

:;-)

~plJ

Kiq

FIgUre 3

(8 marks)

PHYS - P2 + P3 - JAN 2002 - Page 6 of 6

(12mark:s)
Total 20 marks

END OF TEST

133

CARIBBEAN EXAMINATION COUNCIL - JanuaO" 2001

PHYSICS - PAPER 02 - GENERAL PROFIENCY - 1% Hr.

..... "

. . . . . . . . . . . . . . . . . . . . . . . . . . . . . . . . . . . . . . . . . . . . . . . . . . . . . . . . . . . . . ,u

JIo

134

"

"

"

"'."

"

t.

--.

"

'

"
;.~;;;;;-;;;-_.

"

"

"

(4mb)

1. You are to spend no more than Yi hr on this question.


In this question you will find the value of the resistance, R. of a heating element.
The voltage, V, across the heating element is varied and the power, P, which is dissipated

(c) Figure 1 shows the small jet plane flying at a constant speed and at a constant height.

is measured. (Note that R remains constant during the heating.) A graph of P against V is
plotted on page 4.
(a) Select 6 pairs of values ofP and V from the graph and insert these along with
corresDondimz: values of J11 in Table 1 below.
(8mb)
P(watts)

V (volts)
y: (voltsZ)
Table 1
(b) Plot a graph of P against

y: on graph paper.

Air reslstanee
)10

(9mb)

(c) Find the slope, S, ofthe graph you have plotted.

Figure 1
(i) In addition to the two forces shown on the diagram, two other forces, the weight

and the thrust ofthe engines, act on the plane. On Figure 1, draw TO THE SAME
SCALE, lines representing these other forces.

(4mb)

"*

(d) The equation for the heating element is P "" x VZ. Use this equation and your value
for S to find the resistance ofthe heating element.

.............

"

. . . . . . . . . . . . . . . . . . . . . . . . . . . . . . . . . . . . . . . . . . . . . . . . . . . . . . . . . . . . . . . . . . . . . . . . . . . . . . . . . . . . . . . . . . . . . . .jO

(3mb)
(ii) The engines now emit exhaust gases at a higher rate. Explain how this causes the

plane to accelerate.

(3mb)

(e) Estimate the power, Ph which would be dissipated in the element if the applied voltage
is 300 volts.

.........................

........................................

"'

,.

lIl

'"

(2mb)
(iii) The plane now circles at a new constant speed. Figure:1 shows one view ofthe

plane as it circles.

(3mb)
(f) If300 V were applied for 15 minutes, calculate the energy used in kWh.

(3mb)

Total 30 marks

2. This question concerns the motion of a small jet plane.


(a) The plane accelerates from rest along the runway at 3.4 m s 2 Ifit takes 18 seconds to
take off, at what speed does it leave the ground?
0

(3mb)

(b) State Newton's First and Third laws ofmotion.

Figure 2
Explain how the 'banking' (tilting to the horizontal) of the plane allows the lift to maintain
the plane's circular motion and its height.

:::::::::::::::::::::::::::::::::::::::::::::::::::::::::::::::::X::::::::::::::::::::::::::::::::::::::::::::::::::::::::::::::::::

PHYS - P2 + P3 - JAN 2001 - Page I of 6

(3mb)

13415 marks
Total

PIPa

135

3. (a) Draw a labelled diagram of a step-up transformer.

-400

-300

-200

-100

(b) Table 2 lists the main sources ofenergy loss in a real transformer. Complete Table 2.

100

200

300

4GO

9t'C

(2mb)
(ii) She now repeats the experiment with a different gas. Draw, on the same axes. a

MEmOD USED TO REDUCE

SOURCE OF ENERGY LOSS

Figure 3

ENERGY LOSS

graph she could obtain from the second experiment.

1. Heating in coil wires

(2mb)

-----

2. Eddy current losses in core

(iii) Explain how results like those she obtained led to the establishment of the Kelvb

3. Magnetisation and demagnetisation


ofcore

................................... "'

temperature scale.
~

"."'"

(lmk)

4. Imperfect flux linkage

(iv) What is the value in Kelvin of a Celsius temperature of2S0C?


(1mb)

Table 2

(b) (i) A bottle full of air is corked at atmospheric pressure of 1.00 x 10 5 Pa and a
temperature of2S0C. The bottle is heated. Calculate the temperature at which the
pressure ofthe air will reach 2.5 x 10 5 Pa. Assume the volume ofthe bottle does

(4mks)

(c) Explain why alternating current (a.c.) is preferred to direct current (d.c.) For the
transmission ofelectrical energy over long distances.

not change.

(3mb)

(3mb)

(d) (i) A StepMUP transformer bas 66 000 turns on the secondary coil and 16500 turns on
the primary coil. Calculate the primary voltage when the secondary voltage is 440 V.

(3mks)
(ii) Determine the CI1lTel1t in the primary when the secondary is delivering energy at a
rate of26.4 kW.
. . . . . . . . . . -4 ,

..............*

, .,

or."

"'''.''' ."'

"

(3mb)
Total 15 marks
4. (a) (i) A scientist conducts an experiment where she measures 11 series of values of
pressure, P, and temperature, 0, of a gas at a constant volume. On the axes shown
in Figure 3, sketch the graph she would obtain from plotting her results.

(ti) In terms of the kinetic theory of gases, explain the origin ofthe pressure inside the

bottle.

(3mb)
(iii) State whether the following quantities increase, decrease or stay the same when the

bottle is heated:
(a) The speed of the gas particles
(b) The average space between the particles
(c) The number ofgas particles

..
..
..

.'"

(3mb)
Total 16 marts

5. (a) A transverse seismic (earthquake) wave travels with a speed of 5 000 m S1 and cause:
the ground to vibrate up and down. The graph in Figure 4 shows the variation,. with

time, of the displacement ofthe ground at a point, as the wave passes.


135

PHYS - P2 + P3 - JAN 2001 - Page:% of 6

136

Vertical

displacement 1m

-mnmTf

Flgnre4
(i) Calculate the frequency ofthe wave.

.
.

...... ,"' "


..................................................................................................
........................................................................................................................
,
,

(3mb)

(ii) Calculate the wavelength of the wave.

.......................................................................................................................................

"

"

. . . . . . . . . . . . . . . . . . . . . . . . . . . . . . . . . . . . . . . . . . . . . . . . . . . . . . . . . . . . . . . . . . . . . . . . . . . . . . . . . . . . . . . . . . . . . . . . . . . . . . . . . . . . . . . . . . . . . . . . '111

(3mb)
(iii) On the graph, draw two small crosses at the times at which the ground is moving at

the greatest rate. Explain your choice ofpoints.


. . . . . . . . . . . . . . . . . . . . . . . . . . . . . . . lI' lI'
..

.
111

(3mb)

(iv) Write down a time which the ground is stationary

..................................................................................................................

,,

(lmk)
(v) On the axes shown in Figure S. draw a graph to represent a seismic wave ofhalf
the amplitude and twice the frequency ofthe wave shown in Fignre 4.
Vertlcal
cll.splaoemeDtIm

II

.co

-v /volt

Voltage

time/s

Figure 5
(4 marks)
Total 14 marks

PHYS - P2 + P3 - JAN 2001 - Page 3 of 6

136

CARIBBEAN EXAMINATION COUNCIL - January 2001

PHYSICS - PAPER 03 - GENERAL PROFIENCY - 1 Hr.

Answer any THREE questiolls


<.

1. (a)
o'

(b)

2. (a)

(i)

Define the term "refractive index' and give TWO examples of observations which show
that light can be refracted.
( 3 marks)

137

What is

Oi)

an alpha particle?
(2 marks)

a beta particle'!

(i)

Draw a diagram, showing how a prism can split a beam of white light into its
your diagram the relative positions of red and violet
constituent colours.
should be indicated.

(b)

Describe an experiment to compare the ranges of alpha-particles and beta-particles in air.


(6 marks)

(ii)

Explain why a converging lens is often placed in the path of the diverging
beam emerging from the prism.

(c)

Radioactive isotopes have medical uses and cancer can be treated by radiatior

On

(iii)

therapy. A cobalt 60 G~Co) nuclidedc;cays to an isotope ofnickel (Ni), by emitting a beta


particle and gamma rays.

Use your answer to (b) (i) above to deduce how the wavelength oflight is related
to the refractive index.
( 7 marks)

(i)

(c)

(ii)

State which of the radiations emitted is suitable for treating cancer and explair
why.

(iii)

After 15.6 years, 7/8 ofthe sample ofcobalt 60 has decayed. Determine the half
life of cobalt 60.
(9 marks:

(d)

-------- ~I!I
....

Represent the decay of cobalt 60 by a nuclear reaction.

Iodine i31 is a radioactive isotope that is used as a tracer and taken in food by a patient
It has a half-life of 8 days. Explain why this is a suitable half-life for its use as a tracer
(3 marks:

Total 20 marks

Metne

3. (a)

( 3 marks

State the unit of energy and the law of conservation of energy.

rule

(b)

For the following processes. state the main energy conversions involved:
(i)
(li)

A burning candle
A car travelling at constant speed

Figure 1
(iii)

Figure 1 shows an experiment in which a beam of laser light is initially incident at A on


a metre rule. A transparent slab is then placed in the path of the beam so that the angle
of incidence on the slab is 60. The beam emerges at an angle, e, to the normaJ to the slab.
(i)

(ii)

(c)

The generation of hydro--electricity

( Smarks

A ball of mass 0.5 kg is dropped from a height of 10 m onto a hard'surface. The bal
rebounds to one-half its original height. Neglecting air resistance, determine the

Calculate the angle of refraction in the slab and the vaJue of e if the slab is made
",,"of glass.

(Refractive index of glass = 1.5)

(i)

speed of the ball at the instan~ it hits the surface

(ii)

amount of energy lost on impact with the surface

Yis the distance betWeen the positron ofiJie undeviated beam and the position of

(iii)

speed of the ball as it rebounds from the surface

(iv)

speed and acceleration of the ball at the highest point it reaches after rebounding.
(The acceleration due to gravity, g := 10 m s-2)

the deviated beam. State and explain the effect on y of using


. (a)

(b}

a slab with a higher refractive index


laser light of longer wavelength.

(lZmarks.
(10 marks)
Total 20 marks PHYS P2 + P3 - JAN 2001 - Page 4 of 6

Total 20 marks
137

(iii)
4 (a)

Describe, with the aid ofa diagram, an experiment to compare the thennal conductivities
of different metals.
(5 marks)

(b)

Arrange the following materials in order of their thermal conductivity, starting with the
best conductor: wood, copper, styrofoam, iron.
(3 marks)

(c)

(i)

138

Water.t
100C

A circular section is removed from a flat square sheet of metal as shown in


Figure 2.
.

Figure 4
A metal rod of length I m is placed in contact with metal tins, one containing
boiling water and the other ice as shown in Figure 4. Using the same axes, sketch
graphs of temperature against d.istance along the bar if the bar is
a)

perfectly insulated

b)

not insulated.

(12 marks)

Total 20 marks
5. (a)

.Figure 2
If the sheet is now uniformly heated, what happens to the diameter of the hole
and the length of a side of the square? Explain your answer.
(ii)

(2 marks)

Define the term 'electric field' .

(b)

Explain how tiny droplets of water vapour in a cloud could acquire a negative charge in
(3 marks)
air.

(c)

Lightning occurs when a cloud develops a large negative charge causing the air between
ClOUd and ground to conduct electricity.

Figure 3 shows a bimetallic strip at room temperature which is clamped


horizontally.

~~Llgbtnlng

stroke

Ice

...

'\

I '

Figure 3

a)

State and explain what happens to the bimetallic strip if a piece of ice is
placed on it as shown.

b)

The ice is removed and the strip returns to room temperature. State what
happens to the strip now.
.

PHYS - P2 + P3 - JAN 2001 - Page 5 of 6

+
+

Arrestor

+ +

+
+

+
+

+
+

Copper
~k

Ground

Figure 5
t

Figure 5 shows a lightning stroke from a cloud to a lightning arrestor on top ofa building.

138the air and give


Identify the charge carriers involved in the passage of current through
the directions in which they move.
(3 marks)
/.L

....

(d)

Figure 6 is a series circuit which shows the flow of current through the air, arrestor,
cable system and ground.

139

11.

R.

-L...

vI.

Re

R,

Figure 6
Using the following data:

=3

potential difference between doud and ground

VI.

=5

If.

average current during lightning stroke

Rc

resistance of lightning arrestor system

R,

resistarn:eof conducting channel in ground = 15 Q

x 107 V

x 10" A

= O.OS Q

(i)

Determine the total resistance in the series circuit. Hence find, Ra the resistance

of the conducting channel in air.

(ii)

If the lightning stroke lasts for 3 x 10-5 seconds, detennine the amount ofcharge
that is transferred. Determine also the electrical energy of the stroke.

(12 marks)

Total 20 marks

END OF TEST

..
PHYS - P2 + P3 - JAN 2001 - Page 6 of 6

139
b~

140

Jan~ary 2000

Physics

Paper 2

(b)

Use these values to plot a graph of d against f on pageS.

(c)

Find the slope, S, of the graph you have plotted.

(9marks)_

Time: 1Yz hrs

Instructions:

I.

You must use this anSY.'er booklet when responding to the questions. For each
question. write your ans'wer in the space provided and return the answer
booklet at the end ofthe examination.

(4 marks)

(d)

2.

ALL WORKING MUST BE SHOWN in this booklet. since marks will be


awardedfor correct steps in calculations.

3.

AttemptALL questions.

4.

The use ofnon-programmable calculators is allowed

5.

Mathematical tables are provided.

l.

No more than half an hour should be spent on this question.

Given that S = gm/2, find the value of g". ,

(2 marks)

(e)

(3 marks)
(f)

DATA ANAL\,SIS

Use the graph on page 2 to find the average speed during the first four seconds.

Using the average speed calculated in part (e) above, or otherwise, find the
speed of the object after it has fallen for 4 seconds.

In this question you are to find the value of the acceleration due to gravity on the Moon,
and use your value in calculations.

g".

(a)

On page 2 is a graph of distance. d. against time. 1. for an object falling from

(3 marks)

rest on the Moon. From the graph. obtain six pairs of values of d and 1. Tabulate
these below along with the corresponding values oft",
(9 marks)

Total 30 marks
-. 'I

(a)

Explain what is meant by the 'dispersion' oflight.

(1 mark)

PHYS - P2 + P3 :- JAN 2000.- Page 1 of 7

140

.....

141
Complete the truth table for the following logic gates.

(ii)

What would be the value ofx on~the Kelvinscaleoftemperature

OR

NAND
INPUTS
I B
0
I
0
I
I
I

A
0
0

I
I

OUTPUT

j
f

. INPUTS
A
B
0
0
I
0
0
I
I
I

OUTPUT

(2 marks)

(iii)

If the temperature of a body changes by 40 what would the


temperature change be in kelvin?

i
(I mark)

14 marks)
Total) 5 marks
(c)

4.
(a)

What is meant by the 'upper' and 'lower fixed points' of the Celsius
temperature scale?

(2 marks)
(b)

Figure 2 below shows the position of the meniscus in a mercury thermometer


at oec, at 100 at an tmknown temperature. X.

Figure 3
(i)

Water is heated at the top ofa test-tube as shown in Figure 3 above.


State what would be observed and explain these observations.

(3 marks)
(ij)
a'l:

..111ft:

lCe

In a refrigerator, the cooling element is at the top. Explain how the.


cooling element is able to cool all the contents of the refrigerator.

Figure 2
(i)

Use the information in the diagram to find the temperature. X.


(3 marks)

Total 15 marks

(4 marks)

PHYS - P2 + P3 - JAN 2000 - Page 2 of 7

141

10

142

3.

(b)

(a)

(i)

A 3 V source of e.m.f. consists of two identicaU.5 V dry cells. Draw


(2 marks)
the symbol for this source.

(ii)

The source is in a series circuit with a switch, an ammeter, a

Princpal

-~,

axis

Ghwpri$lll

Fi"".. 1

(i)

semiconductor diode and a 120 resistor. The diode is placed so that


it has zero effective resistance. Draw a circuit diagram, indicating the
direction of current flow when the switch is closed and the polarity of
the ammeter.
(4 marks)

A narrow beam of white light strikes one of two identical glass prisms
as shown in Figure 1 above. Complete the diagram to show the path
taken by the white light. Indicate where the red and blue light strike the
(4 marks).,
principal axis.
Monochromatic blue light strikes the other prism as ShO\\<l1. Complete
the path of the blue light.
(3 marks)

(iii)

The refractive index of glass for blue I ight is 1.55 and that of red light
1.51. Which colour light travels more slowly in glass?

(1 mark)

(iv)

(iii)

Calculate the magnitude of the current.

Find the wavelength of red light in glass ifits wavelength in air is

6.5 x IO1 m.
(3 marks)

(bl
(v)

(i)

IdentitY the logic gate symbols by completing the table below.

(3 marks)
Calculate the frequency of red light in air and its frequency in glass.

I
!I

(4 marks)

(speed of red light in air

3.0 x lOs m s 1

Total 16 marks

.. PHYS~ P2 + P3 - JAN 2000 - Page 3 of 7

NAME OF

SYMBOL

GATE

=D

=Dr

(2 marks)

142

5.

143

A motorist, travelling in a car at a constant speed of 30 m SI on a local


highway, overtakes a policeman on a motorcycle travelling at 20 m SI. The
policeman immediately gives chase and the motion of the vehicles, for the next
10 seconds, is represented on the velocity time graph below.

(iii)

Distance travelled by the tTiotorcyclein the 10 seconds

(3 marks)
Veilldly IlIIlI -I

(d)

Comment on your answers to (c) (ii) and (c) (iii) above.

so
/lICllnrc)'dC'

40

30 I

:>'

(1 mark)

I Car

.e=:

(e)

20

The motorist is arrested by the policeman for exceeding the 100 km h-I speed
limit. He says he is innocent since he had only been driving for 10 minutes and
therefore could not have travelled ]00 km. Explain the motorist's
misunderstanding.

10

10

11 Tln;r I!I

(2 marks)

(a)

Explain the meaning of the term 'constant speed'.


Total J4 marks

(b)

(I mark)
In terms of the forces acting. explain how the car maintains a constant speed.

END OF TEST

(2 marks)
(c)

From the graph calculate the:


(i)

Acceleration of the motorcycle

(3 J1larks)
(ii)

Distance travelled by the car in the 10 seconds

(2 marks)

PHYS - P2 + P3 JAN 2000 - Page 4 of 7

143
(0 \

144

Graph ~DIs&anceIUme Cor object [anint on Moon

Olsl.:lnce 1m

-+_ ...

=t=:t: :

l--

o.._-l...

-h-
~-',

--

-:1::

16

14

!=:.

u r-- '

-:::1

..

kF

+::=./.....---1===

l - _...

t-
i-.

..,

::"

:;

,
~

.....,
":

-:E:::E
---;......-...

---,
u_

-=1~E

c=:..
-,

r
,-~--~'

r
,C;;;;;;:;=:

r-"':

4~~~

t=:=:-'
---l
,:t:==
. -
-_." .

....

'

,~.

~....:..;.....-

.,.......,.....

...--..-1-..

PHYS - P2 + P3 - JAN 2000 - Page 5 of 7

5 Tlme/s

144

TESTCODE

,FORM TP 2028

002493

JANUAR.Y 2000

I.

Describe an experiment to demonstrate-the interference-of sound. In youraccount;-


describe the apparatlis you would use. the observations you would need to make and
explain these observations.
( 8 marks)

(b)

Figure I below is a displacement-time graph which represents a sound wave passing a


certain point in air.

CARlBBEAN EXAMINATIONS COUNCIL

SECONDARY EDUCATION CERTIFJCATE

EXAMINATION

145

(a)

PHYSICS

Paper 03 - General Proficiency

Dlsplament Imm

I hour

Cto JANt1~~ (iii.)


TIme/ms

Figure 1

In addition to the 1 hour. candidates are allowed a


reading time of 10 minutes. Writing may begin
during the IO-minute period.

Deduce the frequency of the sound wave and calculate ils wavelength given that the speed
of sound in air is 330 m 5- 1:
( 5 marks)
tC)

READ TilE FOLLOWING DIRECTIONS CAREFULLY

1.

Answer ANY THREE questions.

2.

ALL WORKING MUST BE SHOWN in your answer booklet. since marks will be awarded for
correct steps in calculations.

3.

The use of non-programmable calculators is allowed.

4.

Mathematical tables are provided.

A source of sound is placed under water. where the speed of sound is 1400 m s ,-I as in
Figure-2 below. Calculate the refractive index for sound passing from water to air, and use
your-result to calculate the angle of refraction in the air for a sound wave that strikes the
water/air boundary at an angle of in~idence of 40. Copy the diagram'into your answer
booklet and sketch the path of the sound wave.
( 7 marks)

Air

40"

Water

Flgur;e2

Total 20 marks

PHYS - P2 + P3 - JAN 2000 - Page 6 of 7

145
Vi1-

1.

(a)

The Geiger-Marsden experiment (alpha-particle $Callering experiment) supported a

nuclear model of the atom over the model that had been previously accepted. Briefly
describe the model which was accepled at the time of this experiment. Outline the
Geiger-Marsden experiment and show how the observations support the nuclear model.
( 8 marks)
(b)

(i)

5.

(a)

:Explain the difference between a motor and a genel1ltor in terms of the energy
conversions involved.
146 ( 2marks)

(b)

FlJUn 3 below shows a simple d.c. motor connected in a circuit.

CoD

A radioactive isotope of lead. Pb214. emits beta-particles which a rall:mcter

detects. In four consecutive periods. the number of counts recorded by the rate
meter were: 4823. 4764. 4829 and 4784. Explain why the count rate is not
constant and find the average count.
(ii)

(c)

In a similarset of mea~urements50 minutes later. the counts recorded were 1216.


1169. 1225 and 1190. Find the aver:lge count allhis time and use the two values
( 6 marks)
you have calculated to lind the halflife of the source.

Brush

The proton number of lead is 82 and the product of the betadecay of Pb-214 is bismuth
CBi). Write down the nuclear reaction for this decay. Write down the symbol for another
isotope of lead and state the number of electrons a neutral atom of this isotope would
contain.
( 6 marks)

FiJUre 3
Explainwhy. when the switch. S. is closed. the coil rotates continuously in one direction.
State the direction of rotation.

Total 20 marks
3.

(a)

(b)

Identify a modification that is needed in converting .the d.c motor in FlJUre 3 to an B.C
generator.
( 8 marks)

Use the kinetic theory of gases to explain


(i)

why particles of smoke in air are seen to move in a random manner when viewed
through a microscope

(ii)

how the air in a tyre exerts a pressure on the tyre wall and why the pressure
( 8 marks)
increases when the temperature of the air increases.

The air in EACH of the four tyres of a pick-up truck exerts a pressure of 2.5 x IO~
N m2 The area of each tyre in contact with the ground is 225 cm~ and the pressure of
the air in the tyres leads to a force which supports the weight of the truck. Calculaie the
mass of the truck; (g 10m S-2)
( 7 marks)

(c)

An electric motor is operating on a current of 2.5 A al a p.d. of 240 V. It is connected


by a long cable to the generator. If the potential difference a,.t Ihe terminals of the
generatoris250V.detennine the resistance of the cable andthe lossofpowerinthecable.
What is the power supplied by the generator?
(JOmarks)

Total 20 marks
END OF TEST

(c)

What effect would an increase in the temperature of the air in the tyres have on the area
of contact between the tyres and the ground? Write down TWO assumptions you have
made in your answer.
(Smarks)

Total 10 marks
4.

(a)

(b)

Define linear momentum. state the law ofconservation ofli near momentum and describe
( 8 marks)
an experiment to demonstrate this law.

0)

A spacecraft accelerates by ejecting gases at high speed from its rockets. Use the
law of conservation of momentum to explain how this results in an increase of
speed of the spacecraft.

(ii)

A spacecraft. of mass 8 000 kg. is travelling at a constant speed whenilsrockets


are turned on. The gases are emilled at a rate of 1.5 kg S-I and are moving
at a speed of 1100 m 5- 1. Find the change in momentum per second of the gas.
Calculate the increase in velocity per second ofthe spacecraft and the total change
in velocity iftbe rockets are fired for J5 seconds. The final speed of the spacecl1lft
is 1200 m S-I. What distance will the spacecraft travel in one week?

(iii)

A second spacecraft is in orbit at a cons!antspeed around the Earth. Dl1lwasketch


to show the force or forces on this spacecraft.
(12 marks)
...."""'. 2ft ..........

PHYS - P2 + P3 - JAN 2000 - Page 7 of 7

146

..

147
l.

PHYSICS PAPER 2 - JANUARY 1999


General Proficiency - 1~ fIrs

No more titan

i hour should be spent on this question.


nATA

ANAI~VSIS

When a driver of a car sees an obstruction in the road the total stopping distance for'the car is
made up of two distances. the thinking distance and the braking distance.

Thinlllnll:

t.:::..t:.t++tl=4+;:t:Mrtftfm:H:iti:7iti

DiAanc"

dIm

nL~

..... 1

TIle, thinking distance: The distance the car travels in the time it takes the driver to make the
decision to stop.
The braking distance: The distance the car takes to stop While the "rakes are applied.

.,
IH'
~ ~ ~ ~ ~ ..
. 1 ...
! r~ ~
~

r:

j iI:

'l

i ~

.. 4

Data on these quantities at different speeds are shown in Table l.

! .. ~-+
. ~ ii

TAnLE 1
CAR
SPEEI> '
(vIm s -I)

'I

p,

22

201

'I"~

:
.1,. " :.......
"
~

l'tt
'

,
+~

,"

24.8
36.0

20

28.8

Pi: iiU ~E~

48.8

28

24

39.2

32

28

63.2
79.2

36

32

51.2
64.8

!.. .i ~ '" t ..
.! ~ i'
! i ~ r i I t 1 t r. 1
i ...

~ ......

:~;:

7::1

-4'r

I ~ H
t-'i r ,;t-~ !~L .;:1

ttti

it;!

t;\:

r~ ~

,,

~ ~~

"
........

(a)

.,....

96.8

A graph of thinking distance. d. against car speed, v, has heen drawn for you lin
page 2. Detennine the gradient. G. ofthis graph. and give its unit.

:~:
,

I ~ : ,

':7,

".

",

. "I.

, ;I

I;

I','

......

..

I : ~

I',

: "

Ol'

I;:

"

.. ;

T"

,I,
"fO!'t
~

,~ .

i ~ 7;

i.;;

4 marks)

;, ;

(b)

, I
;

Use the equation. v

=Gd + 4. to find the value of d when v is 40 m s'\.

... i

I
1;1

;,

~,.
J ~ t ..
I . 1 ..

"I"
;.l

't"

"

ii:" :~ 1~ :!

16

......

... ..

2""

;'"
;.1,
f- !

r: :

~ ;

~ ~ ;~

:L: .,'
,i;.

"

.....

, J .J

., 1

12.8
20.0

16,

~,.

;: ~ :

12

41

(slm)

:~;!

"

14

,I'

(x/ml

..
...~; i , .

II '
.,

18. l;;:

(dim)

12

~
~

t~!
1
.. i
...

I . . , .1

TOTAL STOI'I'ING
DISTANCE

16
~ ~
~_J
.

ri~r'i'p"PI

nRAKIN{;
1>1 STANCE

20
24

:z6 I'".,..i

THINKING
IJISTANCE

18

20

"

" ~..

t:

"
11;"f

"<IIi
, .'.
~

., ..

L '"

,q; 1

1 ..

" "'
!i"':

:z6

2B

30

1f

31

34

36

(3 marks)

CarSpftcl rIms"

Graph for Question 1

(cl

Plot a graph on page 5 of total stopping distance. s. against car speed. v. The gr~ph line
is a curve.
(9 marks)

PHYS - P2 + P3 - JAN 1999 - Pagel of 8


\.

147

~?

148

" .I"
<,

Use the graph of s against v to find the total stopping disulnce if an obstnlction is seen
when the speed of the car is 30 m s -I ,

I"

I. "'. ." 1' ~. ~I' .', ". '.' ',',..


I I''. ',,.,..(..'I',.'
I
t

l t, U 1.......
r-.. . t r,.1"1
;.
'fl'+
~I
..~ !t !'I, ,to.i, ." ~"
f.1~t ~"'!"L+~
,-.I. r ,. 't'.' ,4:"
'.'"
.tt
l!l.'
t::t.
4:":~t J_
i '
'I't
.
;1+;..
+1'
'1"
fl'~l
'
r
fol'
r
;.
t.t.
,;.
.~f'"
. . .. t .~.i .... ~.' L .;.1
t-:
Z,~;'! :r;
.U~ ttl)' 1"1:1 ~1; ii1t 1~~ ~!. ~ r~tt
~tl.i. l~i~ f";~f tf..,-'!; r,fi t ...... ~ tf11
;f,
t, .i~
!r1~ :'I"i r~t~ ,t-t tH40 .~. ~ ....,,, rl.- ~-t~
'1'
~'itl
.t;t t,~j ;f~t H~; i!tt ti'; ~tt~ :t,l it1 ;+'1 1 !"~!;

'i

Q.u, ~ - ~Mt4Cd,

(d)

.. !,
f fi

<'f ""'.

If-.J-t -t.....,.~

'ir!lt"

i.

,t.. .

(2Jilarks)

t,'

tS"'~

tl~t

'r'.j -rn'
'tiM t~~r-t''
.i-fr- t'rt

~f

;~~'"

t"!'

1.:11 :ql .tHl-P hP ItJ:

~-n;

j-d 'tt, KHd tHI' H-t'


I J' I'll
,.--i~
' .! .. 'i
~1
'I t , . t I"
, ..J.--4.
-+rrr"lfl!:t
,t.'" t .~:H-t
'....
-L ii--!
. ~ ITt' f " f oj
t-.
J ~H Itt! llti rrJ:t :r~'tt fi.1:! rtftll 1- 1' til t; I:
-~,.Il'i~' Il~ +11- 'U:.!.. -Itll.
,I..! I". li.'
-I-f
.1-4' 1- 1
'11' ,
4m-t4 H.. r
1+1 4,1. "t'
'Ht t l'. Jft
',' l'Jtr 'tl "-Ht
,..;1.
!1'1
~ t-+
t-t1. ""!
.'" ..;.~tl
-t- !t"
. " .I"l'~
1
r -rr
t '-t +I!:t . r- I .+,-.
Ht ./..,..
i- I
I J t"t. ",., ..-"

t+-';' 1;:" hH

(e)

If the mass afthe car is 8ookg, calculate the kinetic energy oftlle car when it is travelling

at24ms- I ,

I ,,",
. ~...,+
Hp
! ..... ,

1 ,-

kl"~ 4 ' t H

~."~ ... H,4+J,.

l..f,~"t"

~"-4+t-~

-t-~r

10

,"''''

,t.!

4 ..

.-

!,!

(f)

The average force exened by the brakes is 7 000 N, Determine how much work is done
by the brakes to stop the car when the car is initially traveUing at 24 m s -I.

~, .. t '-h
1,.;, r~.r

f-"

!i~! '

1:;

ir~f t.,U: t+.l". trtt ~.l1~ Lttt


~'It hti tiT! ttl.; ,ltl I t h
!"",,..... t "1-~,.'r"t"~tt't!' ~t"'+ ...' +-rt r'

Explain wby the answers to (e) and (f) are not the same.

~Mf. f-"'+'~

t.1

If an extra load of 160 kg was put in the car, find the deceleration that the application of

~#'J't

~~~

; . \.-~ .,;,

~~

'''' t.:
,

,'Of. ,. ... ~f-

"

"1"

...,. .,..
l

:t'"

I t

,;:;

j1~l

~, ..

,I

.,
..,

jJ'~

..

t:! t

, ..

'1'"
..

..

:r:.t .t;:
'., '.-.

i;.:.

I,HI
,.1., ,.,'
.,-It- I~-fi .. , ..

, ........

. , '.f ~

J I , .. l.
I" . ,1.1.,1,
I.,.

~ . " ........

, - ... , .

~ 1 r~~ ~f:'1

' tf,- i l
~ .-,
J r ..
I f '",',
f"

lt

t"j
1r ~
1_
L.
;!: ~ ;;! ~ : ~ ;!

t '

,., .. , ...

'I' "'--rmj !-rt: ~~+


i '1"
;,
~ f ~.;
, , .,f.

~~ . . . .-..

, . ,l~t

'j<

'.fl .,.I, .
,; -. ....
'1-"

i.fl'
't
'I'll
, !I't
t
f.
! ~ 1"
_ f "
f"'

..

-i,~~

+"'r~

tlt '.,

'r'~

or'! it!.

<l~"

... rt';*

,! ,.."
~,~;.
't'
"~.'
.,,+,
~..
l
,:-I-t
q I.
t ItJ Hil HI
I: 1! '1 It !
.. , ' .. - I,l , H '

LC.,

.. mi~-t ~-~.

.. f

~~f' ~~ ..

tit~

~r~t

.:

~-..

i~.

1~,1

~;;:

:!jt

tJr.

f~

f~; ~. "'j

;!;!
t,:::
.t !;,;
t ~ ......

t-

r'f

iJ;.

',', .~,. III jl!,J ' i ' ! ,I"~


..
"-1 'T" t"'~.! i.,. "'.1
.,f
t -- t~tr ~:'f:j '1~';' rtt~ ~~i! ~t.1; ~~:~ :;!i 1~:~ ;i;,~' ~~i~ ;;~l :!:' ~:.: ::.:
f~."

~tf

tt.,
J!".i

t t
;.~;.

tt:tt

,~.<
t!"-t~

~'t'

, ....

'7'1""

.~~'

't,

....... t--J

'~l,

;:,:

t,

"ii'

".4
lr-~"

11;::
t.:
~l .. !r': - "..... ~
i~=, ....
,I tr

.i~"J

!lI'

... 1:

.itt~

':I~

t,'

~~_~
l

\.t

'

..

,":"

.f.~

f.tt!

,!
t-t

" ,

'I;

$;l.

't:*~

~ .....

'~11

't l

lot.

I~'I' !f~t

Li

!:-J

....

i;U t~!! ~~t-~ !;1"~ ::.~~ ;~.!; t:: t,!


~r~'t;
,~~I! ri:~ l:r ~r~i ~~!~ ti~.; ~!l

;! ;.;. ~~;;.;I ' '1"


t!~"! :ir~; ~ ~.::
.l,
" t 1'
l:' ft--!
,t,-+
,",4-,
'''+-1'' 'r!' .. '"
,.,'1"11'1 II r'I 1'1".1.'
-'r'f+l'1:-'-fafTl1l
',
"J :'1
'-1'1"-1t" r'l'llltIl"J I'..iT,tt tj'
it"i.i '"
"J'
t!t .;1

the brakes would cause.

~+-t -,L... .i..~

~:~~ I~t~ "~l,~ ~;!;- if'


~:'tl tiP
tt~r'r;!
;1,
f f"f" [til t!,t ,t,'.
_,J-", i"!,. ~ .....
,.'.

(b)

i~4..

d~

1;""':

f;"

f-:'!l-

o. ...

( I mack )

It~

'1!1!
~~~-1 .~~~ r!l.~1 ~~.iJ 1;1~
1 ".1 ffh -,',:
-I ' t':JI
L f. ~l 1fl.f
~;:.r
f.t~1
t . 1''-'. ,......

"t, .+41.
'ttl'
il ' 1'-1 '[h t'-,:.
lot"
~,
. . , -t~+~
~ , .. T t. t ; '
:..... t:'" ~
'.,i+f !-.Lf!
iI..,r- L,!.; '. '1;
" ,~.,
.,.~.. ! ..
; r ~ f t ~ ~; ~ t"N'! It!; ; f t-t 'i t!. t)' : i r ~ ~: i;
,.. ~t .,.... + 1'-. -f ,-.!" ! t f , .. t
r,t'j~ It l ., i~~'
.. ~""
.~f't . f - rttf- ci.J< .. ft. !'t,' ...~! r-.~'
<'1l
r"!" .Lriy' I'd ~I" .Ii' H" ' d i,.
r~f.
~.tl:t~... t'M' t',4 IJw'"
rt~ .. ';1 t:
'1!

(g)

t'"
'f"

"",;t... : J.......
.J.t.

1:1.. ;;. Pf~

'f:J-.

.. . !

, . ,.
q"-.

;~:~ ,~;l: :~ ..: ~t_~': ;!:!

,..

no>

" -.

'.<- .. ,., ', ,

-li-". -f,j, 1-1.1 h,j .-;.', ,' . '-II" HIi


1 . "1'
I', t'P "-++ trtl H"t ..H.,... -t I .I r ' ,I.J 1,.-., " .. '.," I',. II" .,.1 ';1
;~,;,~ -y~:I' l~it ... 1..1 !~'~. ~~t~ t-1.~l ,ifl~ t!.t! !,!~ :z~! I:!~ 'Ot,! ~.~~: ~~~t f~i:~

;,l;l t;::
';I'~ . t~t

( Jmarks)

,,,..

rtf" ly-J:

:~:t t~l: tI~l ~~:~ :::~

;1

" ' " -I-H ,.;,

( J macks)

~ - ,,,

,lt.'
-,..+

I,;, I ., l i t . 'j.!~
,, ....
', .i.' ."to.
1 'I"
':'
...
l 1

Hf'~f

,f

,-_.1,
..

~I:!

I l

,I!. '

__ ~tf_ I .. t~
j,
.
I~
tL!,J

~ . "1'
'
,
~i
:i~: ::1: ~:!I!i

,;-i;'
!t.~

""

'I
l'

".
_f;

.t.!

t,

',.

;11f ~i"~ 4ti


ItiI
'jll JI.
t-,T ,-t
t,

"

.'::

....

; : . , " .

~;;! !~!~ ~L:


~i:i i~: .:,..
f.~!

J.i

.:.

.. ':'- ....

;-~

~;:: ;"_;t :::

I.',

....

, ..

I';..,. HI'.II l-ll'...,.,.,1,.,


f'.''1'.... ,
0>.. ;...1""

1"!'

ill

ff

!tt-'-t

!t-~t fL. ! t-.+-~ f.!.i; ;l;~ i~t~ ;~r'; 4*.!.. tt::"! ;i'i :~i; ~~rt "ltL~ il.~! ~;t: 'l;i :.~.:
li .. ~ ~.~ ... 1;' f-t4f i~TirTT ~ r;., .tTI-i -"-;-~t ~lm~ J.t~
I~t; ~tf!- f:l.-f l,mI. ':1
f
t! I I'. I' 'rtjl tn' . to' I j ; . . !.! I i I r' r! 'I ' ,..
~'l' d
. l. t i. I.
1~t4 ;~t--~ .!.u.. tt.~ ~~;~ if.~ i;l t~,t~ ,+t~~ !.~; ;:" ;~tl ft+~ ~i.~' t~t~ ~~~ .!i: ~:~:
'.'

. 1. j'1"

....
'.:U
!'r t -~ r' .I-I"~.j
II;
I'"~
" .
rJ
f'~"
,.... t >1'.J.
f, t. If>!
t-' I I!"
'l'

J:,
,
I!
J"'-1
t-ll "I
t f ~.l J.tt"
M f ""1'
t ";.1
"".
. , ...
iI '!J t , J't'p ;t, t~l'! ! . . . . . . 't. t " t 1'1 : ! " , .. ! .. 4f~
'~f:
,.1':
.",.,.
i

( J marks)

t-r

""t"!"t

t--t-+... ~ .........,

~'.

!,It''

t-

.!: ;i:i
,.,*

;ttI

...

Ii

(i)

Write down the main energy change that is occurring when a car is
(i)

~.

...... ~'

,'1 '1:;-1-, T.j.r I.H~


'"
j ;'j
,;
fH' . (Tf 'l-~
"1 "I .~ I q tit i I
I'" IT' I.' If' t ! I ' j t, I'
i,.i t ~r~T ~~:tt t~:.t t~
l:':
J,!j :f".!
~ t.. ~ ; ....;. .. +J. to.... , ...... j. r ;.: t ~ i,;: ;
!
t .... ~
j. l I
..
,... ..
, ,. t -h
.f.. !.!. .l_: ..
. 1'. I , " "'. f ~l ..
'" .. .. Ii."o. : . 1 , .. +: ... "I
t J 1'. ~.. L..... . l . : I' t
rot- ~r
'r'i I
-'i.1 .~f..j.,t "'.1,t i : t ' .. ' . l~t-' .t t f--r"
~: I q ~ ~ I' '. !' If; .. t to. t ! ~ t t .l. I J j t 1-! ~ t ... , 40
!-~~ j t ' "
1'"
1'" f t> . l . .. ~ "1:1
Jr;
..... ,..4 1 .. J,t +.-f
-"
" ' 1 I ,. ' I l l , .
j, 'I" ! t ,
I., l . l l
~ J
"
H,~.-t .. f-',a. ,L.f
t'-!.! ~t:-t t .f! :f-"H r~1' rtt"
, I"
-.*,.

"t

t'. t l.

.H~

1..1,

If"

t-d

I H ..
'1 ~,
I,
r'
;., ....,

f ~I't ~

I
4.:'
-' 1
,~

!-f."L

~t'

~,;,

Tj:iT 'I'~' , ., .. '.


H f' !.~, 'I ,! r '

~,-r~i i!!f .. "

1ft ~

t~ I

t"~ 'L! tr I:
,f
, ','"
",,.t

, t "1 ~

J. ~

...I"!'.

,.
~ r -,;. ,-"
~ ; :
! ~ t
~ f '"
t'''' t~,
~,,,.

., ~
tt. I :~f11
h ~ ~,t '. ~ .. ! .... r :
~t':
Iff' ~:f. ,4;f . , ...
I .... t,"I
'Ij
1'- "'1' .:" .. -.
f ,.,
"rif
't t f f' ' j , t><. J'I'l,
,'I!
I
hi"~
,',;
/"1:
"I'
'1'"
..
1-' It"
f"
...
11 t t . ....
+to. Ii~
.a.
'f 1~!"
.J~ .. J~, t.~
f

.. ,

,.t

1:"1;
I i ' I rf,
" 'f I., I" -;! ' ! ! I"
--.,..1 1.
~t. ~''!t* .~ . i- +Jlt t "
t
;:1~ ~~,:i ft.!! ~t!4 .JLr1.~~:t ~i!t- :Ii~ ~tJ.; i~~"~ .lit;. :-:.L~ ~~lI ~,llt t~1 ~~t. ;i:; .:::
' i ' j i'l: Itli 11:' if:! ill: l-iT' '1 1 , t;i til! ~III'1;i I"j lIt! ;~tl f!~j it~:
l.lt
t~ .. 'hj ..h"
t..l 1 '['.'1 ,f.. ~t ,

,.,.. It" ...


!ll'~
!jl '1-'
",'
+-.
f-.. t .... .J...
H ~ !-t,1
~ ,~ .... -:.1 . t i : i ~t
1"
.'.
.-~ S ..
0" j.-".1'
... t
,. Il
I ! t". ..
H
J

moving aJong a level roild at a constant speed

;:;.

(ii)

..

""

::;: :~:.

accelerating along a level road.

GO ON 10TH!! NEXT PAGE

( 1 marks)
TotalJO marks

OOl492/FIJANUARY 99

PHYS - P2 + P3 - JAN 1999 -:- Page 2 of 8

148

149

2.

. Describe how you would detennint: the density of a small rock of an irregular
shape.

(a)

2.~t~l.J.4!!.d.
(b)

On the gridbelow, draw velocity~time graph for an object which is released from rest
on the Moon, falls for 3 s but does nut reach" the ground.. Label the axes and insen
appropriate scales.

..........
,~

.~. !'~ ;

i.. '1 ~~:


....
~

I )

( 4 marks)
4 :__ 1-,

(ii)

When astronauts larided on the Moon they collected pieces of rock: While on
the. Moon one piece of rock wa.c:; measured. and found to have a volume of
6.4 x .10-' m3 and a mass of 0.36 kg.

_L.,. __

........

..........,,,,,., ... -;...-.--

What was the. density of the rock?

f-t ~1'

-: +--i't

~ .... -!~

( 2marlcs)
(c)
(2 marks)

(iii)

On the Moon the acceleration due to gravity is 1.7 m s -2. The rock was attached
to a spring balance and lowered into water until it was completely submerged.
The reading on the spring balance decreased due to the upthrust of the water.

A small rock is thrown ~orizontally on the Moon with a speed orJ.4 m s -l.
(i)

Write down.thevenical component of its velocity after one second.


( I mark)

(ii)

Write down the horizontal component of its velocity aftti(one second.

Calculate the upthrust of the water on the rock.


(lmark)
(iii)

. Sketch a vector diagram in the space below to show how you could determine the
velocity oftlle rock after one second - you are not required to fmd th velocity.

(4 marks)

(Density of water 1 000 kg 01-3)

PHYS - P2 + P3 - JAN 1999 - Page 3 of 8

149
~

3.

This question concerns the refraction of water waves and light.


(a)

A block of glass is placed on the bottom of a ripple tank so that the water above the block

is very shallow. Plane waves travel from the left across the deep water. through the

shallow water and then continue into the deep water as shown in Figure 1.

150

s.

Speed in shallow water

Deep water

( 3 marks)
Speed in deep water beyond block

( 2 marks)

Deepwater
(ii)

Calculate the frequency of the water waves.

(3 marks)
(iii)

Calculate ihe refractive index for water waves going from the deep to shallow
water.

(3 marks)

(b)

Figure 1
(i)

As thewaves approach the block tbey are travelling at a speed of 6.4 m s -1. Take
measurements of the wavelengths. on the diagram. and use these measurements
to determine the speed of the waves in the shallow water and in the deep water
beyond the block.

The diagrams below show a coin in a porcelain cup - in Figure 2 the cup is empty; .
in Figure 3 water has been poured into the cup. Neither the coin nor the eye change
position but in Figure 2 the coin cannot be seen by the eye willie in Figure 3 the eye can
see the coin.
.

~ye

lL~

Wavelength in Shallow water

Wavelength in deep water

Figure 2

( 1 mark)

"fE)"e

water-H . .

~c...
Figure 3

Draw ONE light ray from tbe coin Figure 2 to explain why the coin cannot he seen
by the eye. On figure 3 draw TWO rays which leave from the SAME point on the
( 3 marks)
coin and enter $e eye.

PHYS - P2 + P3 - JAN 1999 - Page 4 of 8

Total 15 mark...

150

...

d..

.....

151
4.

Beta-panicles. in a vacuum. enter a region with a uniform magnetic field which isperpendicular
to the plane of their motion. as shown below. Figure 4.
"!:<d)

VACUUM

~partides

X X

Describe bow you would detenlline the range of alpha-panicles in air.

maJtfletic: field

into paper.

Figure 4

(a)

What is a beta-panicle?
( 4 marks)

(e)

What would be the range of alpha-particles in a vacuum?

( 1 mark)
(b)

On Figure 4 draw the path of the beta-panicle in the magnetic field. What causes the
beta-particle to take this path?

( 1 mark)

Alpha-particles are emitted by radium -226 (~Ra) and a radon (Rn) nucleus is fonned
Write the equation for this nuclear reaction.

( 3 marks)

(3 marks)

(c)

Write down TWO differences in the path that would be observed if the panicle was an
alpha-panicle moving at the same speed.

(:2

Total 14 marks

marks)

PHYS - P2 + P3 - JAN 1999 - Page 5 of 8

151

It;S

~
(c)

5.

(a)

152

Ccn1irw.e.4:(

An alternating current is applied to the coil. Descnbe and explain the resultant moti~
, of the coil.

State the name ofthe rule which allows us to predict the direction ofthe fotce on acunent
carrying conductor perpendicular to a magnetic field.

( 1 mark)
(b)

Figure Sis a uiagram of a simple moving-coilloudspeaker.

( 4 marks)

Coilslaowing

current out

or paper

(d)

Sound is produced bytbe loudspeaker as a result of the motion referred to in part (c).

What would be the effect of increased coil current on the


(i)

I '

pitch of the sound produced'?

~agnet

( 1 mark)
(ii)

loudness of the sound produced?

( 1 mark)

Coil showinl
current into
paper

(e)

FigureS

The coil is disconnected from the a.c. source and connected to a galvanometer (a
sensitive, centre zero inst:run1ent used to indicate tbe magnitude and direction of smaU
currents). Part A in Figure 5 is then made to vibrate at low frequency by displacing it
by hand. State what you would observe .on the galvanometer and explain your
observation.
.

Identify the parts labelled A and 0 and indicate the polarity of tbe magnet at the points

X,Y,Z.

( 1 mark)

x
y

( 1 mark )

-'-

_
..c--

( 5 mark..,,

Total IS marks

---,..-------..,.---------
( 1 mark)

END OF TEST

152

PHYS - P2 + P3 - JAN 1999 - Page 6 of 8


~.

~~~

153
\

'"
( 1.

(a)

PHYSICS PMER 3 - JANUARY 1999


General Proficiency - IHr

Explain what is meant by a magnetic field. In your answer booklet. draw two bar magnets.
arranged as in Figure 1. and sketch the magnetic field pattern around and in between them.

IS

INS

Figure 1 .
An iron bar is placed near to a coil.aiI: shown in Figure 2.

r'lrOnbar
F(~.:i'=<J

F1gure2
A steady current is passed through the coil. State and explain the effect on the iron bar.

( 8 marks)
(b)

Figure 3 below shows the partS of a meter which measures electric current.

2. (a)

MoviDl iron bar


Fixed iroa blll'
Coil
Spring

..

(b)

Distinguish between energy and IlOwer and write an equation relating these twp
.quantities. Explain what is meant by work and gravitational potential energy. What
are the main energy transformations that occur when
(i)

a person walks up a flight of stairs'!

(ii)

an electric motor raises an object?

(i)

A girl of mass 45 kg walks slowly up a flight of stairs to the next floor in an


hotel. Each step is 0.24 m high and there are 25 steps. How much work does she
do against gravity'! If it takes her 21 s to reach the top. what is her average power
output?
( 5 mark.o;)

(iii)

The hotel has an elevator of mass 240 kg. If. instead of usi.ng the stair.;. the girl
decides to take the elevator. what would be the total work done against gravity
by the elevator motor? What is the efficiency of using the elevator when it is
bei.ng used,to lift only the girl?

AlIlperos(A)

Figure 3
The current flows through a coil. Inside the coil there is a fixed iron bar and a second iron
bar which is attached to a pointer: A spring is also attached to the pointer. When current
flows in the coil the pointer moves across the scale and then comes to a stop.
(i)

(ii)

Explain why the pointer moves when current flows and why the pointer stops
moving at a particular scale reading. Explain whytbe pointermovesJurt~rbefore
stopping when the current increases. Write down the reading on the meter on
Figure 3.
Can alternating current be measured with this meter? Explain your answer. Wlly
should you not pass a current greater than lOA through this meter?
(11

marks}

( 4marks)
(Acceleration due to gravity = -10 m S-2)

Total 20 marks

PHYS - PI + P3 - JAN 1999 - Page 7 of 8


.".....

( 8 mark.<;)

Total 20 marks

153

t/"

154
i \

II'
: I

3.

(a)

In the past. different theories of light have heen put forward hy scientists. State any
ONE property of light which could he used to suppon a particle the<JT}' of light and
explain how the property supports the theory. Descrihe the Young's double slit'
experiment and show how this experiment SUppoMS a wave theory of ligbt.
( 8 marks)

(b)

An observer in a building looks down at a swimming pool at night. He nOlices that a lanlp
at the bottom of the pool produces a circle or light at the surface of the water.

,4. (a)

A three-pin plug has live. neutral and earth wires attached. What is the purpose of
the earth wire? To what is the other end of this earth wire attached and what ,is its
International Colour Code? What is the purpose or a ruse and why is it particularly
dangerous to have an earth wire fitted to an appliance without a fuse? What is the name
and International Colour Code for the w4re into which the ruse is fitted?
( 8 marks)

(b)

Anelectriccoukerdraws a powerof9.5 kW when allbumers areheingused. Ifthe cooker


is operated on a 220 V supply. calculate the current that it draws when all burners are
being used. The cable that is used to connect the cooker to the mains supply has a
resistance of0.036 n mol. If the cahleis 15 m long, calculate its total resistance and the
( 8 marks)
power lost in this cable when the cooker is drawing maximum power.

(c)

Electrical energy has to travel considerahle distances from power stations to our homes.
If the power station was 15 km from your home and the same type ofcable as above used.
what would be the total resistance of the transmission cable? The house needs to be
supplied wit h 220 V in orderto operate the cooker. What would the supply voltage at the
( 4 marL'll
power station need to be?

Air

Total 10 marks

s: (a)

Figure 4

(i)

Copy Figure 4 and add the paths ofTIlREE rays leaving the lamp which strike

the surface and continue. Choose your rays so that

ray I has an angle of incidence of zero


ray 2. has an angle of incidence of about 25

ray 3 has an angle of incidence of about 60 <> .

(b)

State THREE differences between boiling and evaporation. Use the kinetic theory to
explain how evaporation can take place at all temperatures but occurs more rapidly at
higher temperatures. Why is there a drop in temperature in a liquid when rapid
evaporation occurs?
( 8 mark.,)
Following an accident. a patient ofmass 70 kg requires the transferof2 kg of blood. TIle
blood is at.5 <>C when transferred to the patient and is wanned up to the patient's final
temperature of 35 <>c. Ciilculate the energy transferred from the patient's body to the
blood. Find also the change in the patient's body temperature.

(TIle critical angle for the water-air buundary is 49 )


(ii)

(iii)

State which or these rays would be visible above the surface of the water.
Explain why a circle of light is seen and find the refractive index of water.
(11 marks)
Total 20 marks

This temperature change may cause 'shock' and to prevent this the patient is wrapped
in a 500 VI electrically-heated blanket. How long would it take for the patient's body
temperature to be restored to its initial value?
It is found in practice tbat it takes more tban the time calculated aquve. for the patient's
body temperature to be restored to its initial value. Account for this difference.
(Average specific heat capacity of blood := 4 500 J kg -I K _I;
Average specific heat capacity of body tissue := 2500 J kg -I K -I)
(11 marks}
Total 20 marks

PHYS - P2 + P3 - JAN 1999 Page 8 of 8

END or TEST

154

"

155
11
(ii) The most common isotope of boron (B) may be represented as 5 B . A

1995 Paper 2
1. State Archimedes Principle

boron atom has two shells of electrons with two electrons in the first shell. In the
space below, draw a clear diagram to represent the number and distribution of
protons neutrons and electrons in a neutral

11
5

B atom.

(4 marks)
(iii) Write the symbol for another possible isotope of boron. (1 mark)

(b)

A hot-air balloon, as shown in Figure 1 above, has a volume of 700 m3


and the density of the air inside the balloon is 0.90 kg m -3. The mass of
the balloon's material and load is 280 kg.

(i) Calculate the mass of air inside the balloon.


marks)

(3

(ii)

Determine the total weight in newtons of the material, load and air. (g =
10 N kg-1) (2 marks)

(iii)

The balloon remains stationary about 100 in above the ground. State what
forces are acting on the balloon, and explain how they result in no motion.
( 3 marks)
If the density of the air outside the balloon is 1.3 kg m-3 , calculate the
weight of the air which is displaced by the balloon.
(2 marks)
Explain why the balloon begins to rise when the air is heated to a higher
temperature. (2 marks)
Total 14 marks

(iv)

(v)

(a)
2,
elements.

Table I below represents the first three periods of the Periodic Table of

(b)

The count-rate (activity) a measured for two radioactive samples. A and B. and
graphs of count-rate against time are plotted, as shown in Figure 2 above.

( i)
By reading off vale of the count rate at suitable times show that the hall-life
of Sample A is constant and find its value. ( 4 marks)
(ii)
By considering the relevant graph comment on the half-life of Sample B.
(1 mark )
(iii ) Why are the points on the graph of Sample B not exactly on the line even
though the ratemeter was not faulty?( 1 mark )
(iv)
What difference, if any, would be noticed if the graph of sample B was
heated?
(1 mark)
(v) Sample B is an isotope of thorium (Th) with a proton number of 90 and a neutron
number of 142. It decays by the emission of an alpha particle to form radium (Ra).
Write a nuclear reaction to represent this decay.
(3 marks)
Total 16 marks

(i) The atomic number of chlorine (Cl) is 17. How many protons are there in
one atom of silicon (Si)?
(1 mark )

155

156
3.
A family uses electricity for heating water used in their home. The 1.5 kW
electric water-heater is controlled by a thermostat so that it is switched on for an
average of four hours each day.

(c)

One of the reasons for having a glass cover over the collector plate of the
solar water-heater shown in Figure 4-above, is to take advantage of the
glass-house (or green-house) effect.
Explain the glass-house effect (3 marks)
Total 14 marks

4.

Figure 5 below shows a wire. XY, between the poles of a magnet connected
to a centre-zero galvanometer, G.

(a) A simplified diagram of the thermostat is shown in Figure 3 below.

(a)

(i)

(i)
(ii)

Explain how the thermostat works. ( 3 marks)


What would be the effect on the working of the thermostat if
(ii) d with the iron uppermost?
the bi-metallic strip were inserte
(2 marks)
(iii) tion of the water-heater if
(b) (i) What is the cost per year of the opera
one kilowatt-hour costs 40 cents?
(4 marks)
(ii) To save money the family considers installing a solar waterheater costing $4,400. How many years must pass before the cost of
the solar heater is equal to the cost of the electrical energy saved (the
"pay-back time")?
( 2 marks)

What would you expect to observe on the galvanometer if the


following actions were carried out?
The wire is moved up and down.
The wire is moved up and down faster.

(2 marks)
(1 mark)

The wire is moved from pole to pole horizontally. (1 mark )


(iv) The magnet poles are moved up and down with the wire
stationary. (1 mark)
(b)

Figure 6 below shows two coils, side by side, in separate circuits, A and B.

Describe what you would expect to observe on the centre-zero galvanometer


when the switch in Circuit A
(i)
is closed
(1 mark )
(ii)
remains closed
(1 mark )
(iii) is opened
(1 mark)

156

157

June 1995 Paper 3


1.

Two mechanics use ropes to lift an engine out of a car. The ropes are at an
angle of 45 to the horizontal, see diagram below. Both men pull with the
same force. i.e., 400 N.

(a)Draw a vector diagram showing how the TWO forces can be added to give
an upward force to balance the weight of the engine, and find the weight of the
engine.
(5 marks)
(b)
(i)Explain why it is easier to use a pulley system than to use the ropes
attached to the engine.
(1 mark )
(ii) Suggest TWO ways that energy would be wasted by the pulley system
so that it would not be 100% efficient.
( 2 marks)

(i) The symbol for a transformer is shown above. Complete the circuit diagram
to show how a transformer, together with the components listed below,
could be used to charge a 12 V car battery from a 120 V mains supply.
Components: Rectifier diode, variable resistor. ammeter
The variable resistor and ammeter are used to obtain the correct charging
current through the battery.
(5 marks)
(ii) The potential difference from the secondary cod of such a transformer is
18 V, and the charging current through the 12 V battery is 2 A. What is
the value of the variable resistance?
( 3 marks) Total
16 marks

(iii) Describe fully how you would measure the efficiency of such a pulley
system in the laboratory.
(5 marks)
(c) The forearm is a lever in which the bicep muscles provide an upward
force which can raise the arm and carry a load. The diagram below
represents the forearm as it holds a load of 60 N. The forearm has a
weight of 25 N and the centre of gravity, G. of the forearm is marked
along with the point, P, at which it pivots on the adjoining bone.
(i) Use the principle of moments to calculate the fore. B. exerted by the
biceps to hold the arm and load stationary.
(ii jThe adjoining bone pushes down or. the forearm at the point, P, with a
force. R. By considering the upward and downward forces calculate the
value of R. ( 7 marks)
Total 20 marks

157

158

2.

Three resistors are arranged as in the diagram above.


(i)
Calculate the total effective resistance between A and B. (4 marks)
(ii)
A p.d. of 12 V is found to exist between A and B. Calculate the value
of the current. I, flowing into the junction at A.
(2 marks)
(iii) Determine the current in the 6 ohm resistor. (2 marks)
(iv)
Total 20 marks

(a)

3.

The diagram above shows a double mains outlet of a type common in the
Caribbean. Are these outlets in series or parallel? Explain why they are
connected in this way.
(3 marks)
(b) An extension cord is rated at 10 A meaning that it might overheat if the
current exceeds this value. Use the information below to calculate the
current that would flow through the extension cord if both appliances were
used at the same time.
Appliance
Electric Iron
Hair-drier

Power
960 W
600 W

(a)

Given a straight vibrator and a ripple tank, describe how you would
demonstrate the reflection and refraction of water waves. Draw
diagrams to represent what would be observed in EACH
case.
(8 marks)

(b)

Ultra-sound (high frequency sound) is used in medicine to perform


internal investigations upon patterns.

As the ultra-sound passes from muscle into bone, some is reflected and
diagram below represents a beam of ultrasome is refracted. The
sound incident upon a muscle/bone boundary.

Voltage
120 V
120 V

Would it be safe to use the cord with an electric iron and a hair-drier at the same
time?
(4 marks)
(c) Describe how you would perform an experiment to test the hypothesis that
the sum of the currents in the branches of a parallel circuit is equal to the
current entering the parallel section. Include with your description a
circuit diagram showing clearly the position(s) of any
meters) you would use.
(5 marks)

Medium
Air
Muscle
Bone

Speed of Ultra-sound
330ms-1
1 600 m s -1
4 000 ms -1

158

(i) The ultra-sound used has a frequency of 8.0 x 10 6 Hz. Calculate the
wavelength of the ultra-sound in bone.
(3 marks)
(ii) Use the information in the table to calculate the angle of refraction in the
bone. Copy and complete the diagram to show what happens to the ultrasound. (5 marks)
(iii) Ultra-sound can be totally reflected at certain boundaries. What is the
critical angle for ultra-sound incident upon the muscle-bone boundary?
Is total reflection possible as ultra-sound. travelling in muscle, meets the air?
Explain your answer.
(4 marks)
Total 20 marks
4. Suppose you are employed by a school as a temporary laboratory technician. One
day the physics teacher asks you to set out and test equipment so that the
students can perform an experiment to determine the specific neat capacity of
a liquid, using an electrical heating method.
(a)
List the apparatus you would set out, apart from the electrical heater
and the liquid. for EACH group of students. (5 marks)

159
5.

(a) State Newton's THREE laws of motion.


A communication satellite travels in a circular orbit around the Earth. Explain (i)
how this circular motion is maintained and (ii) the way in which Newton's third
law applies to the satellite.(8 marks)
(b)An oil drilling platform. or mass 8.0 x 10 7kg. is being towed at a speed of 0.75
ms-1 by tugs to its working position. When the platform is rear to its working
position, the rugs are disconnected and the platform slows down.
(i) If the deceleration of the platform is 7.5x 10-3 m, what is the unbalanced
force acting on it and how long would it take to come to a halt? (6 marks)
(ii) What momentum did the platform possess when it was moving with a speed
of 0.75ms-1, and what was the change of momentum per second as it slowed to a
halt? (6 marks)
Total 20 marks

(b)
Draw and label a diagram of the circuit you would use to perform the
experiment.
(3 marks)
The teacher then tells you that EACH group is to be provided with 200 g of a
liquid with a specific heat capacity of 2 200 J K-1 kg -1 . She would like the
liquid to have a temperature rise of 15 K in 5 minutes, but there are no
suitable heaters in the laboratory. She asks you to make some heaters using
nichrome wire. (See diagram below.)

(c)
Determine one quantity of thermal energy required for this
temperature rise, and the power of the required heater. ( 6 marks)
(d)
If the supply voltage is 11 V calculate the resistance of the wire and
the length needed for EACH heater if the resistance is 10 ohm per meter. ( 6
marks)
Total 20 marks

159

1994 Paper 2
1. Table 1 below gives information related to pan of a natural radioactive series. It
shows that Element A emits a beta-particle and a gamma-ray and changes into
Element B. Element B then undergoes a radioactive change to become
Element C and so on.

Radiation
emitted

Atomic
number

Mass
number

83

214

84

214

1.6 x 10-4 sec

82

210

19 years

83

210

5 days

84

210

138 days

Stable

20 minutes

Table 1
(a)
(b)
(c)

What is a beta particle?


(1 mark)
What u a gamma ray?
(1 mark)
What are the atomic number and mass number of an alpha particle?

Atomic No

Mass No

Figure 1 above represents a 'dimmer' in a house lighting circuit. The brightness of the
lamp may be altered using the variable resistor. X. The supply voltage (p.d.) is 220 V
and the resistance of the lamp is 440
(a)
With the variable resistor set at zero resistance, calculate
(i)
the current in the circuit
(3 marks)
(ii)
the power output of the lamp.
(3 marks)
(b)
With the variable resistor set at its maximum value of 660 . Calculate

(2 marks)

(d) Name the radioactive emission when Element B changes to Element C. and
Element C to Element D.
C to D
(2 marks)
(c) State the atomic number and mass number of Element F. ( 2 marks)
(f)

How many protons and neutrons does the nucleus of Element D contain ?
(2 marks)

(g)

Name a pair of elements in Table I that are ISOTOPES of the same


element. ...................................................................................(1 mark )
Explain what is meant by the half-life of a radioactive isotope. ( 1 mark )

(h)

2.

Half life

Element

160

(i)
the current in the circuit
(2 marks)
(ii)
the potential difference across the lamp
(2 marks)
(iii)
the power output of the lamp
(2 marks)
(iv)
the rate at which energy is supplied to the circuit.
(2 marks)
(c) Account for any difference between the answers to (iii) and (iv) above.
(1 mark )
Total 15 marks

(i) If the activity of a sample of Element D at a certain time is 600 emissions


per second.
calculate the activity of the sample 15 days later.
(3 marks)
Total 15 marks

160

161
3.

(a)

Light and sound are both forms of waves. State TWO differences
between light waves and sound waves. ( 2 marks)
4.

A plank of wood is used to help lift a heavy box. (weight 1000N), into the back of a
truck as shown in Figure 3 above. The plank is 10 m long and the object rises to a
vertical height of 1.5 m. A force of 180 N is used to pull the object along the plank.
(a)
Define the unit of energy the joule.
(2 marks)
(b)
(b)

The diagram above shows a loudspeaker at the end of a room with an


open door leading into another room. A sound of frequency 255 Hz is
emitted.

(i)
Draw ON THE DIAGRAM a representation of the sound wave
emerging from the door.
(2 marks)
(ii) Why is someone standing at Point T able to hear the sound but
not able to see the loudspeaker?
(3 marks)
(iii)
If the speed of sound is 340 m s -1. Calculate the wavelength of
the sound waves.
(3 marks)

(c)
(d)
(e)

Determine the change in gravitational potential energy of the box as it is lifted


into the truck
(3 marks)
How much work is done by the 180 N force? (3 marks)
State the principle of conservation of energy and use it to explain why your
answers to (b) and (c) above are not the same. ( 4 marks)
Consider the plank of wood as a simple machine and calculate its efficiency.
(3 marks)
Total 15 marks

(c)
In a storm the lightning is often seen a considerable time before the
thunder is heard.
(ii) Account for this fact.

(2 marks)

(ii) If the lightning is seen 8 seconds before the thunder is heard,


calculate the distance of the thunder cloud from the observer.
(2 marks)
What assumption have you made in your calculation in (ii) above?
(1 mark)
Total 15 marks

161

162

June 1994 Paper 3


1.(a) A girl, seated in an aeroplane that is preparing to take-off has to fasten her
seat belt. Describe the motion of the girt relative to the ground and identify
the forces if any, causing her motion
(i) when the plane is taxiing at a steady speed to the end of the runway
3.

(ii) when the plane is accelerating down the runway


(iii) if the plane has to stop suddenly.
(b).

[ 6 marks)

An aeroplane of mass 2 x 10 5 kg starts from rest at the end of the


runway and accelerates uniformly at 1.6 m s -2 until it attains a speed of
180 km h-1 (50 m s -1).
Calculate
(i)
the time spent during acceleration
(ii)
the accelerating force
(iii) the kinetic energy gained.
(9 marks)

(ii) Calculate the value of the critical angle for ice.


(3 marks)
(iii) Draw a large diagram to show the path of the ray of light
through the ice. Mark the sizes of the angles of incidence and show
clearly where the ray leaves the ice.
(6 marks)
Total 20 marks
a) Describe an experiment to determine the specific heat capacity of a
metal.
Make clear how you would calculate the specific heat capacity from
your readings. State what precautions you would take to ensure that the
result is accurate.

The pilot now decides to abort the take-off and manages to stop the
aeroplane while travelling 500m further down the runway. Describe the
energy transformation that takes place as the aeroplane stops and
determine the size of the average retarding force.
(5 marks)
Total 20 marks
2(a) (i) In terms of the wave theory, explain why refraction of light
occurs. Illustrate your answer with a labelled diagram. (4 marks)
(iii) With the aid of ray diagrams, explain the terms 'refractive index'
and criticalangle'. (4 marks)

b)
Ahen incubates her eggs at 38 C for three weeks until they hatch. A
farmer decides to incubate some eggs. He makes an incubator as shown by
Figures 2 and 3 above.
Outline the mechanisms by which the eggs are kept warm by the
hen and in the incubator.
(4 marks)
(ii) Suggest TWO methods by which the farmer can alter the temperature of
the eggs.
Explain how EACH method works.
(4 marks)
(iii) EACH egg in the incubator has a mass of 0.06 kg and a specific heat capacity of
4000 J kg -1 K -1 In order to keep the eggs at a constant temperature of 38 C.
EACH egg must receive heat energy at a rate of 3 W. I f the lamp is switched
off, how long will it take for the temperature of the eggs to fall to 36 C
(4 marks)
Total 20
marks

A ray of light hits a block of ice at P. as shown in the diagram above.


The refractive index of ice is 1.6.
(i)
Calculate the angle of refraction of the ray of light incident at P.
(3 marks)

162

163
4.(a) Draw a labelled diagram of a simple moving-coil loudspeaker. (6 marks)
(b)
Explain what would happen if sound waves were incident on such a
loudspeaker.
(2 marks)
(c) The amplifier of a radio is connected to the loudspeaker via a transformer. The
transformer has 1 000 turns in the primary coil and 200 turns in the secondary
which is connected to the coil of the speaker. The coil of the speaker has a
resistance of 4 .
When the radio is turned on. the power delivered to the speaker is 0.1
W. Calculate
(i) the current flowing in the loudspeaker and the p.d. across it
(6 marks)
(ii) the current flowing in the primary of the transformer and the p.d. across it.
(6 marks)
Total 20 marks
5.(a) Sketch a graph to show the variation of current with applied potential
difference for a filament lamp and state how the lamp's resistance changes as
the current increases. (4 marks)
(b)Christmas tree lights are connected in sets which are either in series or
parallel. State and explain what you would observe if a bulb were to blow in
EACH case. (4 marks)
(c)

A set of Christmas tree lights consists of eight 15 W bulbs in series,


whilst a second set has eight 15 W bulbs in parallel. A salesman tells a
customer that the sets of lights will draw identical currents when plugged into
the 120 V mains supply.

(i) Determine the potential difference and current for EACH bulb in the two sets
and hence determine whether the salesman is correct.
(8 marks)
(ii)Calculate the cost of operating EACH set of lights for 10 hours if electricity
costs $1.50 per kWh.
(4 marks)
Total 20 marks

163

June 1993 P 2
1.

164

A length of paper tape is pulled through a timer by a toy car. The timer
makes one dot on the tape every 0.02 s. A section of the tape, drawn full
size, is shown in Figure 1 below.

A second length of papa tape is pulled through the same timer by a toy
truck. A section of this tape, also drawn full size, is shown in Figure 2
below.

(i)

What time interval is represented between the first and last dots on
the tape in Figure 1 and in Figure 2?
(2 marks)
(ii) By considering displacements from A in EACH case. complete the
following tables:

Time/s

0.

0.04

Displacement/ cm

12

2.4

CARTable 1
Time/s

0.0

0.02

0.04

Displacement/cm

0.0

0.2

0.8

TRUCK Table 2
(iii)

(4 marks)
Use the information in the completed Tables 1 and 2 above to plot
displacement (x) time (t) graphs on the axes provided on the next
page.
(6 marks)
(iv) Describe briefly the motion of the toy car and the toy truck.
(2 marks)
(v)
Use the information in the completed Table 2 to
determine the average speed of the truck over the total
time interval.
(2 marks)
Total 16 marks

2.(a) (i)
Define 'specific heat capacity'
(4 marks)
(ii) Explain what is meant by the term 'efficiency'.
(b)
A large amount of the thermal energy produced in modern power stations
is wasted.
(i)state the useful energy changes taking place in an oil-fired power station.
(3 marks)
(ii) In such a power station thermal energy is wasted at the ram of 660 MW .
How many Joules of energy are wasted in ONE second'?
(1 mark)
(iii) A nearby river is used to absorb the waste heat. A mass of 6 10 4 kg of
water passes the power nation every second. What is the temperature rise of the
water if it absorbs all the waste heat?? [Specific heat capacity of water = 4200 J
kg -1 K -1]
(iv)An alternative way of disposing of the waste heat is to use it to evaporate
water in the power nation.
How much water would have to be evaporated EACH second if the latent heat
(3 marks)
of vaporisation of water is 2.3 106
(v)
The electricity is produced in the power station at a potential of 6 600 V
and this is raised to 132 000 V for distribution.
State how the potential is raised and give ONE advantage of transmitting at a
higher potential.
(2 marks) Total 16 marks

164

165

3. (a) The danger of electrical shock to the human body depends largely on the
body's resistance. With dry hands. this resistance is about 100 k but it falls to
1 k when the hands am wet. A current of about 10 mA causes slight shock,
wheras a current of about 1.0 A can cause death.
(i) No current flows when someone touches just ONE terminal of a 12 V
car battery. Explain this fact
(1 mark)
(ii) A boy touches one terminal of a 12 V car battery with his right hand and
the other terminal with his left hand at the same time. Calculate how much
current flows in EACH of the cases below:
What effect, if any. does the current have on the boy in
EACH case? (7 marks)
(b) During a heart attack, the heart beats very irregularly. The heart may resume
its normal beat if a large current is passed through the body for a short time.
This current can be supplied by a machine which stores a charge of 4 x 10 -2 C
at a potential of 6 000 V.
(i)Calculate the electrical energy that is stored in the machine. (3 marks)
(ii) The total charge is passed in a current which lasts 5 ms. Calculate this
current
(3 marks)
Total 14 marks
4.
Short wavelength radio waves (microwaves) have many applications, e.g. in
weather stations.
(iv)
(i)
The radar used in weather stations transmits microwaves of frequency 3
10 9 Hz. These are emitted in pulses EACH of 1.0 10 -6 s duration.
Calculate: - the wavelength of the radiation
- the number of waves animal in a pulse
(Speed of electromagnetic waves = 3 108 m s-1 )
(ii)
The power of the transmitter is 200 kW. What is the total energy in a
pulse?
(3 marks)
(iii) After the emission of a pulse, nothing is transmitted for one millisecond.

(iv)

Calculate how far the pulse travels us this time.


(3 marks)
To detect a daunt object, the pulse is reflected from it and must return
to the weather station before another pulse is emitted. What is the
furthest distance at which an object may be detected using this
system?
(2 marks)
Total 14 marks

1993 Paper 3
1. (a)
(b)

(c)

Distinguish between kinetic energy and gravitational potential energy.


(2 marks)
A stone is dropped from the top of a building onto the ground. Neglecting
air resistance, compare its kinetic energy and its gravitational potential
energy
(i) at the point of release and
(ii) half-way down.
Explain whether or not the stone possesses any kinetic energy or
gravitational potential energy when it is lying on the grand.
(4 marks)
For smaller buildings, engineer; design elevators to travel slowly because it is
more economical
An elevator driven by an electric motor carries a group of people. of total
mass 900 kg. to the upper floor of a building. The mass of 900 kg accelerates
uniformly from rest to a speed of 4 m s -1, in a time of 3 s, and continues at
this speed until it is brought to rest by applying brakes.

(i)

Calculate the acceleration of the total mass and the force required to produce
this acceleration.
(ii) Determine the maximum kinetic energy attained by the 900 kg mass.
(iii)

If the maximum speed reached were 2 m s-1, what would have been other
value of the accelerating fate and of the maximum kinetic energy?
Explain why it is more economical to run slow elevators.
Total 20 marts

(14 marks)

2. (a)
The atmospheric pressure on a roof is 1.0 x 105 Pa.
(i) Give an equivalent unit for the Pascal (Pa).
(ii) Explain in terms of the particle theory of gases, how this pressure arises
(4 marks)
(b) When a volatile liquid evaporates, the remaining liquid cools. Explain the
cooling in terms of the particle theory of matter. (4 marks)
(c) A solar cooker consists mainly of a curved reflecting metal surface that
focuses sunlight onto a pot containing food to be cooked The reflecting surface has
an area of 0.6 m2 and the solar power incident on it is 800 W m 2. Seventy-five per
cent of the incident energy is absorbed by the pot and its contents and as
temperature rises by 70 oC in one hour.
(i) Calculate the energy that is incident on the reflecting surface every second.
(ii) Determine how much energy is absorbed by the pot and its contents during the
hour.

165

166
(iii)

(ii) Represent the formation and decay of cobalt 60 by two balanced


nuclear equations

How much thermal energy is required to increase the temperature by 70 C


if the heat capacity of the pot and its contents is 8 000 J K -1 ? Explain why
this value of energy is not the same as that in (ii) above.
(12 marks)

(iii)

Total 20 marks
(a)Newton investigated light and observed that light of several colours
appeared on a scram when white light was passed through a prism.
Use fully labelled diagrams to illustrate how Newton used two prisms to
show that

3.

(i)

the different colours were merely components of white light and


not the result of the interation of white light with the material of
the prism

(12 marks)
Total 20 marts

5.
(a)State what is meant by the term 'magnetic field' and slaw how the direction
of the field is defined.
(2 marks)
(b)

(ii)
any single colour produced by one prism could NOT be split into
other colours.
( 8 marks)
(b)

(c)

4.

A converging lens of focal length 10.0 cm is used in a projector to form


an enlarged image of a slide on a screen. The slide is 4.0 cm high and is
placed 12.0 an from the lens. Determine by scale drawing on graph
paper (i) the position at which the screen should be placed to receive a
sharp image and (ii) the sire of the image.
(7 marks)

Determine the number of half-lives for cobalt 60 in a period of


26 years and hence calculate how much of a 8.0 g sample of
cobalt 60 will remain after this period.

Sketch (i) the magnetic flux pattern (magnetic field) around


a long straight current
carrying wire
(ii) the magnetic flux pattern mound the current-carrying wire
when it is placed perpendicular to a uniform magnetic field.
Indicate on the diagram the direction of the force on the
wire.
(6 marks)

(c)

The diagram below shows the earpiece of a telephone. Explain how


the earpiece produces sound.

In some older people. the eye lens becomes opaque. This condition is
known as a cataract and is sometimes corrected by removal of the eye
lens. Use a diagram to illustrate what effect this removal of the lens will
have on the eye's ability to form images on the ream and state what type
of spectacle lens may correct this problem.
(5 marks)
Total 20 marks

(a)Outline (i) the Rutherford model of the atom and (ii) the Thomson
model of the atom. Explain bow the results of the Geiger-Marsden
experiment supported only one of these two
models.
(8 marks)
(b)Cobalt 60 (

60
27

Co ) is one of the mom widely used radio-isotopes in

the treatment of cancer.


h is formed from natural cobalt by the capture of a neuron and it has a
half-life of 5.2 years. It decays into Nickel (Ni) by the simultaneous
emission of a beta particle and two gamma rays.
(i)

Which of the two emissions is more likely to be used in the


cream= of cancer and why?

(d)

A step-up transformer. which may be considered to be ideal, has an


output voltage, Vs. of 1,800 V when its primary is connected to a 120

166

167
V mains supply.
(i)
If the number of turns in the primary is 100, how many turns are
there in the secondary?
(ii)
If the current drawn by the load in the secondary is 0.5 A. what
is the current in the primary? (6 marks)
(e)

Someone suggests that for an ideal step-up transformer, the law of


conservation of energy is violated since the output voltage Vs . is
greater than the input voltage, Vp . How would you convince him that
the law is not violated?
(2 marks)
Total 20 marks
END OF TEST

167

June 1992 Paper 2


1.

(a)

Identify the following materials as good or poor thermal and electrical


conductors by writing the words 'GOOD' or 'POOR' in the spaces
provided.

MATERIAL

THERMAL
ELECTRICAL
CONDUCTOR CONDUCTOR

Air
Mercury

[Specific heat capacity of water = 4.2 x10 3 J kg -1 K-1]


[Specific latent heat of fusion of ice = 3.3 x 10 5 J kg -1 ]

2.

There are now several artificial satellites orbiting the earth. A student suggests
that these satellites can remain in orbit because they have escaped the earth's
gravity.
(i) Explain why these satellites would not stay in orbit if the student's
suggestion were correct.
(2 marks)
(ii) A weather satellite orbits at a height of 800 km above the surface of the
earth.

Aluminium

(b)

(3 marks)
The diagram below represents an ice-cream maker in which the
temperature of the ice-cream mixture is reduced by surrounding it with
ice and salt.

(i)
Indicate on the diagram by means of an arrow, the direction of
heat flow between the mixture and the ice/salt (1 mark )

(c)

(ii)
Explain why the inner container is made of a good thermal
conductor while the outer container is made of a poor thermal conductor.
(2 marks)
0.30 kg of ice at 0 C is placed in a container.
(i)
How much energy is required to completely melt the ice?
(3 marks)
(ii)
How much energy is required to raise the temperature of the melted
ice to 15 C?
(3 marks)
(iii) Some water at 30 C is added to 0.30 kg of ice at 0 C. The ice melts
and the final temperature of all the water is 15 C.
Use your answers to (i) and (ii) above to determine the mass of water
added.
(4 marks)

168

Calculate the tune taken for radio waves to arrive at the earth's surface
after being transmitted from the satellite.
(3 marks)
[Speed of electromagnetic waves=3.0 x l0 8 m s -1]
(iii) If the mass of the satellite is 27 kg, what is the gain in gravitational
potential energy by the satellite as it is lifted from the earth's surface to
its orbital height?
[Assume that there is no change in the force of gravity up to this height.]
(3 marks)
(iv) If the satellite takes 100 minutes to complete one revolution of the earth,
calculate its speed and kinetic energy given that the length of its orbit is
4.5 x 107 m. ______________________________ (5 marks)
(v) The radio signal is received from the satellite by an aerial with a concave
parabolic reflector. With the aid of a diagram, explain why a parabolic
reflector is used.
(3 marks)
Total 16 marks

3. (a) At the beginning of the twentieth century, scientists believed that neither
matter nor energy could be created or destroyed. The work of a great
scientist showed that in fact. for a chased system the total amount of massenergy remains constant.
(i)
Name the scientist referred to in the statement above.
(l mark)
(ii) Give the equation for the mass-energy relationship in a nuclear
reaction. State clearly what EACH symbol represents
(3 marks)
(b)

During a nuclear fission reaction. the uranium isotope

235
92

U absorbs a

neutron and splits into Barium (Ba), Krypton (Kr) and two neutrons,
with the release of energy.
This is represented in the equation below:
90
1
U + 01n144
56 Ba + 36 Kr + 2 0 n + Energy

235
92

168

169
(a)
The table below gives the masses of the particles involved in this reaction.

PARTICLE
Neutron

MASS/kg
1.675 10 -27

Uranium

390.98910 -27

Barium

238.893 10 -27

Krypton

149.24110 -27

The diagram above represents a d.c. dynamo.


(i) Write in the name of the components labelled A, B, C and D on the
dotted lines provided.
(4 marks)
The terminals X and Y are connected to a 2 Ohm resistor. The peak
value of the potential difference is 4 V.
(ii) Calculate the peak value of the output current
(1
mark)
(iii) The coil makes 10 revolutions in one second. On the axes below,
sketch a graph to represent the variation of output current with time
during the rust two revolutions of the coil. Indicate the scale used on
EACH axis

(i) What is the total mass of the uranium nucleus and the neutron?(1 mark )
(ii) What is the total mass of the two nuclei and the two neutrons produced as a
result of the fusion of
(iii)

235
92

U?

(2 marks)

How much energy is produced as a result of the fission reaction?


(Speed of light = 3.0 10 8 m s -1)
(3 marks)

(c) The fission of

235
92

U yields approximately one million times as much energy

as the burning of an equal mass of oil Fission is therefore used in nuclear reactors to
provide electrical energy.
However, that are disadvantages in using this method.
(i)
State TWO DISADVANTAGES of generating electrical energy using
nuclear fission reactors.
(2 marks)
(ii) Scientists estimate that every second the sun's mass is reduced by 4 10
9
kg.
Name and explain the process that lends to this reduction in mass.
(4 marks)
Total 16 marks

If the number of turns of wire on the coil is increased, with no other


change, state the effect, if any, on
(iv) the frequency of the output current
mark)
(v)

4.

the peak value of the output current.


(1 mark)
If instead of increasing the number of turns of wire, the speed of
rotation of the soil is increased with no other change, vase the effect, if
any, on
(vi)

the frequency of the output current

(vii) the peak value of the output current

(b)

(1

(1 mark)
(1 mark)

A mechanic states that a transformer cannot be used with the d.c.


dynamo to obtain a higher potential difference as transformers work
only with alternating current.
State whether the mechanic is correct and explain your answer.
(4 marks) Total 16 marks

169

170
5.

(a)

The diagram below shows plane water waves incident on a gap.

A loudspeaker such as that mentioned in (i) and (ii) is positioned in a


corridor. 1 metre wide as shown in the diagram above.
An observer notices that when she stands at X. she hears equally well from the
three speakers. However, when she stands at Y. she hears mainly from one
speaker and much less from the others.
(iii) From which speaker does she hear most sound when standing at Y?
(1 mark)
(iv)
With reference to the data in the table and diagram on page 10. give an
explanation for her observations while at Y.(2 marks)
(c)

The waveforms shown in the diagrams below represent three different sounds.

(i) Draw the waves after they have passed through the gap. (2 marks)
(ii) What name is given to this effect?
(1 mark )
(b)
Some sound systems have three loudspeakers to cater for the full range of
frequencies to be transmitted. The following table gives the wavelength
ranges for three speakers and frequency
ranges for two of them.
Speaker
Woofer(low range)
Squawker(medium range)
Tweeter(high range)

Frequency range/Hz
1,000-10,000
3,500-20,000

Wavelength range/m
14-.35
0.35-.035
0.1-0.018

(i) From the information given for the squawker. calculate the velocity of
sound.
(2 marks)
(ii) Determine the missing frequency range and insert it in the table
above.
(2 marks)

A - tweeter
B - squawker
C - woofer

Compare the loudness and pitch of


(i) P and Q
(2marks)
(ii) Q and R
(2 marks)
(iii) Explain in what other way sounds P and Q differ.
(2 marks)
Total 16 marks

170

June 1992 P3
SECTION I
You must answer the question in this section.
No more than hour should be spent on
this question.

In pole vaulting, most of the kinetic energy that an athlete possesses at the
end of his run-up is converted into gravitational potential energy. The
centre of gravity of the athlete is raised from a height, h, to a height, h. It is
known that the height that he achieves depends on his speed at the end of
his run-up. The data in the table below were obtained for an athlete while
pole vaulting.
v 2/m 2 s-2
Height
Speed.
-1
achieved.
v/ms
3.3
7.25
3.8
7.80
4.2
8.40
4.5
8.80
4.9
9.35
5.3
9.70
(a) Calculate the values of v and, IN YOUR ANSWER BOOKLET,
tabulate these values of v 2
along with the corresponding values of the height achieved.
(3 marks)
(b) Starting both scales from 0, plot a graph of v2 (y axis) against h (x
axis). (9 marks)
(c) Determine the slope of the graph.
(5 marks)
(d) The intercept ho on the x- axis represents the height of the centre of
gravity of the athlete
above the ground before he takes off. Determine ho from the graph.(2 marks)

171

Read off from the graph the value of h which corresponds to v = 9.0 m s -1.
(2 marks)
(f)
(i) Calculate the kinetic energy the athlete would have when his
speed was 9.0 m if his mass were 80 kg.
(ii)If this kinetic energy were all converted into potential energy, what increase
in height of the centre of gravity would you expect?
(iii) What total height would he achieve in this case?
(7 marks)
[g
= 10m s -2]
(g)
Suggest TWO possible causes to account for the difference between
this total height and the corresponding value from the graph. (2 marks)
Total 30 marks
SECTION II
Answer ANY THREE questions in this section.
2..
(a) Aristotle and Newton had different ideas about forces and the
relationship between force and motion. State Aristotle's and Newton's
views on any forces acting on
(i) a stationary car
(ii) the car moving at a constant velocity, v
(iii) the same car moving at a higher constant velocity, 2 v . (6
marks)
(e)

The graph above represents the motion of a car of mass 500 kg. The car
accelerates from rest to a velocity of 25 m s '.
(i) Define acceleration and use the graph to determine the acceleration of the
car. Calculate the net force acting on the car to produce this acceleration. (8
marks)
(ii) The 500 kg car travelling at 25 m s" now collides head-on with another of
mass
800 kg travelling at 18 m s -1 in the opposite direction and they move together
after impact What is the magnitude of their velocity just after impact and in
what direction do they move off?
(6 marks)
Total 20 marks

171

172
3.

The diagram below represents apparatus which can be used to investigate the
relation between the pressure and volume of a fixed mass of gas at a constant
temperature.

4 (a) Describe with the aid of a diagram, an investigation to measure the


critical angle of glass or perspex.
(8 marks)
(b)
An object is at the bottom of a water-filled tank. When it is viewed
from above it does not appear to be at its actual position. Draw a ray diagram
to illustrate this and state what is observed. (4 marks)

The figure above represents a ray of light leaving a light pipe through
the face Y Z. The light ray is incident at M at an angle of 68 to the
normal. Given that the critical angle is 42 for the glass-air interface,
calculate
(t) the refractive index of the glass
(a)
Describe how you might use this apparatus to carry out the
investigation.
How would you attempt to maintain a constant temperature
(6 marks)
(b) (i) A bottle is partially filled with hot water at 70 C and then corked.
Calculate the pressure of the air in the bottle when the water has cooled to 30
C. (Assume that the volume of air is constant.)
Atmospheric pressure = 100 kPa..
(4 marks)
(ii) Use the kinetic theory to explain what causes the pressure of the air in the
bottle and why it changes when the temperature is lowered.
(4 marks)
(c)
(i)
A weather balloon. made of rubber, is used to carry
instruments into the upper atmosphere. Why should it be only partially inflated
when it is released from the ground?
(ii)
Manufacturers recommend a specific pressure for car tyres to
ensure best performance. Explain why the pressure of the air in car tyres should
be checked only after the car has been driven for some distance. (6 marks)
Total 20
marks

(ii)

the angle of refraction at the face Y Z.

(8 marks)
Total 20
marks

5. (a) Describe. with the aid of a circuit diagram, how you could investigate the
relationship between current and potential difference fora small (24 W) filament
lamp, using a d.c. source. Sketch the graph of 1 against V that you would expect
to obtain from this investigation.
(8 marks)
(b)
Four 6 W parklights connected in parallel to the 12 V battery of a
motorcar are left on for a period of 8 hours. Calculate
(i) the total current drawn from the battery
(ii) the resistance of a single parklight what it is on
(iii) the total energy supplied by the battery.
(9 marks)
(iv) A 15 V battery charger is used to replace the energy supplied by the battery.
This provides a n e.m.f. of 3 V. If the charging current is 3 A. how long. would it
take to recharge the battery?
(3 marks)
Total 20 marks

172

173
6. (a)

(i)

The diagram below shows a Geiger-Muller tube.

Name the parts A. B and C and explain how the tube acts as a
detector of radioactive emissions.
( 6 marks)
(ii) Explain why a particles have a range of only a few centimetres in
air. ( 2 marks)
(iii) You are provided with a Geiger-Muller tuba and ratemeter and two unmarked
radio-active sources. One source emits only particles and the other emits
particles and rays. How would you differentiate between the two sources?
(5 marks)
(b) A sample of radioactive material was investigated by measuring its activity
(the number of disintegrations per minute) at intervals of one hour. The
results were plotted on a graph and the curve obtained is shown in Figure
I on page 7. Determine
(i) the time at which the activity was 3 x 10 4 min -1
(ii) the half-life of the material.
Use your answer to (ii) to calculate the time at which the activity would be 5 x 103
min-1.
(7 marks)
Total 20 marks

173

June 91 Paper 2

174
2.(a)

1.

Figure 2 above shows the apparatus used to observe, through a microscope, small
graphite particles suspended in water. The graphite is seen as bright points of light.
(i) Describe what would be seen through the microscope as the particles are
observed for a few seconds.
(2 marks)
(ii) If warmer water were used instead, what change in your observation would you
expect?
(1 mark)
(iii) What conclusions can be drawn from these observations?
(2 marks)

(i)

(ii)

(b)

How much energy does the refrigerator referred to in the


Energy guide above use per month?
(3 marks)
Assuming 1 month = 30 days, what is the average power
consumption (in watts) of the refrigerator?

The plug on the refrigerator has 3 pins. Name EACH pin.


(3 marks)
(ii) Although the refrigerator would work if a two-pin plug were used, explain
why the manufacturer insists that the third pin must not be cut off.
(2 marks)
(iii) Mothers often complain about their children wasting electricity by
repeatedly opening the refrigerator door because the cold air falls our. Using
the following data, calculate how much energy, in kilowatt hours, has to be
removed for the air in the refrigerator to be cooled from 30C to 5 C.
Volume of refrigerator
= 0.40 m3
Density of air
= 1.29 kg m-3
Specific beat capacity of air = 600 J kg -1 K -1
(5 marks)
1kWh
= 3.6x106J
Total 16 marks

(b)
The average speed of a perfume molecule at room temperature is about 100
metres per second. The speed at which the scent of perfume travels across a room
however, is much less than this. Explain this difference.
(3 marks)
(c)

Use the kinetic theory to explain why


(i)
(ii)

(i)

(d)

putting more air in a car tyre increases its pressure(2 marks)


the pressure in a car tyre increases when it is hot. (2 marks)

Nitrous oxide gas in a cylinder at 17 C exerts a pressure of 5.0 x 10 5 Pa.


Before it is used it is warmed to 37 C. What is the new pressure?

3.
(a)
motion.

Complete the following statements of Newtons 1st and 3rd laws of

1st Law: An object remains at rest ............................................................................

3rd Law: If an object A exerts a force on object B ....................................................


(4 marks)
(b)

A passenger travelling on a bus falls forward when the bus stops


suddenly. Account for this in terms of Newtons first law.
(2 marks)

174

175
(c)

The diagram below shows the Moon in its orbit around the Earth.
The Moon moves at constant speed.
(i)
Draw on the diagram any force or forces acting on the Moon.

4.

(a)

A nuclide of uranium-238 is represented by the symbol


238
92

(i)
(ii)

How many neutrons does this nuclide have?


How many protons does this nuclide have?...... (2 marks)
The mass number of the uranium nuclide is 238. Using the
same notation, an electron is represented by
0
1

Its mass number is O. Does this mean that the electron has zero
mass? Explain your answer.
......................................................................
......................................................................
......................................................................
(2 marks)
(b)
Uranium-238 dccays by emitting an alpha-particle to form thorium-234
(symbol Th). Complete the equation for this reaction.

(ii) How does Newtons third law apply in this case?

A ball is thrown vertically upwards. Figure 3


above shows the position of the ball at equal
intervals of time. At A it is moving upwards
and at B it has reached its highest point
(i) Draw on the diagram any force or
forces acting on the ball at points A
and B. IGNORING AIR
RESISTANCE.
(ii) State the magnitude of the
acceleration of the ball at A and at B.
At A: ...................................................................................................................
At B: ..................................................................................................................

U 234 Th + He

238
92

(2 marks) .
(c) The thorium-234 decays, in turn, by beta decay to form protactinium (symbol
Pa). Complete the equation for this decay.
234

Th Pa + e

A radioactive material decays with a half-life of 3 days.


(i)
Plot a graph to show how an initial mass of 8.00 g decays over a period of
15 days.
(5 marks)
(ii)
(iii)

How much of the radioactive material remains after 5 days? (1 mark)


If the pressure on the material were 10 times greater, how much would remain
after 9 days?
. (1mark) Total 16 marks

5.

175

176

June 1991 Paper 3


SECTION I
(a)

Label the parts A, B, C. D in the diagram of the eye above.


(4 marks)
The eye in Figure 4 above is long-sighted By drawing rays on the
diagram, show how the long sightedness is corrected with the use of the
converging lens.(4 marks)
(b) The spectacle lens used has a focal length of 80 cm. It produces a virtual
image of the object being viewed. If the object is 10 cm high and 40 cm
from the lens, find by completing the scale drawing below, the position
and magnification of the image.

(8 mark)
Total 16 marks

1.
The graph on the opposite page shows the current drawn by a small light
bulb for various applied p.d.s.
(a)
With the aid of a circuit diagram. Describe how these values might have been
obtained.
(4 marks)
(b)
Use the graph to find the resistance of the bulb when the p.d. is 0.70 V.(3
marks)
(c)
Using the graph, draw up a table of values of the current f and the resistance
R for at least eight different values of the p.d., V.
Plot a graph to show how the resistance of the light bulb varies with the
applied pd. (Attach this sheet to your answer booklet.)
What conclusion can you draw from the graph you have plotted? (20 marks)
(3
(d)The bulb is usually operated from a 2.8 V supply. Find its normal power.
marks)
Total 30 marks
SECTION II
Answer ANY THREE questions In this section.
2.(a) Velocity is a vector quantity but speed is a scalar. Explain what this
statement means, giving
TWO other examples of vectors and TWO examples of scalars.
(4 marks)
(b) The following extract is taken from a letter to a Barbados newspaper.
A truck with open platform, obviously intent on beating the light rushes
onto the highway and turns sharply to the left .... On the platform are about
a
dozen
concrete blocks.
I am sitting at the wheel of my car awaiting the green light. Well, the truck
duly makes its turn on to the highway but the concrete blocks ... have other
ideas ... they prefer to continue going straight ... towards me!
Fortunately, nearly all disintegrate into a powdery mess on the road, while one ...
crashes into my hubcap and wheel.
Mark on a diagram the direction of the trucks velocity at TWO different
instants. Hence, show that the truck is accelerating even though it has
constant speed. Add the direction of the acceleration to your diagram.
Explain why the blocks prefer to continue going straight.
(6 marks)

176

177
A helicopter heads due North and has an airspeed of 80 km h -1. The wind
is blowing to the North-East at 33 km h -1. By means of a scale drawing,
find the magnitude and direction of the helicopters velocity relative to the
ground.
How far does the helicopter travel in 15 minutes?
(10 marks)
Total 20 marks
3. (a)
Describe the processes by which energy is (i) produced in the sun, (ii)
transmitted to the earth
and (iii) trapped by the atmosphere (the glass-house effect).(8 marks)
(b) Each square metre of the earth receives energy from the sun, at the rate of
730W. A solar pane of area 12 m 2 is exposed to the sun. Calculate the amount of
energy incident on the panel in 6 hours.
What mass of water could it heat from 30 C to 80 C between 9 am and 3 pm. ?
(9 marks)
A number of assumptions have been made in order to simplify the problem at (b) above
State THREE of these assumptions. (3 marks)
(The specific heat capacity of water is 4.2 x 10 3 J kg -1 K -1. )
Total 20 mark
(a)

4.
(a) Describe, with the aid of diagrams where appropriate, observations
which lead us to believe that
(i)
Light has wave properties.
(ii) The wavelength of light is much smaller than the wavelength of sound.
(8 marks)
(b)
Plane sound waves travelling IN AIR are incident, at an angle, on a
flat water surface. Then wavelength is 5.0 m. The speed of sound in air is 300 m s
-1
, but in water it increases: to 1500m s -1.
Draw a diagram showing the approximate directions and wavelengths of the waves
as they cross the air-water boundary.
Calculate the frequency and wavelength of the sound waves in the water.
(7 marks)
(c)
The sound waves can only enter the water if the angle of incidence is
small enough. Account for this observation and calculate the maximum
size of the angle of incidence for the sound to enter the water.
(5
marks)
Total 20 mark

177

June 1990 Paper 2

178

1.
(a) Figure 1 represents a transverse wave on a string vibrating with a
frequency of 100 Hz.

(i)
(ii)

State the amplitude and wavelength of the wave.


Calculate the period and velocity of the wave. (3 marks)

(2 marks)

(b)The diagrams in Figure 2 above show series of plane wavefronts approaching (i) a
narrow gap and (ii) a wide gap. Complete the diagrams to show the wavefronts after
they have passed. d through EACH gap.
(4 marks)
(c) Figure 3 below shows plane electromagnetic wavefronts travelling in air with
velocity of 3.0 x 10 -8 m s -1, and meeting a plane refracting surface at an angle of
30. The wavelength of the waves in air is 4.0 x 10 -7 m and the velocity of the waves
in the second medium is 2.2 x 108 m.

Calculate:
(i) The frequency of the waves in air
(2 marks)
(ii) The wavelength of the waves in the second medium and the angle of refraction
(5 marks) Total 16 marks

178

179
2.
(a) A car engine converts chemical energy into mechanical energy. Even in
well designed alp however, the efficiency is not usually greater than 35%.
(i) Give the formula for calculating the efficiency of the engine. (2 marks)

(ii) A car uses gasolene of energy content 40 MJ per litre. If its engine is 30%
efficient how much useful work does it do for every litre of gasolene consumed? (2
marks)
(iii) The car, travelling at constant speed, experiences an average retarding force of
120N. How far can the car travel on 1 litre of gas?
(3 marks)
(b)
The diagram in Figure 4 below represents a hydraulic machine used for lifting
cars.

(iii)
How were the alpha-particles detected?( 1mark )
(iv)
Explain why most of the alpha-particles were able to pass through the foil
undeviated.
(v)
The Geiger-Marsden experiment supported a particular model of the atom. Name
ONE scientist who had proposed another model just before this one.
Briefly describe the model proposed before the one supported by the Geiger-Marsden
experiment.
(2 marks)
(b)
The following represents two nuclear reactions:

A small force applied at the small movable piston A, is capable of supporting a large
weight at the large piston B, for example a car on the hoist.
(i)
State the principle on which the action of this machine is based.
( 2 marks)
(ii)
Explain how the principle is applied to obtain a force great enough to
support a car.
(2 marks)
(iii)
A force of 480 N is applied to the piston, A, which has an area of 0.01 m2.
What is the resulting press= on the liquid?
(2 marks)
(iv)
What is the pressure on the large piston?
(1mark )
3 (a) In the Geiger-Marsden experiment represented in Figure 5 above, a stream of
alpha-particles, travelling in a vacuum, struck a piece of thin gold foil. The
behaviour of the alpha-particles was observed.
(i)
State the composition of an alpha-particle. (2 marks)
(ii)
Why was a vacuum necessary? ( 2 marks)

216
84

Po 212
82 Pb + X

212
82

Pb 212
83 Bi + Y

Name the emissions X and Y and write the symbols for them.
(4 marks)
(c)
The activity of a sample of a radioactive element decreases to 1/4 of its original
value in one hour.
Calculate the half-life of the sample.
(3 marks)
Total 16 marks

4. (a) (i) There is evidence that suggests that all matter is made up of particles. With reference to
this statement, draw diagrams to illustrate the difference between the structure of liquid and that of
gases.(3 marks)

(ii) Explain why gases are more easily compressed than liquids. (3 marks)

179

180
5 (a)

The graphs above represent the variation with time of the output current from an
alternating current (a.c.) generator and a dry cell in separate circuits.
(i) State the essential difference between direct current and alternating current
( 2 marks)
(ii) A split-ring commutator is now connected to the a.c. generator. On the axes below,
draw a graph to represent the output current assuming no other change in the circuit

Figure 6 above shows the experimental arrangement used to investigate how the
volume of a fixed mass of air varies with temperature. Readings were taken of the
length of the air column for a range of temperatures which was then converted to
kelvin temperatures.
(i)
Sketch the graph you would expect to obtain as a result of plotting the length
of the air column against the kelvin temperature, T.
(2 marks)
(ii)
State the 'law' which this experiment is used to investigate.
(iii)
Theory suggests that the volume of the gas becomes zero at a certain
temperature. What is the value of this temperature?
( 1 mark )
(c) A container used for storing compressed gas is fitted with a valve which opens
when the pressure is 1 x 104 Pa. The cylinder contains gas at 10 c and 8.5 x 10 5 Pa.
At what temperature will the valve open?
(5 marks)
Total 16 marks

(3 marks)
The commutator is removed and a p-n diode is placed in series with the a.c. generator.
(iii) Distinguish between the p-type material and n-type material of which the diode is
made.
(4 marks)
(iv) On the axes below, draw a graph of current against time to represent the output of
the a.c. genterator and diode

180

181

June 1990 P3
1. Some students performed an experiment to investigate the relationship between the
centripetal force and the time for one revolution for an object moving in a circle. A
diagram of the experimental arrangement is shown below.

(b) A 5.0 resistor has a maximum power rating of 5.0 W. It is connected to a d.c. power
supply.
(i) Calculate the maximum permissible current in the resistor.
(3 marks)
The resistor is then connected to an ac. supply and an alternating current flows with a peak
value equal to the maximum permissible current. The rate of energy conversion using a.c.
will be less than that using d.c. Explain this fact.
(1 mark)

A length of string was tied to a rubber bung and the other end passed through a
glass tube. Weights were then added to this end of the string. The radius of the
circular orbit was found to be 0.5 m and the bung was whirled in a horizontal
circle. The time taken for the rubber bung to complete 50 revolutions was recorded.
The procedure was repeated using a range of weights. The radius of the cirele was
kept constant throughout the experiment
Results of the experiment arc tabulated below:
Weight
W/N

Average time for


50 revolutions /s

Time for 1 revolution


T/s

T2/s2

1
T2 /s 2

1.42

0.50

42.0

0.840

0.706

0.75

36.8

0.736

0.542

1.00

31.6

0.632

0.399

1.25

28.5

0.570

0.325

1.50

26.8

0.536

0.287

1.75

25.5

0.510

0.260

181

182

(a) Calculate die missing values of 1/T2 and IN YOUR ANSWER BOOKLET
tabulate these values of 1/T2 along with the corresponding values of weight.(4
marks)
(b)
What provided the centripetal force needed for the hung to maintain its
circular path?
(2 marks)
(c) The students found the time for 50 revolutions in EACH case. What TWO
sources of error does this help to reduce?
( 2 marks)
(d) Plot a graph of weight. W (y axis) against 1/T2 (x axis). ( 9 marks)
(e) Calculate the slope of the graph.(5 marks)
(f) Theory shows that the slope = 4 2 mr, where m is the mass of the bung and r is
the radius of its orbit.
Use your value for the slope to find the mass of the bung.(4 marks)
(g)
How would you alter the procedure to investigate the relationship between
the radius of the orbit, r, and the time for one revolution. T? (4 marks)
Total 30
marks
SECTION II

2.

Answer any THREE questions in this section.


In a tape recorder (cassette player), an electric motor rotates the tape in the
cassette. The magnetic tape of the cassette is used to generate a current which
drives a loudspeaker.
(a)
Draw a diagram to represent the arrangement of the components in a
simple direct current motor and state the purpose of EACH component.
The tape is noticed to be rotating too slowly. Suggest TWO reasons why
this might occur.
(10 marks)
(b)
The tape is coated with a magnetic material which must be demagnetised
before it is used for recording. Suggest how this may be done.
(3 marks)
(c)
To record a song, a moving-coil microphone is plugged into the taperecorder. This connects the microphone to an electromagnet known as the
recording head. As the tape passes this head, it is magnetised.
Explain how the sound entering the microphone causes a varying magnetic
field in the recording head.
When the singing is louder, the magnetisation of the tape is greater. Briefly
explain this fact. (7 marks)

(i) The heater operates from a d.c. supply. Draw a diagram of a circuit which
may be used to determine the power supplied by the heater. What readings
should be taken to obtain a value for the specific heat capacity of the
copper? Show how these readings would be used.

(b)

(ii) The block is now replaced by one of equal mass, made of iron, which has a
larger specific heat capacity. What effect would this have on the
temperature rise if the other factors remain the same?
(10 marks)
Explain EACH of the following:
(i)
The engines of most motor cars arc water-cooled, whereas smaller engines
e.g. those of motor-cycles are usually air-cooled.
(ii)

The difference in the specific heat capacity of land and water gives rise to
land and sea breezes.

(iii)

When ice cubes at 0C arc added to a cup of hot coffee, they are more
effective in cooling the coffee than adding an equal mass of water at 0 C.
(10 marks)
Specific heat capacity of air 1000 J kg -1 K -1
Specific heat capacity of water
4 200 J kg -1 K -1
Total 20 marks
4.
(a)
The table below shows the values for frequency and wavelength of certain
waves, A, B and C, travelling on deep sea water.

3.

(a)

The diagram below shows part of the apparatus used to measure the
specific heat capacity of 1 kg copper block. An immersion heater and
a thermometer are inserted into holes drilled in the block.

C
6

Wavelength /m

10

15

Frequency /Hz

0.5

182

183
Calculate the speed of the waves in EACH case and state TWO ways in which the
speed of deep sea waves differs from that of electro-magnetic waves in a vacuum.
(5 marks)
(b)
A circular wave-front is created by dropping a stone into a lake. Suggest
TWO reasons why the amplitude of the wave-front decreases as it spreads out.
(3 marks)
(c) Sound waves are used in echo sounding to find the depth of the sea at different
places e.g. under oil-rigs. The sound wave sent out at the surface of the sea is
reflected from the sea bed and the time taken for the echo to return is measured.

6. (a) Define the moment of a force and state the principle of moments. A cardboard
lamina hangs freely from a horizontal nail with its plane vertical. What is the moment
of the weight of the lamina when in equilibrium? Give a reason for your answer.
Describe how this arrangement may be used to find the centre of gravity of the lamina.
(10 marks)

Near an oil-rig, the echo returns in 4.6 seconds. If the speed of sound in water is
(4 marks)
1500 m s -1, calculate the depth of the sea near this oil rig.
(d)
State TWO conditions necessary for total internal reflection to occur.
(e)
Draw a diagram showing sound waves sinking an air-water boundary at
the critical angle. (Represent the sound by a ray.)
Calculate the value of the critical angle.

(8 marks)

Speed of sound in air


300 m s-1
Speed of sound in water = 1 500 m s -1
Total 20 marks
5(a) During the 19th century, Michael Faraday conducted experiments on
electrolysis. He
investigated the factors which determine the masses of substances liberated.
Suggest TWO difficulties which Faraday might have experienced in attempting to
obtain reliable results.
(2 marks)
(b) A student wished to cover a metal coin with a coating of copper using the
process of electrolysis. Draw a clearly labelled diagram of die circuit he might
have used to achieve this. State TWO precautions which he should have taken to
obtain a good coating of copper.
(6 marks)
(c) In the process described above in (b) . a current of 0.1 A flowed for 12 minutes
and the potential difference between the electrodes was 2 V.
Calculate:
(i)
(ii)

The charge which has flowed round the circuit


The number of electrons which has flowed round the circuit

(iii)

The energy needed to move the electrons between the electrodes of the

voltameter.
Would you expect a temperature change to have occurred in the electrolyte?
Give a reason for your answer.

A UNIFORM plank of length 3 m is supported by two wooden trestles L and M as


shown in the diagram above. L exerts a force of 80 N upwards and M a force of 160 N
upwards. M is 0.5 m from the centre of gravity of the plank.
(i) Calculate the weight . W, of the plank and, by taking moments about the centre of
gravity, find the value of x .
(ii) A boy of weight 200 N walks along the plank from M towards T. How far from M
will he be when the plank just rises from L?
(10 marks)
Total 20 marks
7.
(a) The human eye is optically similar to a simple lens camera. State how EACH
achieves the following:
(i) Control of the amount of light catering
(ii) Formation of sharp images of objects at different distances
Where is the image formed in EACH case? Name TWO surfaces of the human eye at
which refraction of light occurs.
(8 marks)
(b)
An object is placed perpendicular to the principal axis of the converging lens in
a camera used for close-up photography. The object is 30 cm from the lens and an
inverted real image is formed at a distance of 10 cm from the lens.
Calculate the magnification of the image and, by means of a scale drawing on graph
paper. determine the focal length of the lens.(8 marks)
(c) The lens is removed from the camera and held at a distance of 4 cm from an ant of
length 2 mm
Use a scale diagram to find the length of the image of the ant. ( 4 marks

(12 marks)
Charge on electron = 1.6 x 10

-19

C)

Total 20 mark

183

You might also like